ORTHOPEDIC MCQS ONLINE BANK OITE 21
ORTHOPEDIC MCQS BANK ONLINE OITE 21
For OITE 21 FIGURES CLICK OITE21FIG
- 01.1 A 49-year-old man has a persistent Trendelenburg gait after undergoing open
- reduction and internal fixation of a posterior wall acetabular fracture 6 months
- ago. The radiographs reveal a normal joint space with no heterotopic ossification
- and no signs of osteonecrosis. Weakness in what muscle group is the most likely
- cause of his limp?
- 1- Gluteus maximus
- 2- Gluteus medius
- 3- Tensor fascia lata
- 4- Iliopsoas
- 5- Vastus lateralis
- answer
- Question 01.1
- Answer = 2
- back to this question
- next question
- Reference(s)
- Hoppenfeld S, deBoer P (eds): Surgical Exposures in Orthopaedics: The Anatomic Approach. Philadelphia, PA, Lippincott Williams 3c Wilkins, 1984, pp 353-386.
- 01.2 A 25-year-old patient undergoing arthroscopy is found to have a bucket-handle
- tear of the peripheral third of the anterior horn and body of the lateral meniscus.
- Management should consist of
- 1- meniscal trephination.
- 2- meniscal repair.
- 3- partial meniscectomy.
- 4- total meniscectomy.
- 5- resecting the anterior horn and repairing the body.
- answer
- back
- Question 01.2
- Answer = 2
- back to this question
- next question
- Reference(s)
- Fairbanks TJ: Knee joint changes after meniscectomy. J Bone Joint Surg Br 1948;30:664-670. Cannon WD Jr, Morgan CD: Meniscal repair. Arthroscopic Repair Techniques. Instr Course Lect 1994;43:77-96.
- 01.3 Which of the following findings is not typically observed after nonsurgical
- management of a calcaneal fracture?
- 1- Subtalar arthritis
- 2- Hindfoot valgus
- 3- Limb shortening
- 4- Subfibular peroneal impingement
- 5- Anterior ankle impingement
- answer
- back
- Question 01.3
- Answer = 2
- back to this question
- next question
- Reference(s)
- Carr JB, Hansen ST, Benirschke SK: Subtalar distraction bone block fusion for late complications of os calcis fractures. Foot Ankle 1988;9:81-86. Beaty JH (ed): Orthopaedic Knowledge Update 6. Rosemont, IL, American Academy of Orthopaedic Surgeons, 1999, pp 597-612.
- 01.4 A 60-year-old woman undergoes a bone scan because laboratory studies show
- an elevated alkaline phosphatase level. Results show increased uptake in
- multiple sites, including her skull. A lateral radiograph of the skull is shown in
- Figure la, and a CT scan is shown in Figure 1b. What is the most likely
- diagnosis?
- 1- Eosinophilic granuloma
- 2- Multiple myeloma
- 3- Lymphoma
- 4- Paget's disease
- 5- Polyostotic fibrous
- dysplasia
- answer
- back
- A
- B
- Figures 1
- Question 01.4
- Answer = 4
- back to this question
- next question
- Reference(s)
- Paget's disease, in McCarthy EF, Frassica FJ (eds): Pathology of Bone and Joint Disorders with Clinical and Radiographic Correlation. Philadelphia, PA, WB Saunders, 1998, pp 165-174. Mirra JM, Brien EW, Tehranzadeh 1: Paget's disease of bone: Review with emphasis on radiologic features. Part IL Skeletal Radiol 1995;24:173-184. Cooper KL: Radiology of metabolic bone disease. Endocrinol Metab Clin North Am 1989;18:955-976.
- 01.5 What is the single most important factor that determines the strength of a flexor
- tendon repair?
- 1- Caliber of the suture
- 2- Type of stitch configuration
- 3- Type of stitch material
- 4- Number of suture strands that cross the repair site
- 5- Use of an epitendinous suture
- answer
- back
- Question 01.5
- Answer = 4
- back to this question
- next question
- Reference(s)
- American Society for Surgery of the Hand: Hand Surgery Update. Rosemont, IL, American Academy of Orthopaedic Surgeons, 1996, pp 127-139. Noguchi M, Seller JG III, Gelberman RH, Sofranko RA, Woo SL: In vitro biomechanical analysis of suture methods for flexor tendon repair. J Orthop Res 1993 ;11:603-611.
- 01.6 What type of external loading produces a spiral fracture?
- 1- Tension
- 2- Torsion
- 3- Bending
- 4- Compression
- 5- Shear
- answer
- back
- Question 01.6
- Answer = 2
- back to this question
- next question
- Reference(s)
- Buckwalter JA, Einhorn TA, Simon SR (eds): Orthopaedic Basic Science: Biology and Biomechanics of the Musculoskeletal System, ed 2. Rosemont, IL, American Academy of Orthopaedic Surgeons, 2000, pp 372-399.
- 01.7 A 55-year-old woman with chronic posterior tibial tendon deficiency has a
- painful planovalgus foot. Examination reveals that neither hindfoot valgus nor
- forefoot rotation is passively correctable. Management consisting of shoe wear
- modifications and use of an insert has failed to relieve her pain. Treatment
- should now consist of
- 1- reconstruction of the posterior tibial tendon and a medial displacement calcaneal
- osteotomy.
- 2- rotational osteotomy of the talar neck and reconstruction of the spring ligament.
- 3- triple arthrodesis and lengthening of the Achilles tendon.
- 4- arthrodesis of the ankle (tibiotalar) joint and a medial displacement calcaneal
- osteotomy.
- 5- subtalar fusion and transfer of the anterior tibialis tendon to the medial navicular.
- answer
- back
- Question 01.7
- Answer = 3
- back to this question
- next question
- Reference(s)
- Mizel MS, Miller RA, Scioli MW (eds): Orthopaedic Knowledge Update: Foot and Ankle 2. Rosemont, 1L, American Academy of Orthopaedic Surgeons, 1998, pp 253-277. Mann RA: Flatfoot in adults, in Coughlin MJ, Mann RA (eds): Surgery of the Foot and Ankle, ed 7. St Louis, MO, Harcourt Health Science, 1999, pp 733-767.
- 01.8 After plating of a both-bone forearm fracture in an adult, which of the
- following actions is most important to achieve maximal forearm rotation?
- 1- Early motion
- 2- Surgery within 7 days
- 3- Restoration of the radial bow
- 4- Compression fixation of both bones
- 5- Repair of the interosseous ligament
- answer
- back
- Question 01.8
- Answer = 3
- back to this question
- next question
- Reference(s)
- Schemitsch EH, Richards RR: The effect of malunion on functional outcome after plate fixation of fractures of both bones of the forearm in adults. J Bone Joint Surg Am 1992;74:1068-1078.
- 01.9 A 13-year-old football player who is scheduled to attend summer football camp
- reports continued popping of the ankle joint. History reveals that the patient
- sustained an ankle sprain last season that was managed with taping. A radiograph
- and MRI scan are shown in Figures 2a and 2b. The next step in management
- should consist of
- 1- an ankle brace.
- 2- an incisional biopsy of
- the talus.
- 3- a total body bone scan
- and radiographs directed
- at abnormal areas.
- 4- a CBC and serum alkaline
- phosphatase, calcium,
- and phosphorus levels.
- 5- arthroscopic examination and possible drilling.
- answer
- back
- A
- B
- Figures 2
- Question 01.9
- Answer = 5
- back to this question
- next question
- Reference(s)
- Gross RH: Fractures and dislocations of the foot, in Rockwood CA, Wilkins KE, Beaty JH (eds): Fractures in Children, ed 4. Philadelphia, PA. Lippincott-Raven, 1996, pp 1429-1497. Crawford AH: Fractures and dislocations of the foot and ankle, in Green NE, Swiontkowski NF (eds): Skeletal Trauma in Children, ed 2. Philadelphia, PA. WB Saunders, 1998, pp 431-458. Higuera J, Laguna R, Peral M, Aranda E, Soleto J: Osteochondritis dissecans of the talus during childhood and adolescence. J Pediatr Orthop 1998;18:328-332. Kumai T, Takakura Y, Higashiyama I, Tamai S: Arthroscopic drilling for the treatment of osteochondral lesions of the talus. 1 Bone Joint Surg Am 1999;81:1229-1235.
- 01.10 In the presence of an epidural abscess, which of the following findings is
- considered the most consistent indication for surgery?
- 1- Deteriorating neurologic condition
- 2- Elevated erythrocyte sedimentation rate
- 3- Elevated C-reactive protein
- 4- Anterior vertebral body bone loss
- 5- Posterior vertebral body bone loss
- answer
- back
- Question 01.10
- Answer = 1
- back to this question
- next question
- Reference(s)
- Carragee EJ, Kim D, van der Vlugt T, Vittum D: The clinical use of erythrocyte sedimentation rate in pyogenic vertebral osteomyelitis. Spine 1997;22:2089-2093. Ozuna RM, Delamarter RB: Pyogenic vertebral osteomyelitis and postsurgical disc space infections. Orthop Clin North Am 1996;27:87-94.
- 01.11 A 62-year-old construction worker with a full-thickness supraspinatus tear that
- extends 2.0 cm across its insertion underwent repair using the suture-to-bone
- technique. Two weeks after surgery, physical therapy should consist of
- 1- strict passive motion of the arm.
- 2- iontophoresis and scar mobilization.
- 3- lifting the weight of the arm only, while standing.
- 4- lifting weights that do not exceed 5 lb.
- 5- passive internal and external rotation only.
- answer
- back
- Question 01.11
- Answer = 1
- back to this question
- next question
- Reference(s)
- Brems 1J: Rotator cuff tears: Mobilization and tissue coverage, in Craig EV (ed): . The Shoulder. New York, NY, Raven Press, 1995, pp 35-54. Cofield RH: Rotator cuff disease of the shoulder. J Bone Joint Sur- Am 1985;67:974-979.
- 01.12 A 45-year-old woman with systemic lupus erythematosus who underwent total
- hip arthroplasty 6 months ago has recurrent instability despite well-positioned
- components. She has a 28-mm head, and the operative leg is 0.5 cm longer than
- the contralateral leg. Her most recent dislocation occurred while in a brace.
- Appropriate management should now should consist of
- 1- revision to a constrained acetabular component.
- 2- revision to a longer femoral neck.
- 3- revision to a larger femoral head.
- 4- trochanteric advancement.
- 5- application of an abduction brace for 3 months.
- answer
- back
- Question 01.12
- Answer = 4
- back to this question
- next question
- Reference(s)
- Callaghan 1J, Dennis DA, Paprosky WG, Rosenberg AG (eds): Orthopaedic Knowledge Update: Hip and Knee Reconstruction. Rosemont, IL, American Academy of Orthopaedic Surgeons, 1995, pp 163-170.
- 01.13 What process most closely resembles the radiographic and histologic
- presentations of multicentric giant cell tumor?
- 1- Paget's disease
- 2- Campanacci's disease (osteofibrous dysplasia)
- 3- Ollier's disease (enchondromatosis)
- 4- Transient osteoporosis
- 5- Hyperparathyroidism
- answer
- back
- Question 01.13
- Answer = 5
- back to this question
- next question
- Reference(s)
- Cummins CA, Scarborough MT, Enneking WF: Multicentric giant-cell tumor of bone. Clin Orthop 1996;322:245-252. Sim FH, Dahlin DC, Beabout JW: Multicentric giant-cell tumor of bone. J Bone Joint Surg Am 1977;59:1052-1060.
- 01. 14 A 25-year-old man involved in a
- motorcycle accident sustained the
- pelvic injury shown in Figure 3a.
- He underwent an emergent
- laparotomy, and the anterior pelvis
- was reduced and fixed. The
- postoperative AP radiograph is
- shown in Figure 3b. The next step
- in management should consist of
- 1- weight bearing as tolerated on both
- lower legs.
- 2- touchdown weight bearing on the right
- lower leg.
- 3- supplemental external fixation.
- 4- reduction and stabilization of the
- posterior pelvic ring.
- 5- a sacroiliac fusion.
- answer
- back
- A
- B
- Figures 3
- Question 01.14
- Answer = 4
- back to this question
- next question
- Reference(s)
- Kellarn JF, Fischer TJ, Tornetta P III, Bosse MJ, Harris MB (eds): Orthopaedic Knowledge Update: Trauma 2. Rosemont, IL, American Academy of Orthopaedic Surgeons, 2000, pp 229-237. Matta JM, Tornetta P III: Internal fixation of unstable pelvic ring injuries. Clin Orthop 1996;329:129-140.
- 01.15 The occurrence of restricted passive flexion of the proximal interphalangeal
- joint during extension but not flexion of the metacarpophalangeal joint is most
- indicative of
- 1- flexor tendon adhesions.
- 2- extensor tendon adhesions.
- 3- intrinsic contracture.
- 4- quadrigia effect.
- 5- a lumbrical plus finger.
- answer
- back
- Question 01.15
- Answer = 3
- back to this question
- next question
- Reference(s)
- Bunnell S, Doherty EW, Curbs RM: Ischemic contracture, local, in the hand. Plast Reconstr Surg 1948;4:424-433. Smith RJ: Non-ischemic contractures of the intrinsic muscles of the hand J Bone Joint Sur- Am 1971;53:1313-1331.
- 01.16 A 6-year-old girl underwent correction of a left clubfoot deformity with serial
- casting during infancy. Her parents report that she often trips and walks on the
- lateral border of her left foot. Examination reveals a dynamic intoeing gait on
- the left side. Active range of motion of the left ankle shows 30° of plantar
- flexion and 10° of dorsiflexion. The hindfoot is well corrected, and there is only
- mild residual forefoot adductus. Management should consist of
- 1- repeat serial casting until symmetric dorsiflexion is obtained.
- 2- a ground reaction short leg brace.
- 3- opening medial cuneiform and closing cuboid osteotomies.
- 4- posterior tibialis transfer through the interosseous membrane with anastomosis to
- the anterior tibialis and the peroneus brevis tendons.
- 5- anterior tibialis tendon transfer to the lateral cuneiform.
- answer
- back
- Question 01.16
- Answer = 5
- back to this question
- next question
- Reference(s)
- Cooper DM, Dietz FR: Treatment of idiopathic clubfoot: A thirty-year follow-up note. J Bone Joint Surg Am 1995;77:1477-1489. Garceau GJ: Anterior tibial transposition in recent congenital clubfoot. J Bone Joint Surg Am 1990;22:932.
- 01.17 What spinal nerve roots make up the long thoracic nerve?
- 1- C1-T1
- 2- C4-C6
- 3- C5-C6
- 4- C5-C7
- 5- C6-C8
- answer
- back
- Question 01.17
- Answer = 4
- back to this question
- next question
- Reference(s)
- Jenkins DB: Hollinsheads Functional Anatomy of the Limbs and Back, ed 6. Philadelphia. PA. WB Saunders, 1991, p21 1.
- 01.18 Neuromuscular strength training in female high school athletes results in
- 1- a lower incidence of knee injuries when compared with untrained male high
- school athletes.
- 2- a lower incidence of knee injuries when compared with trained high school male
- athletes.
- 3- decreased injury rates when compared with untrained female high school athletes.
- 4- increased injury rates when compared with untrained female high school athletes.
- 5- no significant differences in injury rates between trained and untrained female
- high school athletes.
- answer
- back
- Question 01.18
- Answer = 3
- back to this question
- next question
- Reference(s)
- Hewett TE, Lindenfeld TN, Riccobene IV, Noyes FR: The effect of neuromuscular training on the incidence of knee injury in female athletes: A prospective study. Am. J Sports Med 1999;27:699-706. Arendt E, Dick R: Knee injury patterns among men and women in collegiate basketball and soccer. NCAA data and review of literature. Am J Sports Med 1995;23:694-701.
- 01.19 A sedentary 30-year-old man is evaluated for increasing pain near the insertion
- of the Achilles tendon. He denies changes in his activity level. History reveals
- that the patient is being treated with ciprofloxacin for chronic sinusitis and
- seasonal allergies and. Initial management should consist of
- 1- debridement of the Achilles tendon.
- 2- a below-knee cast.
- 3- a corticosteroid injection.
- 4- discontinuation of the fluoroquinolone antibiotics.
- 5- iontophoresis.
- answer
- back
- Question 01.19
- Answer = 4
- back to this question
- next question
- Reference(s)
- Movie T, Gad A, Gunter P, Foldhazy Z, Rolf C: Pathology of the Achilles tendon in association with ciprofloxacin treatment. Foot Ankle Int 1997;18:297-299. McGarvey WC, Singh D, Trevino SG: Partial Achilles tendon ruptures associated with fluoroquinolone antibiotics: A case report and literature review. Foot Ankle Int 1996;17:496-498.
- 01.20 A 28-year-old construction worker was treated with an unreamed nail for a grade
- III open tibial fracture 9 months ago. He has been unable to work because of
- persistent lateral leg pain. Examination reveals posterolateral tenderness. Current
- AP and lateral radiographs are shown in Figures 4a and 4b. Management at this
- time should consist of
- 1- observation.
- 2- exchange nailing of the tibia.
- 3- dynamization of the tibial nail.
- 4- open reduction and internal
- fixation with bone graft of
- the fibula.
- 5- posterolateral tibiofibular
- synostosis.
- answer
- back
- A
- B
- Figures 4
- Question 01.20
- Answer = 4
- back to this question
- next question
- Reference(s)
- Ebraheim NA, Savolaine ER, Skie MC, Jackson WT: Fibular nonunion in combination with fractures of the tibia. Orthopedics 1993;16:1229-1232.
- 01.21 A 17-year-old boy with knee effusions is diagnosed with Lyme disease. What is
- the current recommended treatment?
- 1- Tetracycline
- 2- Amoxicillin
- 3- Cephalexin
- 4- Ciprofloxacin
- 5- Imipenem
- answer
- back
- Question 01.21
- Answer = 2
- back to this question
- next question
- Reference(s)
- Beaty JH (ed): Orthopaedic Knowledge Update 6. Rosemont, IL, American Academy of Orthopaedic Surgeons, 1999, pp 191-203. Sigal LH: Antibiotics for the treatment of rheumatologic syndromes. Rheum Dis Clip North Am 1999;25:861-881.
- 01.22 What is the most accurate way to diagnose pseudarthrosis following lumbar
- fusion with pedicle screw instrumentation?
- 1- Surgical exploration
- 2- Diskography
- 3- Plain radiography that includes flexion-extension lateral views
- 4- Bone scan
- 5- Computed tomography
- answer
- back
- Question 01.22
- Answer = 1
- back to this question
- next question
- Reference(s)
- Larsen 1M, Capen DA: Pseudarthrosis of the lumbar spine. J Am Acad Orthop Surg 1997;5:153-162. Larsen JM, Rimoldi RL, Capen DA, Nelson RW, Nagelberg S, Thomas JC Jr: Assessment of pseudarthrosis in pedicle screw fusion: A prospective study comparing plain radiographs, flexion/extension radiographs, CT scanning, and bone scintigraphy with operative findings. J Spinal Disord 1996;9:117-120.
- 01.23 A 14-year-old child sustained a Salter type II fracture of the distal femur 6
- months ago. Follow-up examination reveals that the fracture has healed in good
- position, but it appears that there may be a partial growth plate closure. An MRI
- scan shows 60% obliteration of the physis. The next most appropriate step in
- management should consist of
- 1- physeal bar resection.
- 2- obtaining a scanogram now and in 6 months to construct a Moseley growth chart.
- 3- femoral lengthening.
- 4- contralateral distal femoral epiphyseodesis.
- 5- bilateral distal femoral epiphyseodesis.
- answer
- back
- Question 01.23
- Answer = 5
- back to this question
- next question
- Reference(s)
- Moseley CF: Leg length discrepancy and angular deformity of the lower limbs, in Morrissy RT, Weinstein SL (eds): Lovell and Winter's Pediatric Orthopaedics, ed 4. Philadelphia, PA, Lippincott-Raven, 1996, pp 849-896. Atar D, Lehman WB, Grant AD, Strongwater A: Percutaneous epiphysiodesis. J Bone Joint Sur-, Br 1991;73:173. Blair VP III Walker S1, Sheridan JJ, Schoenecker PL: Epiphysiodesis: A problem of timing. J Pediatr Orthop 1982:281-284.
- 01.24 Figures 5a and 5b show the radiographs of a 53-year-old woman who sustained a
- Mason type I radial head fracture 2 years ago. Management consisting of
- physical therapy and static bracing has failed to improve elbow motion.
- Examination reveals an elbow flexion arc of 60° to 110°, and forearm rotation is
- unrestricted. Management should now consist of
- 1- manipulation under
- anesthesia.
- 2- radial head resection.
- 3- distal biceps
- lengthening.
- 4- capsular contracture
- release.
- 5- a corticosteroid
- injection.
- answer
- back
- A
- B
- Figures 5
- Question 01.24
- Answer = 4
- back to this question
- next question
- Reference(s)
- Breen TF, Gelberman RH, Ackerman GN: Elbow flexion contractures: Treatment by anterior release and continuous passive motion. J Hand Surg Br 1988;13:286-287. Gates HS III, Sullivan FL, Urbaniak 1R: Anterior capsulotomy and continuous passive motion in the treatment of post-traumatic flexion contracture of the elbow. J Bone Joint Surg Am 1992;74:1229-1234.
- 01.25 Item deleted after statistical review
- (and no answer or references cited)
- back
- next question
- 01.26 Initial management of severe frostbite of the finger should consist of
- 1- rubbing the hands together over a heat source and the application of compressive
- bandages.
- 2- rapid rewarming of the frozen extremity in a 104°F to 111.2°F (40°C to 44°C)
- bath.
- 3- allowing the extremity to thaw at room temperature and providing warm liquids
- by mouth to raise the patient's core temperature.
- 4- plunging the extremity into cold water baths of increasing temperatures.
- 5- intra-arterial infusion of warmed, saline-diluted blood products.
- answer
- back
- Question 01.26
- Answer = 2
- back to this question
- next question
- Reference(s)
- Washburn B: Frostbite: What it is, how to prevent it and emergency tent. N Eng J Med 1962;266:974-989. Pedowitz WJ: Soft tissue disorders of the foot, in Coughlin MJ, Mann RA (eds): Surgery of the Foot and Ankle, ed 7. St Louis, MO, Harcourt Health Science, 1999, pp 1373-1397.
- 01.27 Which of the following is considered a typical feature of a T-type acetabular
- fracture?
- 1- Disruption of the iliac wing
- 2- Disruption of the obturator ring
- 3- A spur sign
- 4- Secondary congruence
- 5- Predominantly anterior column displacement
- answer
- back
- Question 01.27
- Answer = 2
- back to this question
- next question
- Reference(s)
- Letournel E, Judet R: T-Shaped fractures, in Fractures of the Acetabulum. Springer-Verlag, 1981, pp 163-179. Saterbak AM, Marsh JL, Brandser E, Turbett T: Acetabular fractures classification of Letournel and Judet: A systematic approach. Iowa Orthop J 1995;15:184-196.
- 01.28 A 44-year-old woman has a painful bunion. Nonsurgical management and shoe
- wear modifications have failed to provide relief. Examination reveals a
- hypermobile first ray and generalized ligamentous laxity. Radiographs do not
- show any evidence of arthrosis. Treatment should now consist of a
- 1- phalangeal (Akin) osteotomy.
- 2- distal soft-tissue procedure.
- 3- distal metatarsal osteotomy with an Akin osteotomy.
- 4- first tarsometatarsal arthrodesis (Lapidus) with a distal soft-tissue procedure.
- 5- first metatarsophalangeal arthrodesis.
- answer
- back
- Question 01.28
- Answer = 4
- back to this question
- next question
- Reference(s)
- Mizel MS, Miller RA, Scioli MW (eds): Orthopaedic Knowledge Update: Foot and Ankle 2. Rosemont, IL, American Academy of Orthopaedic Surgeons, 1998, pp 123-134. Hansen ST: The dysfunctional forefoot, in Functional Reconstruction of the Foot and Ankle. Philadelphia, PA, JB Lippincott, 2000, pp 215-226.
- 01.29 Figures 6a and 6b show the
- radiographs of a male infant.
- What is the most likely diagnosis?
- 1- Larsen's syndrome
- 2- Fragile X syndrome
- 3- Marfan syndrome
- 4- Stickler syndrome (hereditary arthro-
- ophthalmopathy)
- 5- Contractural arachnodactyly
- answer
- back
- A
- B
- Figures 6
- Question 01.29
- Answer = 1
- back to this question
- next question
- Reference(s)
- Laville JM, Lakermance P, Limouzy F: Larsen's syndrome: Review of the literature and analysis of thirty-eight cases. J Pediatr Orthop 1994;14:63-73. Letter LD: Larsen syndrome: Clinical features and treatment. A report of two cases. J Pediatr Orthop 1990;10:270-274.
- 01.30 A 22-year-old Olympic sprinter pulls up halfway through a race and grabs his
- midthigh. What is the most likely site of hamstring injury?
- 1- Muscle belly
- 2- Tendon-bone insertion distally
- 3- Pelvic origin
- 4- Musculotendinous junction
- 5- Midsubstance of the distal tendon
- answer
- back
- Question 01.30
- Answer = 4
- back to this question
- next question
- Reference(s)
- Nicholas J, Henshman E: The Lower Extremity and Spine in Sports Medicine, ed 2. St Louis, MO, Mosby, 1995, pp 999-1023. Garrett WE Jr, Rich FR, Ndcolaou PK, Vogler 1B III: Computed tomography of hamstring muscle sprains. Med Sci Spore Exerc 1989;21:506-514.
- 01.31 A 30-year-old man sustained multiple injuries in a motor vehicle accident,
- including bilateral femoral shaft fractures, a pelvic ring fracture, and a floating
- right elbow. Following stabilization of the life-threatening injuries, treatment of
- the right humerus should consist of
- 1- closed reduction and application of a cast.
- 2- closed reduction and placement of a reamed humeral nail.
- 3- closed reduction and percutaneous pinning.
- 4- open reduction and internal fixation.
- 5- application of an external fixation device.
- answer
- back
- Question 01.31
- Answer = 4
- back to this question
- next question
- Reference(s)
- Johnson KD: Management of fractures of the femur, tibia, and upper extremity in the multiply injured patient. Instr Course Lect 1990;39:565-576. Dabezies FJ, Banta CJ II, Murphy CP, d'Ambrosia RD: Plate fixation of the humeral shaft for acute fractures, with and without radial nerve injuries. J Orthop Trauma 1992;6:10-13.
- 01.32 A 62-year-old man is scheduled to undergo a total hip arthroplasty. History
- reveals that he underwent radiation therapy for a pelvic malignancy 1 year ago.
- A cementless acetabular component should be avoided in this situation because
- of the increased risk for
- 1- osteolysis.
- 2- acetabular fracture.
- 3- aseptic loosening.
- 4- infection.
- 5- recurrent tumor.
- answer
- back
- Question 01.32
- Answer = 3
- back to this question
- next question
- Reference(s)
- Massin P, Duparc J: Total hip replacement in irradiated hips: A retrospective study of 71 cases. J Bone Joint Surg Br 1995;77:847-852.
- 01.33 Which of the following terms best describes the primary motion at C1-2?
- 1- Flexion
- 2- Rotation
- 3- Lateral bend
- 4- Translation
- 5- Extension
- answer
- back
- Question 01.33
- Answer = 2
- back to this question
- next question
- Reference(s)
- Garfin SR, Vaccaro AR (eds): Orthopaedic Knowledge Update: Spine. Rosemont, IL, American Academy of Orthopaedic Surgeons, 1997, pp 3-17.
- 01.34 The number of polymer chain cross-links may be increased in ultra-high
- molecular weight polyethylene bearing surfaces by
- 1- annealing.
- 2- remelting.
- 3- high-pressure molding.
- 4- gamma irradiation.
- 5- ethylene oxide.
- answer
- back
- Question 01.34
- Answer = 4
- back to this question
- next question
- Reference(s)
- Beaty JH (ed): Orthopaedic Knowledge 6. Rosemont, IL, American Academy of Orthopaedic Surgeons, 1999, pp 47-53.
- 01.35 In Figures 7a and 7b, what letter marks the site of origin of the medial collateral
- ligament of the elbow?
- 1- A
- 2- B
- 3- C
- 4- D
- 5- E
- answer
- back
- A
- B
- Figures 7
- Question 01.35
- Answer = 3
- back to this question
- next question
- Reference(s)
- Money BF: Anatomy of the elbow joint, in Money BF (ed): The Elbow and Its Disorders, ed 2. Philadelphia, PA, WB Saunders, 1993, pp 16-52.
- 01.36 A 23-year-old woman who was struck by a motor vehicle has a Glasgow Coma
- Scale score of 15 and a systolic blood pressure of 110 mm Hg on arrival in the
- emergency department. Examination reveals a rotationally unstable pelvic ring
- injury and multiple long bone fractures. Initial management consists of IV
- administration of 3 L of lactated Ringer's solution over 2 hours. Reexamination
- now reveals that the patient is tachycardic and her systolic blood pressure has
- dropped to 60 mm Hg. What is the most likely cause of this event?
- 1- Insufficient fluid resuscitation
- 2- Unrecognized spinal trauma
- 3- Unrecognized brain injury
- 4- Myocardial infarction
- 5- Failure to stabilize the pelvic ring injury
- answer
- back
- Question 01.36
- Answer = 1
- back to this question
- next question
- Reference(s)
- Browner BD, Jupiter JB, Levine AM, Trafton PB (eds): Skeletal Trauma, ed 2. Philadelphia, PA. WB Saunders, 1998, pp 141-148.
- 01.37 A 10-year-old girl with chronic glomerulonephritis is a candidate for a renal
- transplant. She has progressive genu valgum and has begun to have pain in the
- hips bilaterally. She has an antalgic, wide-based, and externally rotated gait. Hip
- range of motion with internal rotation is painful and markedly limited. Which of
- the following studies will best define the source of hip pain?
- 1- Blood urea nitrogen and serum creatinine levels
- 2- Serum calcium, phosphorus, and 1,25-dihydroxyvitamin D levels
- 3- AP and frog-lateral radiographs of the hips
- 4- Arthrogram of the hips
- 5- CT scan of the pelvis
- answer
- back
- Question 01.37
- Answer = 3
- back to this question
- next question
- Reference(s)
- Loder RT, Hensinger RN: Slipped capital femoral epiphysis associated with renal failure osteodystrophy. J Pediatr Orthop 1997;17:205-211. Loder RT, Aronsson DD, Dobbs MB, Weinstein SL: Slipped capital femoral epiphysis. Instr Course Lect 2001;50:555-570.
- 01.38 In the initial stages of Charcot-Marie-Tooth disease (hereditary motor-sensory
- neuropathy), a forefoot cavus deformity may develop as the result of an
- imbalance between which of the following muscles?
- 1- Posterior tibialis and peroneus longus
- 2- Tibialis anterior and gastrocnemius-soleus complex
- 3- Tibialis anterior and peroneus longus
- 4- Flexor digitorum longus and extensor digitorum longus
- 5- Abductor hallucis and quadratus plantae
- answer
- back
- Question 01.38
- Answer = 3
- back to this question
- next question
- Reference(s)
- Mann R: Pes caves, in Coughlin MJ, Mann RA (eds): Surgery of the Foot and Ankle, ed 7. St Louis, MO, Harcourt Health Science, 1999, pp 768-783. Holmes JR, Hansen ST Jr- Foot and ankle manifestations of Charcot-Marie-Tooth disease. Foot Ankle 1993;14:476-486. Alexander U, Johnson KA: Assessment and management of pes caves in Charcot-Marie-Tooth disease. Clip Orthop 1989;246:273-281.
- 01.39 A basketball player sustained blunt trauma to his long finger. Examination
- reveals swelling and tenderness at the proximal interphalangeal (PIP) joint. He
- is able to extend the digit and flex the PIP and distal interphalangeal (DIP)
- joints. When the PIP joint is bent 90° over the edge of the table and middle
- phalanx extension is manually blocked, the DIP joint goes into rigid extension
- when the patient attempts to extend his finger. Management should consist of
- 1- surgical repair of the oblique retinacular ligament.
- 2- surgical repair of the central slip.
- 3- a dynamic extension outrigger splint.
- 4- buddy taping and early active motion.
- 5- extension splinting of the PIP joint.
- answer
- back
- Question 01.39
- Answer = 5
- back to this question
- next question
- Reference(s)
- Coons MS, Green SM: Boutonniere deformity. Hand Clin 1995;11:387-402. Elson RA: Rupture of the central slip of the extensor hood of the finger-. A test for early diagnosis. J Bone Joint Surg Br 1986:68:229-231.
- 01.40 A 40-year-old woman who abuses alcohol and drugs reports increasing pain in
- the right sternoclavicular region for the past 3 weeks. She has no fever, and
- clinical examination of the region is normal except for pain. Plain radiographs
- are normal. Laboratory studies show that the erythrocyte sedimentation rate is
- moderately elevated and the CBC and WBC are normal. What is the most likely
- diagnosis?
- 1- Osteoarthritis of the sternoclavicular joint
- 2- Septic arthritis of the sternoclavicular joint
- 3- Osteolysis of the medial clavicle
- 4- Instability of the sternoclavicular joint
- 5- Idiopathic hyperostosis of the sternoclavicular joint
- answer
- back
- Question 01.40
- Answer = 2
- back to this question
- next question
- Reference(s)
- Rockwood CA Jr, Wirth MA (eds): Injuries to the sternoclavicular joint, in Rockwood CA, Green DP (eds): Fractures in Adults. Philadelphia, PA, Lippincott Raven, 1996, pp 1415-1470. Wirth MA, Rockwood CA: Chronic conditions of the AC and SC joints, in Chapman MW (ed): Operative Orthopaedics. Philadelphia, PA, JB Lippincott, 1993, pp 1683-1693.
- 01.41 The beneficial effects of aspirin in preventing thromboembolic disease are the
- result of
- 1- reversible inhibition of cyclooxygenase.
- 2- activation of the fibrinolytic system.
- 3- binding to antithrombin.
- 4- inhibition of thromboxane-A2 synthesis.
- 5- replacement of normal clotting factors with decarboxylated factors.
- answer
- back
- Question 01.41
- Answer = 4
- back to this question
- next question
- Reference(s)
- Buckwalter JA, Einhorn TA, Simon SR (eds): Orthopaedic Basic Science: Biology and Biomechanics of the Musculoskeletal System, ed 2. Rosemont, IL, American Academy of Orthopaedic Surgeons, 2000, pp 218-237.
- 01.42 An otherwise healthy postpartum 30-year-old woman has had a painful left hip
- for the past 8 weeks. The onset of hip pain occurred at the end of her third
- trimester of pregnancy. Examination reveals that the hip is moderately irritable
- on range of motion. Plain radiographs of the hip are normal. The most
- productive diagnostic work-up should consist of
- 1- a hip aspiration.
- 2- a bone scan.
- 3- MRI.
- 4- an obturator nerve block.
- 5- a stress hip radiograph.
- answer
- back
- Question 01.42
- Answer = 3
- back to this question
- next question
- Reference(s)
- Montella BJ, Nunley JA, Urbaniak 3R: Osteonecrosis of the femoral head associated with pregnancy: A preliminary report. J Bone Joint Surg Am 1999;81:790-798.
- 01.43 A 6-month-old girl has a congenital right foot deformity that has failed to
- respond to serial casting. An AP radiograph and a T1-weighted MRI scan are
- shown in Figures 8a and 8b. Management should consist of
- 1- continued serial casting.
- 2- an abductor hallucis tenotomy.
- 3- proximal osteotomy of
- the first through fifth
- metatarsals.
- 4- excision of the
- metatarsal bracket with
- placement of
- interposition material.
- 5- lengthening of the first
- metatarsal with dynamic
- external fixation.
- answer
- back
- A
- B
- Figures 8
- Question 01.43
- Answer = 4
- back to this question
- next question
- Reference(s)
- Mubarak SJ, O'Brien TJ, Davids JR:- Metatarsal epiphyseal bracket: Treatment by central physiolysis. J Pediatr Orthop 1993:13:5-8. Light TR, Ogden JA: The longitudinal epiphyseal bracket: Implications for surgical correction. J Pediatr Orthop 1981;1299-305.
- 01.44 A patient reports groin pain and has painful range of motion of the hip 3 weeks
- after undergoing locked intramedullary nailing of a comminuted femoral shaft
- fracture. The first step in assessment should consist of obtaining which of the
- following imaging studies?
- 1- Radiographs of the femur
- 2- Radiographs of the femoral neck
- 3- Radiographs of the acetabulum
- 4- MRI
- 5- Bone scan
- answer
- back
- Question 01.44
- Answer = 2
- back to this question
- next question
- Reference(s)
- Peljovich AE. Patterson BM: Ipsilateral femoral neck and shaft fractures. J Am Azad Orthop Sur- 1998;6:106-113. Riemer BL, Butterfield SL, Ray RL, Daffner RH: Clandestine femoral neck fractures with ipsilateral diaphyseal fractures. J Orthop Trauma 1993;7:443-449.
- 01.45 Which of the following statements best describes the mechanism that occurs
- during flexion-distraction (Chance-type) injuries of the thoracolumbar spine?
- 1- The anterior column fails first because the axis of rotation is within the disk space.
- 2- The anterior column fails first because the axis of rotation is posterior to the facet
- joints.
- 3- The posterior column fails first because the axis of rotation is anterior to the vertebral
- body.
- 4- The posterior column usually remains intact and is protected by the facet capsules.
- 5- The middle column fails in compression with retropulsion of the vertebral body.
- answer
- back
- Question 01.45
- Answer = 3
- back to this question
- next question
- Reference(s)
- Beaty JH (ed): Orthopaedic Knowledge Update 6. Rosemont, IL, American Academy of Orthopaedic Surgeons, 1999, pp 653-671. Gertzbein SD, Court-Brown CM: Flexion-distraction injuries of the lumbar spine: Mechanisms of injury and classification. Clip Orthop 1988;227:52-60.
- 01.46 An 11-year-old boy injures his knee playing football. Examination reveals a 2+
- effusion and tenderness at the inferior pole of the patella. He has an extension lag
- of 20° and cannot actively extend the knee. Passive extension is full. Lachman
- and pivot-shift test results are negative. Radiographs of the knee show a
- minimally displaced small bone fragment at the inferior pole of the patella and a
- mildly high-riding patella. What is the most likely diagnosis?
- 1- Sinding-Larsen-Johansson syndrome
- 2- Bipartite patella
- 3- Patellar dislocation
- 4- Patellar sleeve fracture
- 5- Intrasubstance patellar tendon rupture
- answer
- back
- Question 01.46
- Answer = 4
- back to this question
- next question
- Reference(s)
- Houghton GR, Ackroyd CE: Sleeve fractures of the patella in children: A report of three cases. J Bone Joint Surg Br 1979;61:165-168. Wu CD, Huang SC, Liu TK: Sleeve fracture of the patella in children: A report of five cases. Am J Sports Med 1991;19:525-528.
- 01.47 A 25-year-old mechanic reports intermittent pain and tingling in the distal radial
- forearm and wrist with repetitive forceful use of the hand. Examination reveals
- mild tenderness and paresthesias over the distal quarter of the radius with
- palpation. Active ulnar deviation of the wrist, Finkelstein's test, and forceful
- pinching all increase the symptoms, as does wearing a watch or a tight shirt
- sleeve. What is the most likely diagnosis?
- 1- de Quervain's tenosynovitis
- 2- Radial tunnel syndrome
- 3- Intersection syndrome
- 4- Lateral antebrachial nerve compression
- 5- Superficial radial nerve compression
- answer
- back
- Question 01.47
- Answer = 5
- back to this question
- next question
- Reference(s)
- American Society for Surgery of the Hand: Hand Surgery Update. Rosemont, IL, American Academy of Orthopaedic Surgeons, 1996, pp 183-195. Szabo RM: Entrapment and compression neuropathies, in Green DP, Hotchkiss. RN, Pederson WC (eds): Green's Operative Hand Surgery, ed 4. New York, NY, Churchill Livingstone, 1999, pp 1404.-1447.
- 01.48 Regaining full active knee extension and functional quadriceps power is most
- difficult in the prosthetic knee patient following surgical correction of a
- 1- quadriceps tendon tear.
- 2- patellar fracture.
- 3- patellar tendon rupture.
- 4- patellar prosthetic component dislodgement.
- 5- patellar dislocation.
- answer
- back
- Question 01.48
- Answer = 3
- back to this question
- next question
- Reference(s)
- Leopold SS, Greidanus N, Paprosky WG, Berger RA, Rosenberg AG: High rate of failure of allograft reconstruction of the extensor mechanism after total knee arthroplasty. J Bone Joint Surg Am 1999;81:1574-1579. Cadambi A, Engh GA: Use of semitendinosus tendon autogenous graft for rupture of the patellar ligament after total knee arthroplasty: A report of seven cases. J Bone Joint Surg Am 1992;74:974-979.
- 01.49 The parents of a 3-year-old girl report
- that she had a transient episode of diffuse
- left knee pain and a limp 1 week ago that
- lasted about 24 hours. History reveals no
- antecedent trauma, although the parents
- describe the child as being very active.
- She is afebrile and there is no obvious
- systemic illness. Examination does not e
- elicit any clear locking or effusion.
- Radiographs of the hips are unremarkable;
- radiographs of the knee are shown in
- Figures 9a and 9b. Management should
- consist of
- 1- immobilization with the knee flexed to 45°.
- 2- immobilization with the knee in full extension.
- 3- arthroscopic microfracture technique.
- 4- fixation with absorbable pins.
- 5- observation.
- answer
- back
- A
- B
- Figures 9
- Question 01.49
- Answer = 5
- back to this question
- next question
- Reference(s)
- Caffey J, Madell SH, Royer C, Morales P: Ossification of the distal femoral epiphysis. J Bone Joint Surg Am 1958;40:647-654. Tolo VT: The lower extremity, in Morrissy RT, Weinstein SL (eds): Lovell and Winter's Pediatric Orthopaedics, ed 4. Philadelphia, PA, Lippincott-Raven, 1996, pp 1047-1075. Sales de Gauzy J, Mansat C, Darodes PH, Cahuzac JP: Natural course of osteochondritis dissecans in children. J Pediatr Orthop B 1999;8:26-28. Nawata K, Teshima R, Mono Y, Hagino H: Anomalies of ossification in the posterolateral femoral condyle: Assessment by MRI. Pediatr Radiol 1999;29:781-784. Hefti F, Beguiristain J, Moller-Madsen B, et al: Osteochondritis dissecans: A multicenter study of the European Pediatric Orthopedic Society. J Pediatr Orthop B 1999;8:231-245.
- 01.50 A 19-year-old man sustained multiple puncture wounds to his side, neck, and
- posterior and anterior chest wall following the explosion of a homemade pipe
- bomb 6 weeks ago. The patient now reports difficulty with overhead activity.
- Examination reveals loss of scapular stability with the scapula retracted and the
- inferior pole rotated medially. Winging is increased with attempts to elevate the
- arm. Electromyography confirms the diagnosis of what type of nerve palsy?
- 1- Spinal accessory
- 2- Suprascapular
- 3- Long thoracic
- 4- Axillary
- 5- Musculocutaneous
- answer
- back
- Question 01.50
- Answer = 3
- back to this question
- next question
- Reference(s)
- Connor PM, Yamaguchi K, Manifold SG, Pollock RG, Flatow EL, Bigliani LU: Split pectoralis major transfer for serratus anterior palsy. Clin Orthop 1997;341:134-142.
- 01.51 What muscle originates from Gerdy's tubercle and is innervated by the deep
- peroneal nerve?
- 1- Peroneus brevis
- 2- Peroneus tertius
- 3- Tibialis anterior
- 4- Tibialis posterior
- 5- Extensor digitorum longus
- answer
- back
- Question 01.51
- Answer = 3
- back to this question
- next question
- Reference(s)
- Anderson 3E: Grant's Atlas of Anatomy, ed 8. Baltimore, MD, William & Wilkins, 1983. Netter FH: Atlas of Human Anatomy. Summit, NJ, Ciba-Geigy, 1989, pp 457-514.
- 01.52 A 52-year-old woman has pain and limited shoulder motion after undergoing an
- open repair of a medium-sized rotator cuff tear involving the supraspinatus and
- infraspinatus tendons 1 year ago. Examination reveals mild atrophy, active and
- passive total elevation of 100°, active and passive external rotation of 0°, and
- passive internal rotation to the gluteal level. Physical therapy has failed to
- provide relief. Management should now consist of
- 1- a subacromial corticosteroid injection.
- 2- arthroscopic acromioplasty.
- 3- distal clavicle resection.
- 4- manipulation under anesthesia.
- 5- arthroscopy and soft-tissue releases.
- answer
- back
- Question 01.52
- Answer = 5
- back to this question
- next question
- Reference(s)
- Warner JJ, Allen AA, Marks PH, Wong P: Arthroscopic release of postoperative capsular contracture of the shoulder. J Bone Joint Surg Am 1997;79:1151-1158. Warner JJ, Greis PE: The treatment of stiffness of the shoulder after repair of the rotator cuff. Instr Course Lect 1998;47:67-75.
- 01.53 In muscle anatomy, what characteristic is seen in fast glycolytic (type IIb)
- fibers?
- 1- Low strength
- 2- High fatigue resistance
- 3- High aerobic capacity
- 4- High anaerobic capacity
- 5- Small motor unit
- answer
- back
- Question 01.53
- Answer = 4
- back to this question
- next question
- Reference(s)
- Buckwalter JA, Einhorn TA,, Simon SR (eds): Orthopaedic Basic Science: Biology and Biomechanics of the Musculoskeletal System, ed 2. Rosemont, IL, American Academy of Orthopaedic Surgeons, 2000, pp 684-716. Beaty JH (ed): Orthopaedic Knowledge Update 6. Rosemont, IL, American Academy of Orthopaedic Surgeons, 1999, pp 3-23. Taylor DC, Dalton 1D Jr, Seaber AV, Garrett WE 1r: Experimental muscle strain injury: Early functional and structural deficits and the increased risk for reinjury. Am J Sports Med 1993;21:190-194.
- 01.54 A 6-year-old boy sustains a both-bone forearm fracture, and management
- consists of reduction and cast immobilization. Follow-up radiographs show 25°
- of angulation of the radius on the AP view and 15° on the lateral view. What is
- the true angle of the fractured bone?
- 1- Less than 15°
- 2- 17°
- 3- 20°
- 4- 25°
- 5- Greater than 25°
- answer
- back
- Question 01.54
- Answer = 5
- back to this question
- next question
- Reference(s)
- Green SA, Gibbs P: The relationship of angulation to translation in fracture deformities. J Bone Joint Surg Am 1994;76:390-397. Noonan KJ, Price CT: Forearm and distal radius fractures in children. J Am Acad Orthop Surg 1998;6:146-156.
- 01.55 An 18-year-old woman who has had back pain for the past year now reports
- severe back pain after falling while playing soccer. She also notes a tingling
- sensation in her legs. Examination reveals hyperreflexia in the lower
- extremities. Plain radiographs shown in Figures 10a and 10b and MRI scans
- shown in Figures 10c and 10d reveal a fracture at T10. What is the most likely
- diagnosis?
- 1- Traumatic fracture
- 2- Osteomalacia
- 3- Pyogenic infection
- 4- Lymphoma
- 5- Gonococcal
- osteomyelitis
- back
- A
- B
- Figures 10
- Go to next slide
- for remaining
- figures and
- answer link
- Figures 10
- D
- C
- answer
- back to question
- Question 01.55
- Question 01.55
- Answer = 4
- back to this question
- next question
- Reference(s)
- An HS, Vaccaro AR. Dolinskas CA. Coder IM. Balderston RA, Bauerle WB: Differentiation between spinal tumors and infections with magnetic resonance imaging. Spine 1991;16:S334-5338. McLain RF, Weinstein 3N: Tumors of the spine, in Hericowitz HN, Eismont FJ, Garfin SR, Bell GR, Balderston RA, Wiesel SW (eds): Rothman-Simeone: The Spine, ed 4. Philadelphia, PA, AM Saunders, 1999, pp 1171-1206.
- 01.56 Which of the following is considered the most rigid fixation technique for the
- fracture shown in Figures 11a and 11b?
- 1- Tension band
- 2- Syndesmotic screw
- 3- Lag screw
- 4- Lateral plate
- 5- Posterior plate
- answer
- back
- A
- B
- Figures 11
- Question 01.56
- Answer = 5
- back to this question
- next question
- Reference(s)
- Kellam JF, Fischer TJ, Tornetta P III, Bosse MJ, Harris MB (eds): Orthopaedic Knowledge Update: Trauma 2. Rosemont, IL, American Academy of Orthopaedic Surgeons, 2000, pp 203-225. Schaffer JJ, Manoli A II: The antiglide plate for distal fibular fixation: A biomechanical comparison with fixation with a lateral plate. J Bone Joint Surg Am 1987;69:96-604.
- 01.57 An active 50-year-old woman has ulnar-sided wrist pain. History reveals that
- she sustained an extra-articular distal radius fracture 6 months ago and was
- treated with closed reduction and a cast. Examination reveals a prominent ulna,
- and pain is reproduced by dorsiflexing the wrist and rolling the forearm into
- pronation. Radiographs reveal a healed distal radius with 30° of dorsal tilt, 15°
- of radial inclination, and a 4-mm ulnar positive variation. Management should
- now consist of
- 1- a corrective distal radius osteotomy.
- 2- an ulnar shortening osteotomy.
- 3- Darrach resection of the ulnar head.
- 4- a Suave-Kapandji distal radioulnar joint fusion.
- 5- an arthroscopic wafer resection of the ulnar head.
- answer
- back
- Question 01.57
- Answer = 1
- back to this question
- next question
- Reference(s)
- Fernandez DL: Correction of post-traumatic wrist deformity in adults by osteotomy, bone grafting, and internal fixation. J Bone Joint Surg Am 1982;64:1164.-1178. Deitch MA, Stern PJ: Ulnocarpal abutment: Treatment options. Hand Clin 1998;14:251-263.
- 01.58 A 35-year-old man undergoes knee arthroscopy for a torn meniscus.
- Examination reveals a 1+ Lachman and a negative pivot-shift test. During the
- procedure, a partial tear involving 50% of the fibers of the anterior cruciate
- ligament (ACL) is noted. What is the most appropriate next step in
- management?
- 1- Observation
- 2- Debridement of the ACL
- 3- Reconstruction of the ACL
- 4- Radiofrequency thermal shrinkage
- 5- Semitendinosus augmentation
- answer
- back
- Question 01.58
- Answer = 1
- back to this question
- next question
- Reference(s)
- Beaty JH (ed): Orthopaedic Knowledge Update 6. Rosemont, IL. American Academy of Orthopaedic Surgeons, 1999, pp 533-557. Noyes FR, Mooar LA, Moormann CT III, McGinniss GH: Partial tears of the anterior cruciate ligament: Progression to complete ligament deficiency. J Bone Joint Surg Br 1989;71:825-833.
- 01.59 A 70-year-old woman has a flexible planovalgus deformity as the result of
- posterior tibial tendon deficiency. Her symptoms improved following cast
- immobilization but returned when the cast was removed. She does not wish to
- have surgery. Management should now consist of
- 1- continued serial casting until the symptoms resolve.
- 2- a hinged ankle-foot orthosis.
- 3- custom-molded orthoses.
- 4- a high-top shoe with a SACH heel.
- 5- a medial heel wedge.
- answer
- back
- Question 01.59
- Answer = 3
- back to this question
- next question
- Reference(s)
- Mizel MS, Miller RA, ScioIi MW (eds): Orthopaedic Knowledge Update: Foot and Ankle 2. Rosemont, IL, American Academy of Orthopaedic Surgeons, 1998, pp 55-64. Chap W, Wapner KL, Lee TH, Adams J, Hecht PJ: Nonoperative management of posterior tibial tendon dysfunction. Foot Ankle Int 1996;17:736-741.
- 01.60 During the repair process of an unstable fracture, the expression of major
- collagen types during fracture callus formation can best be characterized by
- 1- type I collagen only.
- 2- type II collagen only.
- 3- type I collagen early, followed by type II collagen.
- 4- type II collagen early, followed by type I collagen.
- 5- type I and type II collagen equally throughout.
- answer
- back
- Question 01.60
- Answer = 4
- back to this question
- next question
- Reference(s)
- Pau WT, Einhorn TA: The biochemistry of fracture healing. Curr Orthop. 1996;6:207-213. Buckwalter JA, Einhorn TA, Simon SR (eds): Orthopaedic Basic Science: Biology and Biomechanics of the Musculoskeletal System, ed 2. Rosemont, IL, American Academy of Orthopaedic Surgeons, 2000, pp 372-399.
- 01.61 A 50-year-old patient with rheumatoid arthritis has protrusio acetabuli. The best
- method of surgically reconstructing the acetabulum during hip arthroplasty
- involves
- 1- using cement with a metal-backed acetabular component.
- 2- using the femoral head as a structural graft with cementless component.
- 3- using cancellous autograft with cementless acetabular component.
- 4- performing concentric reaming with a bipolar prosthesis.
- 5- placing the acetabular component medially on host bone.
- answer
- back
- Question 01.61
- Answer = 3
- back to this question
- next question
- Reference(s)
- Callaghan 1J, Dennis DA, Paprosky WG, Rosenberg AG (eds): Orthopaedic Knowledge Update: Hip and Knee Reconstruction. Rosemont, IL, American Academy of Orthopaedic Surgeons, 1995, pp 215-225.
- 01.62 A 32-year-old man has left shoulder pain. History reveals that he underwent an
- open Bankart repair for recurrent shoulder dislocations 3 years ago. Four months
- after the surgery he fell and sustained a hyperabduction injury of the shoulder.
- Examination reveals slightly limited shoulder motion, pain with passive external
- rotation, and he cannot maintain a belly press maneuver against resistance
- (Napolean's sign). An MR-arthrogram is shown in Figure 12. Because physical
- therapy has failed to provide relief, treatment should now consist of
- 1- arthroscopic Bankart repair.
- 2- an open capsular shift.
- 3- a subscapularis transfer.
- 4- a pectoralis major transfer.
- 5- a pectoralis minor transfer.
- answer
- back
- Figure 12
- Question 01.62
- Answer = 4
- back to this question
- next question
- Reference(s)
- Gerber C, Hersche O, Farron A: Isolated rupture of the subscapularis tendon. J Bone Joint Surg Am 1996;78:1015-1023. Wirth MA, Rockwood CA: Operative treatment of irreparable rupture of the subscapularis. J Bone Joint Surg Am 1997;79:722-731.
- 01.63 Which of the following terms best describes a muscle contraction that occurs as
- the muscle lengthens?
- 1- Concentric
- 2- Eccentric
- 3- Isotonic
- 4- Isometric
- 5- Isokinetic
- answer
- back
- Question 01.63
- Answer = 2
- back to this question
- next question
- Reference(s)
- Skeletal Muscle: Form & Function, in Human Kinetics. Champaign, II„ Alan J McComas, 1996, p 177. Kreighbaum E, Barthels K: Biomechanics: A Qualitative Approach for Studying Human Movement, ed 2. Minneapolis, MN, Burgess Publishing, 1985, p 317.
- 01.64 What nerve is most commonly injured during an ilioinguinal approach to the
- acetabulum?
- 1- Femoral
- 2- Ilioinguinal
- 3- Obturator
- 4- L5 nerve root
- 5- Lateral femoral cutaneous
- answer
- back
- Question 01.64
- Answer = 5
- back to this question
- next question
- Reference(s)
- Hoppenfeld S, deBoer P (eds): Surgical Exposures in Orthopaedics: The Anatomic Approach, ed 2. Philadelphia, PA, JB Lippincott, 1994, pp 323-401.
- 01.65 A patient has a gunshot wound to the cervical spine. Which of the following
- factors is considered the best indication for spinal surgery?
- 1- Bullet in the spinal canal
- 2- Bullet in the disk space
- 3- Cerebrospinal fluid leak
- 4- Osteoligamentous instability
- 5- Bone fragments in the spinal canal
- answer
- back
- Question 01.65
- Answer = 4
- back to this question
- next question
- Reference(s)
- Levine AM: Orthopaedic Knowledge Update: Trauma. Rosemont, IL, American Academy of Orthopaedic Surgeons, 1996, pp 329-334.
- 01.66 Figures 13a and 13b show the radiographs of an 8-year-old boy who injured his
- left knee in a fall from his bicycle. A bloody effusion is aspirated, and 10 mL of
- 1 % lidocaine is injected into the joint, but on examination the knee will not
- extend fully. The next most appropriate step in management should consist of
- 1- an early referral to physical therapy to restore range of motion.
- 2- arthroscopy (and possible arthrotomy) for reduction and fixation.
- 3- arthroscopy (and possible
- arthrotomy) for removal
- of the loose fragment.
- 4- MRI of the knee.
- 5- a hinged knee orthosis
- and early range-of-motion
- exercises.
- answer
- back
- A
- B
- Figures 13
- Reference(s)
- Sponseller PD, Beaty JH: Fractures and dislocations about the knee, in Rockwood CA, Wilkins KE, Beaty JH (eds): Fractures in Children, ed 4. Philadelphia, PA, Lippincott-Raven, 1996, pp 1231-1329. Tolo V: Fractures and dislocations about the knee, in Green NE, Swiornkowski MF (eds): Skeletal Trauma in Children, ed 2. Philadelphia, PA. WB Saunders, 1998, pp 431-458. Panni AS, Milano G, Tartarone M, Fabbriciani C: Arthroscopic treatment of malunited and nonunited avulsion fractures of the anterior tibial spine. Arthroscopy 1998;14:233-240. Osti L, Merlo F, Liu SH, Bocci L: A simple modified arthroscopic procedure for fixation of displaced tibias eminence fractures. Arthroscopy 2000;16:379-382.
- Question 01.66
- Answer = 2
- back to this question
- next question
- 01.67 A clinical photograph of a 2-year-old child's hand is shown in Figure 14. The
- image reveals no clear evidence of carpometacarpal joint development.
- Management should consist of
- 1- a splinting program until the child reaches skeletal maturity.
- 2- thumb amputation and index pollicization.
- 3- Ilizarov bone lengthening followed by web space Z-plasty and soft-tissue
- reconstruction.
- 4- web space Z-plasty and tendon transfers to restore opposition.
- 5- toe-to-thumb microvascular reconstruction.
- answer
- back
- Figure 14
- Question 01.67
- Answer = 2
- back to this question
- next question
- Reference(s)
- Manske PR, McCarroll HR Jr. Reconstruction of the congenitally deficient thumb. Hand Clin 1992;8:177-196. Buck-Gramcko D: Pollicization of the index finger-. Method and results in aplasia and hypoplasia of the thumb. J Bone Joint Surg Am 1971;53:1605-1617.
- 01.68 Which of the following laboratory studies would most likely yield the correct
- diagnosis for the patient shown in Figure 15?
- 1- Serum protein electrophoresis
- 2- Calcium and phosphorus levels
- 3- Lactate dehydrogenase level
- 4- Thyroid panel
- 5- Alkaline phosphatase and urine hydroxyproline levels
- answer
- back
- Figure 15
- Question 01.68
- Answer = 1
- back to this question
- next question
- Reference(s)
- Lewis MM (ed): Musculoskeletal Oncology: A Multidisciplinary Approach. Philadelphia, PA, WB Saunders, 1992, pp 57-61. Bergsagel DE: Plasma cell myeloma: An interpretive view. Cancer 1972;30:1588-1594.
- 01.69 Item deleted after statistical review
- (and no answer or references cited)
- back
- next question
- 01.70 The posterior band of the inferior glenohumeral ligament complex functions as a
- posterior stabilizer when the shoulder is in which of the following positions?
- 1- 0° of abduction and internal rotation
- 2- 90° of abduction and external rotation
- 3- Extension and external rotation
- 4- Flexion and internal rotation
- 5- Flexion and external rotation
- answer
- back
- Question 01.70
- Answer = 4
- back to this question
- next question
- Reference(s)
- Blaster RB, Guldberg RE, Rothman CT: Anterior shoulder stability: Contribution of rotator cuff forces and the capsular ligament in a cadaver model. J Shoulder Elbow Surg, 1992;1:140-150. Naggen L, Jenp YN, Malanga G: Major capsuloligamentous restraints to posterior instability of the shoulder. Orthop Traps 1995;19:325.
- 01.71 The parents of a 3-year-old child are concerned about the posture of the child's
- toes. They report frequent erythema and tenderness at the dorsum of the third
- toe and they have difficulty with care of the nail of the fourth toe. Clinical
- photographs are shown in Figures 16a and l6b. Treatment should consist of
- 1- ablation of the nail of the fourth toe.
- 2- proximal interphalangeal joint arthrodesis of the fourth toe.
- 3- distal interphalangeal joint arthrodesis of the third toe.
- 4- release of the extensors of the third toe.
- 5- release of the flexors of the fourth toe
- answer
- back
- A
- B
- Figures 16
- Question 01.71
- Answer = 5
- back to this question
- next question
- Reference(s)
- Hamer AJ, Stanley D, Smith TW: Surgery for curly toe deformity: A double-blind, randomized, prospective trial. J Bone Joint Sung Br 1993;75:662-663. Sullivan JA: The child's foot, in Morrissy RT, Weinstein SL (eds): Lovell and Winter's Pediatric Orthopaedics, ed 4. Philadelphia, PA, Lippincott-Raven, 1996, pp 1077-1135.
- 01.72 A woman who claims to have slipped on a wet floor at home is brought to the
- emergency department by her husband, who appears to be inebriated.
- Examination of the patient reveals multiple contusions to the head, upper
- extremities, and torso, as well as a midshaft transverse fracture of the right ulna.
- In addition to fracture and soft-tissue treatment, management should include
- 1- placing the husband under arrest for driving while intoxicated.
- 2- placing the husband under house arrest for reckless endangerment.
- 3- placing the patient in protective custody.
- 4- consulting social services for investigation of possible spousal abuse.
- 5- consulting social services regarding an alcohol detoxification program.
- answer
- back
- Question 01.72
- Answer = 4
- back to this question
- next question
- Reference(s)
- ZilImer DA: Domestic violence: The role of the orthopaedic surgeon in identification and treatment. J Am Acad Orthop Surg 2000;8:91-96.
- 01.73 A 73-year-old woman has had a painless,
- enlarging popliteal soft-tissue mass for the past
- 4 months. Plain radiographs show a
- nonmineralized soft-tissue shadow. T1 and T2
- weighted MRI scans are shown in Figures 17a
- and l7b. A biopsy specimen is shown in Figure
- 17c. Immunohistochemistry is positive for
- vimentin, smooth muscle actin, and desmin, and
- negative for cytokeratin and S-100. What is the
- most likely diagnosis?
- 1- Malignant fibrous histiocytoma
- 2- Liposarcoma
- 3- Synovial sarcoma
- 4- Fibrosarcoma
- 5- Leiomyosarcoma
- answer
- back
- A
- B
- Figures 17
- C
- Question 01.73
- Answer = 5
- back to this question
- next question
- Reference(s)
- Choong PFM, Pritchard DJ: Common malignant soft-tissue tumors, in Simon MA, Springfield D (eds): Surgery for Bone and Soft-Tissue Tumors. Philadelphia, PA, Lippincott-Raven, 1998, pp 541-553. Enzinger FM, Weiss SW: Soft Tissue Tumors, ed 3. St Louis, MO, CV Mosby, 1995, pp 491-510.
- 01.74 Which of the following is considered a contraindication to the treatment of a
- humeral shaft fracture with functional bracing?
- 1- Radial nerve palsy
- 2- Transverse fracture
- 3- Distal third fracture
- 4- Low-velocity gunshot fracture
- 5- Ipsilateral brachial plexus palsy
- answer
- back
- Question 01.74
- Answer = 5
- back to this question
- next question
- Reference(s)
- Brien WW, Gellman H, Becker V, Garland DE, Waters RI, Wiss DA: Management of fractures of the humerus in patients who have an injury of the ipsilateral brachial plexus. J Bone Joint Sung Am 1990;72:1208-1210. Sarmiento A, Zagorski JA, Zych GA. Latter LL, Capes CA: Functional bracing for the treatment of fractures of the humeral diaphysis. J Bone Joint Surg Am 2000;82:478-486.
- 01.75 In acetabular exposure during a total hip replacement, the surgeon places a
- retractor behind the transverse acetabular ligament. What nerve can be
- damaged?
- 1- Superior gluteal
- 2- Inferior gluteal
- 3- Femoral
- 4- Obturator
- 5- Ilioinguinal
- answer
- back
- Question 01.75
- Answer = 4
- back to this question
- next question
- Reference(s)
- Jerkins DB: Hollinsheads Functional Anatomy of the Limbs and Back, ed 6. Philadelphia, PA, WB Saunders, 1991, p 221.
- 01.76 A 22-year-old man is seen in the emergency department after sustaining injuries in a fight
- 12 hours ago. Examination reveals swelling and local cellulitis over the dorsum of his
- right hand and a small, draining puncture wound over the metacarpophalangeal joint. He
- is afebrile. A clinical photograph is shown in Figure 18. Radiographs show no fractures.
- Management should consist of
- 1- discharge home with sterile dressing changes, antibiotics for 10 days, and a follow-up
- examination in 48 hours.
- 2- povidone-iodine soaks and IV administration of 2 g of cefazolin in the emergency department,
- discharge home with daily povidone-iodine soaks, and antibiotics for 10 days.
- 3- hospital admission for elevation and IV antibiotics.
- 4- hospital admission, IV antibiotics, and hyperbaric
- oxygen treatment two times a day for 4 to 6 days.
- 5- hospital admission, exploration, irrigation and
- debridement, and IV antibiotics.
- answer
- back
- Figure 18
- Question 01.76
- Answer = 5
- back to this question
- next question
- Reference(s)
- Chuinard RG, D'Ambrosia RD: Human bite infections of the hand. J Bone Joint Surg Am 1977;59:416-418. Dreyfuss UY, Singer M: Human bites of the hand: A study of one hundred six patients. J Hand Surg Am 1985;10:884-889.
- 01.77 Item deleted after statistical review
- (and no answer or references cited)
- back
- next question
- 01.78 Figure 20 shows the CT scan of a 30-year-old racquetball player who sustained
- a direct blow to the shoulder 8 weeks ago when he ran into a wall. Management
- should consist of
- 1- observation.
- 2- closed reduction and a spica cast.
- 3- open reduction and a lesser tuberosity transfer.
- 4- hemiarthroplasty.
- 5- arthroscopic stabilization.
- answer
- back
- Figure 20
- Question 01.78
- Answer = 3
- back to this question
- next question
- Reference(s)
- Roberts CS: Optimizing performance of the McLaughlin procedure for posterior dislocations of the shoulder. Tech in Orthop 1998;13:113-116. Zuckerman JD: McLaughlin procedure for acute and chronic posterior dislocations, in Craig EV (ed): The Shoulder. New York, NY, Raven Press, 1995, pp 165-180.
- 01.79 A 45-year-old man underwent a zone II flexor tendon repair. What type of splint
- will allow for the greatest safe excursion of the repaired tendons in the first 6
- weeks?
- 1- Spint with continuous passive motion device
- 2- Static progressive splint
- 3- Dynamic splint
- 4- Dynamic splint with a palmar pulley
- 5- Dynamic splint with a hinge allowing for motion at the wrist
- answer
- back
- Question 01.79
- Answer = 5
- back to this question
- next question
- Reference(s)
- Horn E, Lin GT, Cooney WP, Linscheid RL, An KN: Comparative flexor tendon excursions after passive mobilization: An in vitro study. J Hand Surg Am 1992;17:559-566. Cooney WP, Lin GT, An KN: Improved tendon excursion following flexor tendon repair. J Hand Ther 1989:102-106.
- 01.80 Item deleted after statistical review
- (and no answer or references cited)
- back
- next question
- 01.81 Translation of the atlantoaxial complex, as indicated by the atlanto-dens interval
- (ADI) on lateral flexion and extension radiographs, is considered abnormal in an
- adult when it is more than how many millimeters?
- 1- 1
- 2- 3
- 3- 5
- 4- 7
- 5- 9
- answer
- back
- Question 01.81
- Answer = 2
- back to this question
- next question
- Reference(s)
- Boden SD, Dodge LD, Bohlman HH, Rechtine GR: Rheumatoid arthritis of the cervical spine: A long-term analysis with predictors of paralysis and recovery. J Bone Joint Surg Am 1993;75:1282-1297. Weissman BN, Aliabadi P, Weinfeld MS, Thomas WK Sosman JL: Prognostic features of atlantoaxial subluxation in rheumatoid arthritis patients. Radiology 1982;144:745-751. White AA BI, Johnson RM, Punjabi MM, Southwick WO: Biomechanical analysis of clinical stability in the cervical spine. Clip Orthop 1975;109:85-96.
- 01.82 Which of the following factors will have the greatest affect on the stability of
- the hip after closed reduction of a posterior wall fracture-dislocation?
- 1- Magnitude of the posterior capsular disruption
- 2- Length of time until reduction occurs
- 3- Size of the wall fragment
- 4- Presence of femoral head depression
- 5- Presence of loose bodies in the fovea of the hip joint
- answer
- back
- Question 01.82
- Answer = 3
- back to this question
- next question
- Reference(s)
- Beaty 1H (ed): Orthopaedic Knowledge Update 6. Rosemont, IL, American Academy of Orthopaedic Surgeons, 1999, pp 44153. Malta JM: Fractures of the acetabulum: Accuracy of reduction and clinical results in patients managed operatively within three weeks after the injury. J Bone Joint Surg Am 1996;78:1632-1645.
- 01.83 A 16-year-old girl who is an elite runner has not had a menstrual cycle for the
- past 6 months. History reveals menarche at age 13 years with menstrual cycles
- of 36 days. What is the most likely diagnosis?
- 1- Primary amenorrhea
- 2- Secondary amenorrhea
- 3- Oligomenorrhea
- 4- Thyroid disease
- 5- Klinefelter's syndrome
- answer
- back
- Question 01.83
- Answer = 2
- back to this question
- next question
- Reference(s)
- Teitz CC: The female athlete triad, in Teitz CC (ed): The Female Athlete. Rosemont, IL, American Academy of Orthopaedic Surgeons, 1997, pp 75-80. Griffin LY: The young female athlete, in Stanitski C, DeLee JC, Drez D (eds): Pediatric and Adolescent Sports Medicine. Philadelphia, PA, WB Saunders, 1994, pp 16-23.
- 01.84 The maximal rate of curve progression in idiopathic scoliosis is best correlated
- with which of the following parameters in girls?
- 1- Risser stage 1
- 2- Peak velocity of growth
- 3- Chronologic age of 12 years
- 4- Chronologic age of 13 years
- 5- Menarche
- answer
- back
- Question 01.84
- Answer = 2
- back to this question
- next question
- Reference(s)
- Little DG, Song KM, Katz D, Herring JA: Relationship of peak height velocity to other maturity indicators in idiopathic scoliosis in girls. J Bone Joint Surg Am 2000;82:685-693.
- 01.85 An 11-year-old girl sustained an inversion sprain to the left ankle 12 weeks ago
- and is unable to bear weight on it. Examination reveals diffuse hypersensitivity.
- The foot has a purple hue and is diffusely swollen. A bone scan shows diffuse
- uptake around the hindfoot. Management should consist of
- 1- triple arthrodesis.
- 2- a below-knee walking cast.
- 3- crutches and no weight bearing.
- 4- sympathetic nerve blockade and physical therapy.
- 5- narcotic analgesics.
- answer
- back
- Question 01.85
- Answer = 4
- back to this question
- next question
- Reference(s)
- Mizel MS, Miller R.4, Scioli MW (eds): Orthopaedic Knowledge Update: Foot and Ankle 2. Rosemont, 1L, American Academy of Orthopaedic Surgeons, 1998; pp 101-111. Pedowitz WJ: Soft tissue disorders of the foot, in Coughlin MJ, Mann RA (eds): Surgery of the Foot and Ankle, ed 7. St Louis, MO, Harcourt Health Science, 1999, pp 1373-1397.
- 01.86 A 24-year-old man has had ankle pain for 1 year. Figures 21a through 21d show
- AP and lateral radiographs, an axial T1-weighted MRI scan, and a biopsy
- specimen. What is the most likely diagnosis?
- 1- Nonossifying fibroma
- 2- Chondroblastoma
- 3- Giant cell tumor
- 4- Aneurysmal bone
- cyst
- 5- Telangiectactic
- osteosarcoma
- back
- A
- B
- Figures 21
- Go to next slide for remaining figures and answer link
- Figures 21
- D
- C
- answer
- back to question
- Question 01.86
- Question 01.86
- Answer = 3
- back to this question
- next question
- Reference(s)
- Gitelis S, McDonald DJ: Common benign bone tumors and usual treatment, in Simon MA, Springfield D (eds): Surgery for Bone and Soft-Tissue Tumors. Philadelphia, PA, Lippincott-Raven, 1998, pp 275-286. Primary bone tumors, in McCarthy EF, Frassica FJ (eds): Pathology of Bone and Joint Disorders with Clinical and Radiographic Correlation. Philadelphia, PA, WB Saunders, 1998, pp 195-276.
- 01.87 Flexion-extension radiographs of an adult patient with long-standing rheumatoid
- arthritis have shown Cl-2 instability of 6 mm for the past several years. Current
- flexion-extension radiographs now show the C1-2 atlanto-dens interval to be
- fixed at 5 mm (there is no motion on flexion-extension). What is the most likely
- explanation for this finding?
- 1- The arthritis has stabilized.
- 2- The C1-2 joint has fused.
- 3- The patient has been wearing a cervical collar.
- 4- Medical advancements have been made in the management of arthritis.
- 5- Basilar impression (atlantoaxial impaction) has developed.
- answer
- back
- Question 01.87
- Answer = 5
- back to this question
- next question
- Reference(s)
- Oda T, Fujiwara K, Yonenobu K, Azuma B, Ochi T: Natural course of cervical spine lesions in rheumatoid arthritis. Spine 1995;20:1128-1135.
- 01.88 A woman with degenerative arthritis and a fixed genu valgum deformity of 17°
- undergoes primary total knee arthroplasty under general anesthesia. In the
- recovery room, she is unable to dorsiflex her foot. Immediate management
- should include
- 1- fasciotomies.
- 2- surgical nerve decompression.
- 3- flexion of the knee.
- 4- continuous passive motion.
- 5- electromyography.
- answer
- back
- Question 01.88
- Answer = 3
- back to this question
- next question
- Reference(s)
- Mont MA, Dellon AL, Chen F, Hungerford MW, Krackow KA, Hungerford DS: The operative treatment of peroneal nerve palsy. J Bone Joint Surg Am 1996;78:863-869. Beaty JH (ed): Orthopaedic Knowledge Update 6. Rosemont, IL, American Academy of Orthopaedic Surgeons, 1999, pp 559-582.
- 01.89 A 30-year-old professional soccer player sustains a midshaft fracture of the tibia
- and fibula. History reveals that the patient underwent a successful anterior
- cruciate ligament reconstruction with central one third bone-patellar tendon-bone
- autograft and graft fixation with interference screws 2 years ago. Figures 22a and
- 22b show the pre-and postoperative radiograph. What is the most likely
- postoperative complication in this patient?
- 1- Loss of graft fixation
- 2- Fracture through the tibial
- tunnel
- 3- Galvanic corrosion
- (battery effect)
- 4- Anterior knee pain
- 5- Saphenous neuralgia
- answer
- back
- A
- B
- Figures 22
- Question 01.89
- Answer = 4
- back to this question
- next question
- Reference(s)
- Keating JF, Orfaly R, O'Brien PJ: Knee pain after tibial nailing. J Orthop Trauma 1997;11:10-13. Roberts C, John C, Seligson D: Prior anterior cruciate ligament reconstruction complicating intramedullary nailing of a tibia fracture. Arthroscopy 1998;14:779-783.
- 01.90 Which of the following muscles protracts the shoulder?
- 1- Serratus anterior
- 2- Rhomboid major
- 3- Trapezius
- 4- Latissimus dorsi
- 5- Subscapularis
- answer
- back
- Question 01.90
- Answer = 1
- back to this question
- next question
- Reference(s)
- Kahn JF, Plancher KD, Hawkins RJ: Scapular winging. J Am Acad Orthop Surg 1995;3:319-325.
- 01.91 Which of the following pharmacologic agents may antagonize the
- anticoagulation effect of warfarin?
- 1- Cefamandole
- 2- Cimetidine
- 3- Phenytoin
- 4- Trimethoprim
- 5- Phenobarbital
- answer
- back
- Question 01.91
- Answer = 5
- back to this question
- next question
- Reference(s)
- Simon SR (ed): Orthopaedic Basic Science. Rosemont, IL, American Academy of Orthopaedic Surgeons, 1994, pp 487-517.
- 01.92 Figures 23a and 23b show an AP open mouth view and a CT scan at C1-2.
- Because this is an acute injury with no neurologic deficit, the best course of
- action should be
- 1- acute occipital-cervical fusion with rigid instrumentation.
- 2- use of a soft collar.
- 3- use of a halo vest for 3 months, followed by assessment for C1-2 instability.
- 4- a gradual return to activity with no restrictions.
- 5- flexion-extension radiographs to help determine the need for surgery.
- answer
- back
- A
- B
- Figures 23
- Question 01.92
- Answer = 3
- back to this question
- next question
- Reference(s)
- Levine AM: Orthopaedic Knowledge Update: Trauma. Rosemont. IL. American Academy of Orthopaedic Surgeons, 1996, pp 317-322.
- 01.93 Item deleted after statistical review
- (and no answer or references cited)
- back
- next question
- 01.94 Item deleted after statistical review
- (and no answer or references cited)
- back
- next question
- 01.95 Intradiscal pressure in the third lumbar disk is least in which of the following
- positions?
- 1- Sitting, flexed forward slightly
- 2- Sitting, straight
- 3- Laying, supine
- 4- Standing, straight
- 5- Standing, slightly extended
- answer
- back
- Question 01.95
- Answer = 3
- back to this question
- next question
- Reference(s)
- Nachemson A: The lumbar spine: An orthopaedic challenge. Spine 1976;1:59-71. White AA II, Panjabi MM (eds): Clinical Biomechanics of the Spine, ed 2. Philadelphia, PA, JB Lippincott, 1990, pp 454-461.
- 01.96 A 35-year-old man who sustained a grade II open fracture of the right tibia 9
- months ago underwent reamed intramedullary nailing. The patient continues to
- have pain with weight bearing. Laboratory studies show a normal WBC and
- erythrocyte sedimentation rate, and there has been no change in the radiographs
- in the past 3 months. Current radiographs are shown in Figures 24a and 24b.
- What is the most likely diagnosis?
- 1- Delayed union
- 2- Aseptic nonunion
- 3- Infected nonunion
- 4- Synovial pseudarthrosis
- 5- Failure of the internal fixation device
- answer
- back
- A
- B
- Figures 24
- Question 01.96
- Answer = 2
- back to this question
- next question
- Reference(s)
- Browner BD, Jupiter JB, Levine AM, Trafton PB (eds): Skeletal Trauma, ed 2. Philadelphia, PA, WB Saunders, 1998, pp 68-72. Beaty JH (ed): Orthopaedic Knowledge Update 6. Rosemont, IL, American Academy of Orthopaedic Surgeons, 1999, pp 25-35.
- 01.97 Figure 25 shows the AP radiograph of a 20-year-old man who sustained a
- twisting injury to his foot. An attempt at closed reduction is unsuccessful, most
- likely because of
- 1- buttonholing of the talar head through the extensor retinaculum.
- 2- osteochondral fracture of the posterior facet of the talus.
- 3- interpositioning of the peroneal tendons.
- 4- interpositioning of the posterior tibial tendon.
- 5- fracture of the talar neck.
- answer
- back
- Figure 25
- Question 01.97
- Answer = 4
- back to this question
- next question
- Reference(s)
- Bellabarba C, Sanders R: Dislocation of the foot, in Coughlin MJ, Mann RA (eds): Surgery of the Foot and Ankle, ed 7. St Louis, MO, Harcourt Health Science, 1999, pp 1519-1573. Hansen ST 1r: Foot injuries, in Browner BD, Jupiter JB, Levine AM, Trafton PB (eds): Skeletal Trauma, ed 2. Philadelphia, PA, WB Saunders, 1998, pp 2405-2438. DeLee JC, Curtis R: Subtalar dislocation of the foot. J Bone Joint Surg Am 1982;64:433-437.
- 01. 98 A 65-year-old man has had progressively worsening pain and limited motion in
- the left shoulder for the past year. History reveals that he sustained a
- nondisplaced fracture of the surgical neck of the humerus 10 years ago. Plain
- radiographs are shown in Figures 26a and 26b. What is the most likely
- diagnosis?
- 1- Osteonecrosis
- 2- Posttraumatic osteoarthritis
- 3- Rheumatoid arthritis
- 4- Gouty arthritis
- 5- Chondrocalcinosis
- answer
- back
- A
- B
- Figures 26
- Question 01.98
- Answer = 2
- back to this question
- next question
- Reference(s)
- Matsen FA III, Rock-wood CA Jr, Wirth MA, Lippitt SB: Glenohumeral arthritis and its management, in Rockwood CA Jr, Matsen FA III, Wirth MA, Harryman DT II (eds): The Shoulder. Philadelphia, PA, WB Saunders, 1998, pp 840-964.
- 01.99 A 14-year-old basketball player has had activity-related low back pain for the
- past 6 weeks. Plain radiographs are unremarkable. Selected coronal single-
- photon emission computed tomography scans are shown in Figure 27. The best
- course of action should be
- 1- bilateral facet blocks.
- 2- brace immobilization for 3 to 6 months.
- 3- instrumented posterior spinal fusion.
- 4- a CT-guided needle biopsy.
- 5- an open incisional biopsy.
- answer
- back
- Figure 27
- Question 01.99
- Answer = 2
- back to this question
- next question
- Reference(s)
- Loder RT, Hensinger RN: Fractures of the thoracic and lumbar spine, in Rockwood CA, Wilkins KE, Beaty 1H (eds): Fractures in Children, ed 4. Philadelphia, PA, Lippincott-Raven, 1996, pp 1062-1096. Lonstein JE: Spondylolysis and spondylolisthesis, in Morrissy RT, Weinstein SL (eds): Lovell and Winter's Pediatric Orthopaedics, ed 4. Philadelphia, PA, Lippincott-Raven, 1996, pp 717-737. Congeni J, McCulloch J, Swanson K: Lumbar spondylolysis: A study of natural progression in athletes. Am J Sports Med 1997;25:248-253. Lonstein JE: Spondylolisthesis in children: Cause. natural history, and management. Spine 19994:2640-2648. Anderson K, Sarwark JF, Conway JJ, Logue ES, Schafer WI: Quantitative assessment with SPELT imaging of stress injuries of the pats interarticularis and response to bracing. J Pediatr Orthop 2000;20:28-33.
- 01.100 Which of the following is considered an advantage of the inside-out technique
- for meniscal repairs compared with the outside-in technique?
- 1- Improved ultimate knee motion
- 2- Increased rate of healing
- 3- Diminished infection rates
- 4- The ability to achieve proper suture orientation in posterior horn tears
- 5- Decreased risk of neurologic injury
- answer
- back
- Question 01.100
- Answer = 4
- back to this question
- next question
- Reference(s)
- Rodeo SA: Arthroscopic meniscal repair with use of the outside-in technique. Instr Course Lect 2000;49:195-206. Post WR, Akers SR, Kish V: Load to failure of common meniscal repair techniques: Effects of suture technique and suture material. Arthroscopy 1997;13:731-736.
- 01.101 An anterior (Smith-Peterson) approach to the hip joint uses what internervous
- plane?
- 1- Superior gluteal and femoral
- 2- Superior gluteal and inferior gluteal
- 3- Femoral and obturator
- 4- Sciatic and superior gluteal
- 5- Sciatic and femoral
- answer
- back
- Question 01.101
- Answer = 1
- back to this question
- next question
- Reference(s)
- Hoppenfeld S, deBoer P (eds): Surgical Exposures in Orthopaedics: The Anatomic Approach. Philadelphia, PA, Lippincott Williams & Wilkins, 1984, pp 303-32I.
- 01.102 A 21-year-old man sustains the acetabular fracture shown in Figures 28a
- through 28c. The best outcomes following surgical treatment of this fracture
- have been reported with which
- of the following surgical
- approaches?
- 1- Posterior (Kocher-Langenbeck)
- 2- Triradiate
- 3- Extended iliofemoral
- 4- Modified extensile lateral
- 5- Ilioinguinal
- answer
- back
- A
- B
- Figures 28
- C
- Question 01.102
- Answer = 5
- back to this question
- next question
- Reference(s)
- Letournel E: The treatment of acetabular fractures through the ilioinguinal approach. Clip Orthop 1993;292:62-76.
- 01.103 What type of injury is shown in Figure 29?
- 1- Compression fracture
- 2- Burst fracture
- 3- Flexion-distraction
- 4- Extension injury
- 5- Fracture-dislocation
- answer
- back
- Figure 29 (all three)
- Question 01.103
- Answer = 2
- back to this question
- next question
- Reference(s)
- Dents F: The three column spine and its significance in the classification of acute thoracolumbar spinal injuries. Spine 1983;8:817-831.
- 01.104 A 55-year-old patient underwent a total hip arthroplasty with a diaphyseal -
- locking, fully porous-coated cementless femoral prosthesis 8 years ago.
- Radiographs reveal a distally fixed stem with distal spot welds and proximal
- bone loss. The femoral head is located concentrically within the acetabulum.
- What is the most likely etiology of the bone loss?
- 1- Bone hypertrophy
- 2- Stress shielding
- 3- Osteoporosis
- 4- Osteolysis
- 5- Osteonecrosis
- answer
- back
- Question 01.104
- Answer = 2
- back to this question
- next question
- Reference(s)
- Callaghan JJ, Dennis DA, Paprosky WG, Rosenberg AG (eds): Orthopaedic Knowledge Update: Hip and Knee Reconstruction. Rosemont, IL, American
- Academy of Orthopaedic Surgeons, 1995, pp 127-138.
- 01.105 Endurance strength training has been shown to have which of the following
- demonstrated physiologic effects?
- 1- Decreases aerobic capacity
- 2- Decreases bone mineral density with weight-bearing activities
- 3- Decreases type II muscle fiber cross-sectional area
- 4- Increases heart rate at rest
- 5- Improves blood lipid profiles
- answer
- back
- Question 01.105
- Answer = 5
- back to this question
- next question
- Reference(s)
- Miszko T, Cress M: A lifetime of fitness, in Clinics in Sports Medicine. Philadelphia, PA, WB Saunders, 2000, vol 19, pp 215-232. Blumenthal JA, Emery CF, Madden DJ, et al: Cardiovascular and behavioral effects of aerobic exercise training in healthy older men and woman. J Gerontol
- 1989;44:147-157.
- 01.106 What organ secretes calcitonin?
- 1- Parathyroid
- 2- Thyroid
- 3- Kidney
- 4- Bone
- 5- Skin
- answer
- back
- Question 01.106
- Answer = 2
- back to this question
- next question
- Reference(s)
- Beaty JH (ed): Orthopaedic Knowledge Update 6. Rosemont, IL, American Academy of Orthopaedic Surgeons, 1999, pp 149-165. Silver JJ, Majeska RJ, Einhorn TA: An update on bone cell biology. Curr Opin Orthop 1994;5:50-59.
- 01.107 The volar radioscapholunate ligament (ligament of Testut) functions primarily
- as a
- 1- neurovascular conduit.
- 2- stabilizing ligament of the scapholunate interval.
- 3- stabilizing ligament of the radiocarpal joint.
- 4- stabilizing ligament of the midcarpal joint.
- 5- septal ligament contiguous with the interfossal ridge.
- answer
- back
- Question 01.107
- Answer = 1
- back to this question
- next question
- Reference(s)
- Berger RA, Landsmeer JM: The palmar radiocarpal ligaments: A study of adult and fetal human wrist joints. J Hand Surg Am 1990;15:847-854. Hixson ML, Stewart C: Microvascular anatomy of the radioscapholunate ligament of the wrist. J Hand Surg Am 1990;15:279-282.
- 01.108 A 19-year-old man reports groin pain after undergoing antegrade nailing of a
- femoral shaft fracture 3 weeks ago. Figures 30a and 30b show the current
- radiograph and tomogram. The next most appropriate step in management
- should consist of
- 1- observation.
- 2- electrical stimulation.
- 3- resection of
- heterotopic
- ossification.
- 4- exchange nailing.
- 5- screw fixation
- around the nail.
- answer
- back
- A
- B
- Figures 1
- Question 01.108
- Answer = 5
- back to this question
- next question
- Reference(s)
- Kellam JF, Fischer TJ, Tornetta P III, Bosse MJ, Harris MB (eds): Orthopaedic Knowledge Update: Trauma 2. Rosemont, IL, American Academy of Orthopaedic
- Surgeons, 2000, pp 177-190.
- 01.109 What is the most common mechanism of injury to the adult spinal cord?
- 1- Gunshot wounds
- 2- Falls
- 3- Motor vehicle accidents
- 4- Sporting injuries
- 5- Suicide attempts
- answer
- back
- Question 01.109
- Answer = 3
- back to this question
- next question
- Reference(s)
- Slucky AV, Eismont FJ: Treatment of acute injury of the cervical spine. Instr Course Lect 1995;44:67-80. Vaccaro AR, An HS, Betz RR, Coder JM, Balderston RA: The management of acute spinal trauma: Prehospital and in-hospital emergency care. Instr Course Lect 1997;46:113-125.
- 01.110 A 33-year-old woman has had progressive rheumatoid arthritis for the past 12
- years. Figure 31 shows the postoperative radiograph. What is the most likely
- long-term symptomatic complication following this procedure?
- 1- Nonunion of the first metatarsophalangeal joint
- 2- Chronic infection
- 3- Plantar callosities
- 4- Recurrent hammer toe deformity
- 5- Interphalangeal joint arthritis of the great toe
- answer
- back
- Figure 31
- Question 01.110
- Answer = 3
- back to this question
- next question
- Reference(s)
- Coughlin MJ: Rheumatoid forefoot reconstruction: A long-term follow-up study. J Bone Joint Surg Am 2000;82:322-341. Mizel MS, Miller RA, Scioli MW (eds): Orthopaedic Knowledge Update: Foot and Ankle 2. Rosemont, IL, American Academy of Orthopaedic Surgeons, 1998,
- pp 293-303.
- 01.111 A 12-year-old boy is referred for evaluation of a spinal deformity. Examination
- and history reveal that he and one of his two sisters have numerous light brown
- birthmarks distributed on all areas of the body. The radiograph shown in Figure
- 32 reveals a curve that measures 70°. The best course of action should include
- 1- MRI of the entire spine.
- 2- brace treatment with a thoracolumbosacral
- orthosis.
- 3- posterior spinal fusion and observation for
- possible crankshaft progression of the curve.
- 4- a period of observation to determine whether the
- curve may progress.
- 5- a renal ultrasound and echocardiogram.
- answer
- back
- Figure 32
- Question 01.111
- Answer = 1
- back to this question
- next question
- Reference(s)
- Crawford AH: Neurofibromatosis, in Weinstein SL (ed): The Pediatric Spine: Principles and Practice. New York, NY, Raven Press, 1994, pp 619-650. Crawford AH, Gabriel KR: Dysplastic scoliosis: Neurofibromatosis, in Bridwell KH, DeWald RL (eds): The Textbook of Spinal Surgery, ed 2. Philadelphia, PA,
- Lippincott-Raven, 1997, pp 276-298.
- 01.112 A 33-year-old man fell from a height of 25 feet and sustained a fracture-
- dislocation of the right elbow. Following closed reduction of the elbow in the
- emergency department, plain radiographs show a displaced radial neck and a
- type II coronoid fracture. Examination reveals elbow swelling and wrist
- tenderness. The neurologic examination is normal. Treatment of the elbow
- should include
- 1- excision of the radial head and coronoid fragment.
- 2- excision of the radial head and open reduction and internal fixation of the coronoid.
- 3- open reduction and internal fixation of the radial head and excision of the coronoid
- fragment.
- 4- open reduction and internal fixation of the radial head and coronoid.
- 5- replacement of the radial head and excision of the coronoid fragment.
- answer
- back
- Question 01.112
- Answer = 4
- back to this question
- next question
- Reference(s)
- Hotchkiss RN: Displaced fractures of the radial head: Internal fixation or excision? Am Acad Orthop Surg 1997;5:1-10. Money BF: Acute and chronic instability of the elbow. J Am Acad Orthop Surg 1996;4:117-128. Began W, Money B: Fractures of the coronoid process of the ulna,. J Bone Joint Surg Am 1989;71:1348-1354.
- 01.113 A 21-year-old patient sustains a fracture of the humeral shaft that is treated as
- shown in Figures 33a and 33b. The reported incidence of shoulder problems
- following this method of fracture stabilization is approximately what percent?
- 1- 0% to 5%
- 2- 6% to 10%
- 3- 11% to 15%
- 4- 16% to 20%
- 5- 21 % to 25 %
- answer
- back
- A
- B
- Figures 33
- Question 01.113
- Answer = 1
- back to this question
- next question
- Reference(s)
- Chapman JR, Henley MB, Agel J, Benca PJ: Randomized prospective study of humeral shaft fracture fixation: Intramedullary nails versus plates. J Orthop Trauma 2000;14:162-166. McCormack RG, Briers D, Buckley RE, McKee MD, Powell J, Schemitsch EH: Fixation of fractures of the shaft of the humerus by dynamic compression plate or intramedullary nail: A prospective randomized trial. J Bone Joint Surg Br 2000;82:336-339.
- 01.114 A revision total knee arthroplasty was performed on a patient who was
- presumed to have aseptic loosening. Antibiotic-impregnated cement was used
- at the time of reimplantation. Interface membrane cultures taken routinely at
- the time of the revision revealed coagulase-negative staphylococci 2 days after
- surgery. Appropriate management should consist of
- 1- antibiotic therapy alone.
- 2- surgical debridement with polyethylene exchange.
- 3- one-staged exchange of both components.
- 4- two-staged exchange of both components.
- 5- knee fusion.
- answer
- back
- Question 01.114
- Answer = 1
- back to this question
- next question
- Reference(s)
- Segawa H, Tsukayama DT, Kyle RF, Becker DA, Gustilo RB: Infection after total knee arthroplasry: A retrospective study of the treatment of eighty-one infections.
- J Bone Joint Sur- Am 1999;81:1434-1445.
- 01.115 A 38-year-old man underwent anterior cruciate ligament reconstruction 12
- weeks ago. Examination of the knee now reveals active arc of motion from 15°
- to 80°. Patellar mobility is limited. Radiographs reveal appropriate tunnel
- placement. Management should now consist of
- 1- physical therapy.
- 2- continuous passive motion.
- 3- open debridement.
- 4- arthroscopic debridement.
- 5- manipulation under anesthesia.
- answer
- back
- Question 01.115
- Answer = 4
- back to this question
- next question
- Reference(s)
- Lindenfeld TN, Wojtys EM, Husain A: Surgical treatment of arthrofibrosis of the knee. Instr Course Lect 2000;49:211-221. Richmond JC, al Assal M: Arthroscopic management of arthrofibrosis of the knee, including infrapatellar contraction syndrome. Arthroscopy 1991;7:144-147.
- 01.116 Which of the following radiographic findings is most helpful in differentiating
- chordoma from chondrosarcoma of the sacrum?
- 1- Bright signal intensity on T1-weighted MRI scan
- 2- Lytic destruction
- 3- Midline location
- 4- Uptake on bone scan
- 5- Soft-tissue extension
- answer
- back
- Question 01.116
- Answer = 3
- back to this question
- next question
- Reference(s)
- Smith J, Ludwig RL, Marcove RC: Sacrococcygeal chordoma: A clinicoradiological study of 60 patients. Skeletal Radiol 1987: 16:37-44. Firooznia H, Pinto RS, Lin JP, Baruch HH, Zausner J: Chordoma: Radiologic evaluation of 20 cases. Am J Roentgenol 1976;127:797-805. Temple WJ: Sacral bone tumors. Can J Surg 1994;37:446.
- 01.117 A 21-year-old student sustains a twisting injury to the ankle. Examination
- reveals some tenderness over the anteromedial ankle and significant pain and
- tenderness over the lateral malleolus. A stress radiograph in external rotation is
- shown in Figure 34. Definitive treatment should consist of
- 1- weight bearing as tolerated in a functional brace.
- 2- a short leg cast with no weight bearing for 6 weeks.
- 3- a long leg cast with no weight bearing for 6 weeks.
- 4- open reduction and internal fixation of the lateral malleolus.
- 5- open reduction and internal fixation of the lateral malleolus and repair of the deltoid
- ligament.
- answer
- back
- Figure 34
- Question 01.117
- Answer = 1
- back to this question
- next question
- Reference(s)
- Principles of orthopaedic practice, in Dee R (ed): Ankle Injuries. New York, NY, McGraw Hill, 1997, pp 538-543. Kellam JF, Fischer TJ, Tornetta P III, Bosse MJ, Harris MB (eds): Orthopaedic Knowledge Update: Trauma 2. Rosemont, IL, American Academy of Orthopaedic
- Surgeons, 2000, pp 203-225.
- 01.118 The mother of a 1-month-old infant reports that he seems to sniff and snort
- persistently and his temperature has been very labile. Examination reveals a
- depressed nasal bridge, mild jaundice, and hepatosplenomegaly. Radiographs
- are shown in Figures 35a and 35b. To help confirm the diagnosis, the best
- course of action would be to order
- 1- a rapid plasma reagin (RPR) test.
- 2- a technetium Tc 99m total body bone scan.
- 3- CT of the skull and upper cervical spine.
- 4- needle aspiration of both tibiae.
- 5- bilateral hip ultrasound studies.
- answer
- back
- A
- B
- Figures 35
- Question 01.118
- Answer = 1
- back to this question
- next question
- Reference(s)
- Richards BS (ed): Orthopaedic Knowledge Update: Pediatrics. Rosemont, IL, American Academy of Orthopaedic Surgeons, 1996, pp 35-46. Brion LP, Manuli M, Rai B, Kresch MJ, Pavlov H, Glaser J: Long-bone radiographic abnormalities as a sign of active congenital syphilis in asymptomatic newborns. Pediatrics 1991;88:1037-1040
- 01.119 Figure 36 shows an axial T1-weighted MRI scan at L4-5. The arrow is
- pointing to what structure?
- 1- Spinous process
- 2- Herniated disk
- 3- Epidural fat
- 4- Ligamentum flavum
- 5- Facet joint
- answer
- back
- Question 01.119
- Answer = 3
- back to this question
- next question
- Reference(s)
- Author states there is no reference.
- 01.120 What is the most severe side effect of doxorubicin (Adriamycin)?
- 1- Neurotoxicity
- 2- Ototoxicity
- 3- Cardiac toxicity
- 4- Hemorrhagic cystitis
- 5- Pulmonary fibrosis
- answer
- back
- Question 01.120
- Answer = 3
- back to this question
- next question
- Reference(s)
- Beaty 1H (ed): Orthopaedic Knowledge Update 6. Rosemont, IL, American Academy of Orthopaedic Surgeons, 1999, pp 167-189. Burgert EO Jr, Nesbit ME, Garnsey LA, et al: Multimodal therapy for the management of nonpelvic, localized Ewing's sarcoma of bone: Intergroup study IESS-II J Clin Oncol 1990;8:1514-1524.
- 01.121 Which of the following procedures is most likely to result in a hooked nail
- deformity following fingertip amputation?
- 1- Split-thickness skin grafting
- 2- Reimplantation of the avulsed skin
- 3- Local advancement flap
- 4- Nail removal for associated nail bed repair
- 5- Primary closure
- answer
- back
- Question 01.121
- Answer = 5
- back to this question
- next question
- Reference(s)
- Zook EG, Brown RE: The perionychium, in Green DP, Hotchkiss RN, Pederson WC (eds): Green's Operative Hand Surgery, ed 4. New York, NY, Churchill
- Livingstone, 1999, pp 1353-1380. Zook EG, Russell RC: Reconstruction of a functional and esthetic nail. Hand Clin 1990;6:59-68.
- 01.122 Which of the following shoe insert (orthosis) materials has the greatest shock-
- absorbing properties?
- 1- Cross-linked polyethylene foam
- 2- Rubberized cork
- 3- Polypropylene
- 4- Carbon epoxy resin
- 5- Natural leather
- answer
- back
- Question 01.122
- Answer = 1
- back to this question
- next question
- Reference(s)
- Shiba N, Kitaoka HB, Cahalan TD, Chao EY: Shock-absorbing effect of shoe insert materials commonly used in management of lower extremity disorders. Clin Orthop 1995;310:130-136. Wapner KL: Conservative treatment of the foot, in Coughlin MJ, Mann RA (eds): . Surgery of the Foot and Ankle, ed 7. St Louis, MO, Harcourt Health Science, 1999, pp 115-130.
- 01.123 What type of major pelvic ring injury has the greatest average transfusion
- requirement?
- 1- Lateral compression
- 2- Vertical shear
- 3- Anteroposterior compression
- 4- Fractures through the sacrum
- 5- Fractures through the iliac wing
- answer
- back
- Question 01.123
- Answer = 3
- back to this question
- next question
- Reference(s)
- Turen CH, Dube MA, LeCroy MC: Approach to the polytraumatized patient with musculoskeletal injuries. J Am Acad Orthop Surg 1999;7:154-165. Dalal SA, Burgess AR, Siegel JH, et al: Pelvic fracture in multiple trauma: Classification by mechanism is key to pattern of organ injury, resuscitative requirements, and outcome. J Trauma 1989;29:981-1002.
- 01.124 Where does the nutrient artery to the tibia most commonly enter the bone?
- 1- Anteriorly above the pes anserinus
- 2- Anteriorly 4 cm above the ankle
- 3- Laterally above the fibular head
- 4- Posteriorly below the posterior cruciate ligament insertion
- 5- Posteriorly in the medial malleolus
- answer
- back
- Question 01.124
- Answer = 4
- back to this question
- next question
- Reference(s)
- Brinker MR, Cook SD, Dunlap IN, Christakis P, Elliott MN: Early changes in nutrient artery blood flow following tibial nailing with and without reaming: A preliminary study. J Orthop Trauma 1999;13:129-133.
- 01.125 An 11-year-old soccer player has had left lateral ankle pain for the past 6
- months. Examination shows increased heel valgus and decreased subtalar
- motion on the left side. Ankle range of motion and stability are symmetric.
- Radiographs of the foot and ankle are normal. The next most appropriate step
- in management should consist of
- 1- observation with follow-up in 6 months.
- 2- a full shoe orthosis with medial heel posting left.
- 3- left sinus tarsi injections with local anesthetic.
- 4- comparative stress radiographs of both ankles.
- 5- CT of the left foot.
- answer
- back
- Question 01.125
- Answer = 5
- back to this question
- next question
- Reference(s)
- Beaty JH (ed): Orthopaedic Knowledge Update 6. Rosemont, IL., American Academy of Orthopaedic Surgeons, 1999, pp 583-595. Wechsler RJ, Schweitzer ME, Deely DM, Horn BD, Pizzutillo PD: Tarsal coalition: Depiction and characterization with CT and MR imaging. Radiology
- 1994;193:447-452.
- 01.126 What complication is significantly more common among octogenarians who
- undergo concomitant bilateral total knee arthroplasty than those who undergo
- unilateral total knee arthroplasty?
- 1- Deep wound infection
- 2- Aseptic loosening
- 3- Periprosthetic fracture
- 4- Congestive heart failure
- 5- Pneumonia
- answer
- back
- Question 01.126
- Answer = 4
- back to this question
- next question
- Reference(s)
- Lynch NM, Trousdale RT, Ilstrup DM: Complications after concomitant bilateral total knee arthroplasty in elderly patients. Mayo Clin Proc 1997;72:799-805. Beaty JH (ed): Orthopaedic Knowledge Update 6. Rosemont, IL, American Academy of Orthopaedic Surgeons, 1999, pp 559-582.
- 01.127 A 53-year-old man has had low back pain
- and perineal fullness for the past 18
- months. A plain radiograph is shown in
- Figure 37a, a CT scan is shown in Figure
- 37b, and a biopsy specimen is shown in
- Figure 37c. What is the most likely
- diagnosis?
- 1- Chordoma
- 2- Plasmacytoma
- 3- Fibrosarcoma
- 4- Liposarcoma
- 5- Metastatic carcinoma
- answer
- back
- A
- B
- Figures 37
- C
- Question 01.127
- Answer = 1
- back to this question
- next question
- Reference(s)
- Primary bone tumors, in McCarthy EF, Frassica FJ (eds): Pathology of Bone and Joint Disorders with Clinical and Radiographic Correlation. Philadelphia, PA, WB Saunders, 1998, pp 195-276. Bruckner 1D, Conrad EU: Spine, in Simon MA. Springfield D (eds): Surgery for Bone and Soft-Tissue Tumors. Philadelphia, PA, Lippincott-Raven, 1998, pp 435-450.
- 01.128 What is the most common soft-tissue tumor in children?
- 1- Ganglion
- 2- Hemangioma
- 3- Fibroma
- 4- Lipoma
- 5- Sarcoma
- answer
- back
- Question 01.128
- Answer = 2
- back to this question
- next question
- Reference(s)
- Conrad EU, Enneking WR: Clinical Symposium: Common Soft Tissue Tumors, ed 2. New York, NY, Ciba-Geigy, 1990.
- 01.129 Which of the following is considered the most important factor in fracture
- healing in adults?
- 1- Age of the patient
- 2- Gender of the patient
- 3- Neurologic status of the extremity
- 4- Blood supply
- 5- Fracture pattern
- answer
- back
- Question 01.129
- Answer = 4
- back to this question
- next question
- Reference(s)
- Buckwalter JA, Einhorn TA, Simon SR (eds): Orthopaedic Basic Science: Biology and Biomechanics of the Musculoskeletal System, ed 2. Rosemont, IL, American
- Academy of Orthopaedic Surgeons, 2000, pp 372-399.
- 01.130 A 67-year-old woman reports progressively worsening buttock pain.
- Examination reveals a mass affixed to the region of the posterior ilium. Figures
- 38a through 38d show a plain radiograph, a posterior view bone scan, a CT
- scan, and a biopsy specimen. What is the most likely diagnosis?
- 1- Dedifferentiated chondrosarcoma
- 2- Malignant fibrous histiocytoma
- 3- Fibrosarcoma
- 4- Pagetoid osteosarcoma
- 5- Osteomyelitis
- back
- A
- B
- Figures 38
- Go to next slide for remaining
- figures and
- answer link
- Figures 38
- D
- C
- answer
- back to question
- Question 01.130
- Question 01.130
- Answer = 4
- back to this question
- next question
- Reference(s)
- Grimer RJ, Carter SR, Tillman RM, et al: Osteosarcoma of the pelvis. J Bone Joint Surg Br 1999;81:796-802. Harrington KD: Surgical management of neoplastic complications of Paget's disease. J Bone Miner Res 1999;2:45-48.
- 01.131 A 23-year-old woman has had chronic swelling in the ankle for the past year
- with no history of injury. Plain radiographs are normal. At ankle arthroscopy,
- the articular surfaces appear normal. The synovial tissue is inflamed and friable;
- a biopsy specimen of the synovium is shown in Figure 39. What is the most
- likely diagnosis?
- 1- Tuberculosis
- 2- Synovial cell sarcoma
- 3- Pigmented villonodular synovitis
- 4- Rheumatoid arthritis
- 5- Chondrocalcinosis
- answer
- back
- Figure 39
- Question 01.131
- Answer = 3
- back to this question
- next question
- Reference(s)
- Ghert MA, Scully SP, Harrelson JM: Pigmented villonodular synovitis of the foot and ankle: A review of six cases. Foot Ankle Int 1999;20:326-330. Rao AS, Vigorita VJ: Pigmented villonodular synovitis (giant-cell tumor of the tendon sheath and synovial membrane): A review of eighty-one cases. J Bone Joint Surg Am 1984;66:76-94.
- 01.132 Figure 40 shows the radiograph of a 24-year-old woman who has ulnar-sided
- wrist pain. Nonsurgical management consisting of splinting, physical therapy,
- and activity modifications has failed to provide relief. Examination reveals a
- stable distal radioulnar joint and a negative triangular fibrocartilage complex
- grind. Pain is reproduced when the wrist is dorsiflexed and the forearm is then
- supinated but not when the forearm is
- pronated. Treatment should now consist of
- 1- partial ulnar styloidectomy.
- 2- triangular fibrocartilage complex rim repair.
- 3- debridement of the extensor carpi ulnaris tendon
- sheath.
- 4- a radial lengthening osteotomy.
- 5- an ulnar shortening osteotomy.
- answer
- back
- Figure 40
- Question 01.132
- Answer = 1
- back to this question
- next question
- Reference(s)
- Topper SM, Wood MB, Ruby LK: Ulnar styloid impaction syndrome. J Hand Surg Am 19972:699-704. Topper SM, Wood MB, Ruby LK Ulnar styloid impaction syndrome, in Sulfar P, Amadio PC, Foucher G (eds): Current Practice in Hand Surgery. London, England, Martin Dunitz, 1997, pp 261-268.
- 01.133 A patient with diabetes mellitus has an ulcer on the plantar aspect of the foot.
- Which of these test results best correlates with the patient's ability to heal this
- ulcer?
- 1- Toe pressures of greater than 45 mm Hg
- 2- An arterial brachial index of 0.40
- 3- Capillary refill time in the toes of greater than 3 seconds
- 4- Ability to detect greater than a 5.07 Semmes -Weinstein monofilament
- 5- A hemoglobin A3 level of greater than 4%
- answer
- back
- Question 01.133
- Answer = 1
- back to this question
- next question
- Reference(s)
- Mizel MS, Miller RA, Scioli MW (eds): Orthopaedic Knowledge Update: Foot and Ankle 2. Rosemont, IL, American Academy of Orthopaedic Surgeons, 1998, pp 113-121. Brodsky JW: The diabetic foot. in Coughlin MJ. Mann RA (eds)- Surgery of the root and Ankle, ed 7. St Louis, MO, Harcourt Health Science, 1999, pp 895-969.
- 01.134 A 42-year-old woman who has had a low-grade aching pain in her forearm for
- the past 4 years recently fell on the arm and now reports markedly increased
- pain. A plain radiograph and biopsy specimen are shown in Figures 41a and
- 41b. What is the most likely diagnosis?
- 1- Chondroblastoma
- 2- Chondrosarcoma
- 3- Chondromyxoid fibroma
- 4- Adamantinoma
- 5- Enchondroma
- answer
- back
- A
- Figures 41
- B
- Question 01.134
- Answer = 2
- back to this question
- next question
- Reference(s)
- Weis L: Common malignant bone tumors: Chondrosarcoma, in Simon MA, Springfield D (eds): Surgery for Bone and Soft-Tissue Tumors. Philadelphia, PA, Lippincott-Raven, 1998, pp 275-286. Primary bone tumors, in McCarthy EF, Frassica FJ (eds): Pathology of Bone and Joint Disorders with Clinical and Radiographic Correlation. Philadelphia, PA, WB Saunders, 1998, pp 195-276.
- 01.135 To prevent abnormal patellofemoral contact loading after insertion of a
- retrograde femoral nail through an intra-articular starting portal, the surgeon
- should
- 1- seat the nail beneath the articular surface.
- 2- use a patellar tendon-splitting approach.
- 3- use an unreamed technique.
- 4- perform a lateral release.
- 5- perform a medial parapatellar arthrotomy.
- answer
- back
- Question 01.135
- Answer = 1
- back to this question
- next question
- Reference(s)
- Moed B, Watson JT: Retrograde nailing of the femoral shaft. J Am Acad Orthop Surg 1999;7:209-216. Morgan E, Ostrum RF, DiCicco J, McElroy J, Poka A: Effects of retrograde femoral intramedullary nailing on the patellofemoral articulation. J Orthop Trauma 1999;13:13-16.
- 01.136 A woman who is in the 20th week of her pregnancy seeks an orthopaedic
- consultation after undergoing an ultrasound. The findings reveal that the fetus
- has bilateral clubfeet and both femurs measure less than two standard
- deviations below normal. What is the most likely diagnosis?
- 1- Myelomeningocele
- 2- Bilateral proximal focal femoral deficiency
- 3- Diastrophic dysplasia
- 4- Achondroplasia
- 5- Spondylometaphyseal dysplasia
- answer
- back
- Question 01.136
- Answer = 3
- back to this question
- next question
- Reference(s)
- Horton WA, Hall JG, Scott CI, Pyeritz RE, Rimoin DL: Growth curves for height for diastrophic dysplasia, spondyloepiphyseal dysplasia congenita, and pseudoachondroplasia. Am J Dis Child 1982;136:316-319. Ryoppy S, Poussa M, Merikanto J, Marttinen E, Kaitila I: Foot deformities in diastrophic dysplasia: An analysis of 102 patients. J Bone Joint Surg Br 1992;74:441-44.4.
- 01.137 A 47-year old woman has right groin pain. An AP radiograph of the pelvis and
- a biopsy specimen are shown in Figures 42a and 42b. What is the most likely
- diagnosis?
- 1- Ollier's disease
- 2- Paget's disease
- 3- Metastatic carcinoma
- 4- Aneurysmal bone cyst
- 5- Fibrous dysplasia
- answer
- back
- A
- B
- Figures 42
- Question 01.137
- Answer = 5
- back to this question
- next question
- Reference(s)
- Primary bone tumors, in McCarthy EF, Frassica FJ (eds): Pathology of Bone and Joint Disorders with Clinical and Radiographic Correlation. Philadelphia, PA, WB Saunders, 1998, pp 195-276. Gitelis S, McDonald DJ: Common benign bone tumors and usual treatment, in Simon MA, Springfield D (eds): Surgery for Bone and Soft-Tissue Tumors. Philadelphia, PA, Lippincott-Raven, 1998, pp 275-286.
- 01.138 Acute hypotension, hypoxemia, cardiac arrest, and sudden death are events that
- are most commonly encountered in what stage of total hip arthroplasty?
- 1- Exposure
- 2- Acetabular reaming
- 3- Broaching the femur
- 4- Impaction of the acetabular component
- 5- Cementing of the femoral component
- answer
- back
- Question 01.138
- Answer = 5
- back to this question
- next question
- Reference(s)
- Pitto RP, Koessler M, Kuehle JW: Comparison of fixation of the femoral component without cement and fixation with use of a bone-vacuum cementing technique for the prevention of fat embolism during total hip arthroplasry: A
- prospective, randomized clinical trial. J Bone Joint Surg Am 1999;81:831-843.
- 01.139 A 35-year-old woman falls on the ice and sustains an isolated minimally
- displaced radial head fracture. Management should include
- 1- use of a posterior splint and sling at all times for 6 weeks.
- 2- use of a sling until radiologic union is achieved.
- 3- application of a cast in 7 to 10 days.
- 4- application of a hinged elbow orthosis in 3 weeks.
- 5- a program of active range of motion in 1 week.
- answer
- back
- Question 01.139
- Answer = 5
- back to this question
- next question
- Reference(s)
- Morrey BF: Radial head fractures, in Morrey BF (ed): The Elbow and Its Disorders. Philadelphia, PA, WE Saunders, 1985, pp 355-381. Levine AM (ed): Orthopaedic Knowledge Update: Trauma. Rosemont, IL, American Academy of Orthopaedic Surgeons, 1996, pp 47-55.
- 01.140 An asymptomatic 10-year-old boy is referred for evaluation of a limb-length
- discrepancy that measures less than 2 cm. Examination reveals that the most
- lateral ray of the ipsilateral foot is absent, and the ipsilateral knee is unstable to
- Lachman and anterior drawer tests. Figure 43 shows an AP radiograph of the
- ankle. Management of the knee should consist of
- 1- anterior cruciate ligament reconstruction using a
- quadruple hamstring technique.
- 2- anterior cruciate ligament reconstruction using an
- allograft in the over-the-top position.
- 3- observation.
- 4- an aggressive physical therapy program that
- emphasizes open chain techniques.
- 5- functional knee bracing until skeletal maturity,
- followed by anterior cruciate ligament reconstruction.
- answer
- back
- Figure 43
- Question 01.140
- Answer = 3
- back to this question
- next question
- Reference(s)
- Achterman C, Kalamchi A: Congenital deficiency of the fibula. J Bone Joint Surg Br 1979;61:133-137. Roux MO, Cariioz H: Clinical examination and investigation of the cruciate ligaments in children with fibular hemimelia. J Pediatr Orthop 1999;19:247-251. Stevens PM, Arms D: Postaxial hypoplasia of the lower extremity. J Pediatr Orthop 2000;20:166-172.
- 01.141 A nonrandomized prospective study of the efficacy of a new diagnostic test to
- detect deep venous thrombosis has just been completed. Each patient had
- venography as the definitive test to detect the thrombosis. Of the 100 patients
- tested, 10 were true positives, 30 were false negatives, 40 were true negatives,
- and 20 were false positives. What is the sensitivity of the new test?
- 1- 25 %
- 2- 30%
- 3- 50 %
- 4- 59%
- 5- 67 %
- answer
- back
- Question 01.141
- Answer = 1
- back to this question
- next question
- Reference(s)
- Buckwalter JA, Einhorn TA, Simon SR (eds): Orthopaedic Basic Science: Biology and Biomechanics of the Musculoskeletal System, ed 2. Rosemont, IL, American
- Academy of Orthopaedic Surgeons, 2000, pp 308-316.
- 01.142 When treating tibial plateau fractures, the most important reason that proximal
- tibial transfixation wires should be placed at least 14 mm from the articular
- surface is to
- 1- obtain good fixation in subchondral bone.
- 2- allow radiographic visualization of the joint reduction.
- 3- minimize the risk that septic arthritis will develop.
- 4- increase the range of knee flexion.
- 5- compress the articular fragments.
- answer
- back
- Question 01.142
- Answer = 3
- back to this question
- next question
- Reference(s)
- DeCoster TA, Crawford M.K, Kraut VIA: Safe extracapsular placement of proximal tibia transfixation pins. J Orthop Trauma 1999;13:236-240. Reid JS, Van Slyke MA, Moulton MJ, Mann TA: Safe placement of proximal tibial transfixation wires with respect to intracapsular penetration. J Orthop Trauma 2001;15:10-17.
- 01.143 A 17-year-old girl reports a 4-month history of progressively worsening left
- arm pain. A plain radiograph and biopsy specimen are shown in Figures 44a
- and 44b. After complete staging, management should consist of
- 1- forequarter amputation alone.
- 2- radiation therapy and chemotherapy.
- 3- radiation therapy and wide surgical excision.
- 4- chemotherapy and wide surgical excision.
- 5- wide surgical excision alone.
- answer
- back
- A
- B
- Figures 44
- Question 01.143
- Answer = 4
- back to this question
- next question
- Reference(s)
- Primary bone tumors, in McCarthy EF, Frassica FJ (eds): Pathology of Bone and Joint Disorders with Clinical and Radiographic Correlation. Philadelphia., PA.
- WB Saunders, 1998, pp 195-276. Weis L: Common malignant bone tumors: Osteosarcoma, in Simon MA, Springfield D (eds): Surgery for Bone and Soft-Tissue Tumors. Philadelphia, PA, Lippincott-Raven, 1998, pp 265-274. Goorin A: Chemotherapy for osteosarcoma and Ewing's sarcoma, in Simon MA, Springfield D (eds): Surgery for Bone and Soft-Tissue Tumors. Philadelphia, PA,
- Lippincott-Raven, 1998, pp 239-244.
- 01.144 Figure 45 shows the initial clinical photograph of a 70-year-old woman. The
- deformity of the second toe may recur after attempted surgical reconstruction.
- Dysfunction of what structure allows for recurrence?
- 1- Dorsal capsule
- 2- Plantar plate
- 3- Medial collateral ligament
- 4- Lateral collateral ligament
- 5- Transverse intermetatarsal ligament
- answer
- back
- Figure 45
- Question 01.144
- Answer = 2
- back to this question
- next question
- Reference(s)
- Yao L, Cracchiolo A, Farahani K, Seeger LL: Magnetic resonance imaging of plantar plate rupture. Foot Ankle Int 1996;17:33-36. Deland JT, Sung IH: The medial crossover toe: A cadaveric dissection. Foot Ankle Int 2000;21:375-378.
- 01.145 A 25-year-old rugby player who sustained blunt trauma to the right dominant
- long finger 2 days ago now reports pain over the dorsum of the digit and
- clicking when he flexes and extends the digit. Examination reveals swelling
- and ecchymosis over the metacarpophalangeal joint, as well as a palpable
- subluxation of the extrinsic extensor tendon over the metacarpophalangeal
- joint with joint flexion and extension. Management should consist of
- 1- surgical repair of the sagittal band.
- 2- surgical repair of the spiral oblique retinacular ligament.
- 3- surgical repair of the triangular ligament.
- 4- splinting the metacarpophalangeal joint at 70° of flexion.
- 5- a program of early active motion with buddy taping.
- answer
- back
- Question 01.145
- Answer = 1
- back to this question
- next question
- Reference(s)
- Ishizuki M: Traumatic and spontaneous dislocation of extensor tendon of the long finger. J Hand Surg Am 1990;15:967-972. Rayan GM, Murray D: Classification and treatment of closed sagittal band injuries. J Hand Surg Am 1994;19:590-594.
- 01.146 When placing lateral mass screws in the midcervical spine, where is the
- vertebral artery located in relation to the starting point for screw insertion?
- 1- Medial
- 2- Lateral
- 3- Anterior
- 4- Posterior
- 5- Superior
- answer
- back
- Question 01.146
- Answer = 3
- back to this question
- next question
- Reference(s)
- Levine AM: Orthopaedic Knowledge Update: Trauma. Rosemont, IL, American Academy of Orthopaedic Surgeons, 1996, pp 335-339.
- 01.147 A 30-year-old man underwent an arthroscopic Bankart repair with suture
- anchors 6 months ago. While the patient reports that the shoulder is stable, he
- notes anterior shoulder pain and crepitation. Figure 46 shows an arthroscopic
- view of the anterior shoulder joint. The next most appropriate step in
- management should consist of
- 1- an intra-articular culture.
- 2- rotator cuff repair.
- 3- removal of the anchors.
- 4- arthroscopic releases.
- 5- exchange of the metallic
- anchor for a bioabsorbable
- device.
- answer
- back
- Figure 46
- Question 01.147
- Answer = 3
- back to this question
- next question
- Reference(s)
- Kaar TY, Schenck RC Jr, Worth MA, Rockwood CA Jr.- Complications of metallic suture anchors in shoulder surgery: A report of 8 cases. Arthroscopy 2001;17:31-37. Zuckerman JD, Matsen FA III: Complications about the glenohumeral joint related to the use of screws and staples. J Bone Joint Surg Am 1984;66:175-180
- 01.148 A 32-year-old man notes a lump on the side of his neck and undergoes a lymph
- node biopsy. Following the procedure, the patient reports pain in the shoulder
- girdle and is unable to elevate his shoulder. Which of the following structures
- has most likely been injured?
- 1- Trapezius muscle
- 2- Sternocleidomastoid muscle
- 3- Axillary nerve
- 4- Cranial nerve XI
- 5- Suprascapular nerve
- answer
- back
- Question 01.148
- Answer = 4
- back to this question
- next question
- Reference(s)
- Leffert RD: Neurologic problems, in Rockwood CA, Matsen FA (eds): The Shoulder. Philadelphia, PA, WB Saunders, 1990, p 759. Bigliani LU, Perez-Sanz JR, Wolfe IN: Treatment of trapezius paralysis. J Bone Joint Surg Am 1985;67:871-877.
- 01.149 A 40-year-old woman with steroid-dependent Crohn's disease has had pain and swelling
- of the left ankle for the past 5 days. She has a plugged central line for parenteral feeding.
- Examination of the ankle reveals focal inflammation and limited range of motion. She is
- sensitive to the 4.17 Semmes-Weinstein monofilament test. Radiographs are shown in
- Figures 47a and 47b. The next most appropriate step in management should consist of
- 1- a biopsy of the talus.
- 2- a bone scan.
- 3- aspiration and culture of the ankle.
- 4- no weight bearing and a total contact cast for 1 week.
- 5- a brace with calipers.
- answer
- back
- A
- B
- Figures 47
- Question 01.149
- Answer = 3
- back to this question
- next question
- Reference(s)
- Mielants H, Veys EM: The gut in the pondyloarthropathies. J Rheumatol 1990;17:7-10. Myerson M: Foot and Ankle Disorders. Philadelphia, PA, WB Saunders, 2000, pp 1215-1216.
- 01.150 What structures are located within the femoral canal as it passes beneath the
- inguinal ligament?
- 1- Femoral artery, femoral vein, and femoral nerve
- 2- Femoral artery and femoral vein
- 3- Femoral artery, femoral vein, and lymphatics
- 4- Femoral artery, femoral vein, femoral nerve, and lymphatics
- 5- Internal iliac artery, internal iliac vein, and femoral nerve
- answer
- back
- Question 01.150
- Answer = 3
- back to this question
- next question
- Reference(s)
- Hoppenfeld S, deBoer P (eds): Surgical Exposures in Orthopaedics: The Anatomic Approach. ed 2. Philadelphia. PA. JB Lippincott, 1994. pp 362-275.
- 01.151 A 6-month-old boy with L1 myelomeningocele has bilateral dislocated hips.
- Examination reveals that the hips are clinically reducible, and there are no
- significant hip or knee joint contractures. The best course of action for both
- hips should consist of
- 1- observation.
- 2- application of a Pavlik harness.
- 3- application of a rigid hip abduction orthosis.
- 4- closed reduction with a hip spica cast.
- 5- open reduction via an adductor approach.
- answer
- back
- Question 01.151
- Answer = 1
- back to this question
- next question
- Reference(s)
- Heeg M, Broughton NS, Menelaus MB: Bilateral dislocation of the hip in spins bifida: A long-term follow-up study. J Pediatr Orthop 1998;18:434-436. Broughton NS, Menelaus MB, Cole WG, Shurtleff DB: The natural history of hip deformity in myelomeningocele. J Bone Joint Surg Br 1993;75:760-763.
- 01.152 Sterilization of ultra-high molecular-weight polyethylene by irradiation in an
- inert environment (argon, nitrogen, or vacuum) is recommended because it
- 1- increases crystallinity.
- 2- prevents free radical formation.
- 3- prevents immediate oxidative degradation.
- 4- prevents component shrinkage.
- 5- provides better sterility.
- answer
- back
- Question 01.152
- Answer = 3
- back to this question
- next question
- Reference(s)
- Beaty JH (ed): Orthopaedic Knowledge Update 6. Rosemont, IL, American Academy of Orthopaedic Surgeons, 1999, pp 47-53. Deng M, Shalaby SW: Lang-term gamma irradiation effects on ultrahigh molecular weight polyethylene. J Biomed Mater Res 2001;54:428-435.
- 01.153 A 13-year-old girl reports activity-related pain in her left leg with no history of
- trauma. Figures 48a through 48d show a plain radiograph, T1- and T2-weighted
- MRI scans, and a biopsy specimen. What is the most likely diagnosis?
- 1- Ewing's sarcoma
- 2- Osteomyelitis
- 3- Fibrous dysplasia
- 4- Adamantinoma
- 5- Tibial stress fracture
- back
- A
- B
- Figures 48
- Go to next slide for remaining figures and answer link
- Figures 48
- D
- C
- answer
- back to question
- Question 01.153
- Question 01.153
- Answer = 5
- back to this question
- next question
- Reference(s)
- Jeske JM, Lomasney LM, Demos TC, Vade A, Bielski RJ: Longitudinal tibial stress fracture. Orthopedics 1996;19:263,66,68,70. Shearman CM, Brandser EA, Parman LM, et al: Longitudinal tibial stress fractures: A report of eight cases and review of the literature. J Comput Assist Tomogr 1998;22:265-269.
- 01.154 A 30-year-old woman who underwent total hip arthroplasty for osteonecrosis 6
- months ago is now seeking a second opinion for her limp. What is the most
- significant radiographic finding shown in Figure 49?
- 1- Contralateral osteonecrosis
- 2- Prosthetic loosening
- 3- Heterotopic ossification
- 4- Stress shielding
- 5- Limb-length inequality
- answer
- back
- Figure 49
- Question 01.154
- Answer = 5
- back to this question
- next question
- Reference(s)
- Woolson ST: Leg length equalization during total hip replacement. Orthopedics 1990;13:17-21. Shaw JA, Greet RB III: Complications of total hip replacement, in Epps CH Jr (ed): Complication in Orthopaedic Surgery. Philadelphia, PA, JB Lippincott,
- 1994, pp 1013-1056.
- 01.155 Figure 50 shows the radiograph of a 24-year-old patient who has a slightly painful swollen distal finger. What is the most likely diagnosis?
- 1- Glomus tumor
- 2- Giant cell tumor
- 3- Intraosseous ganglion
- 4- Foreign body granuloma
- 5- Inclusion cyst
- answer
- back
- Question 01.155
- Answer = 5
- back to this question
- next question
- Reference(s)
- Schajowicz F, Aiello CL, Slullitel I: Cystic and pseudocystic lesions of the terminal phalanx with special reference to epidermoid cysts. Clip Orthop 1970;68:84-92. Athanasian EA: Bone and soft-tissue tumors, in Green DP, Hotchkiss RN, Pederson WC (eds): Green's Operative Hand Surgery, ed 4. New York, NY, Churchill Livingstone, 1999, pp 2223-2253.
- 01.156 A 4-year-old boy has a painful spinal deformity. Figures 51a through 51d show
- a lateral spine radiograph, whole body bone scan, a lateral T1-weighted MRI
- scan, and a needle biopsy specimen. What is the most likely diagnosis?
- 1- Tuberculosis
- 2- Metastatic neuroblastoma
- 3- Chordoma
- 4- Diskitis
- 5- Ewing's sarcoma
- back
- A
- B
- Figures 51
- Go to next slide
- for remaining
- figures and
- answer link
- Figures 51
- D
- C
- answer
- back to question
- Question 01.156
- Question 01.156
- Answer = 1
- back to this question
- next question
- Reference(s)
- Mushkin AY, Kovalenko KN: Neurological complications of spinal tuberculosis in children. Int Orthop 1999;23:210-212. Sudarshan K: Tuberculosis of bones and joints. J Bone Joint Surg Am 1997;79:1891.
- 01.157 A 19-year-old man sustained a fracture-dislocation of the ankle and a talar neck
- and body fracture when his foot was run over by a truck. Examination reveals
- no pulse in the ankle and an ischemic foot. The anterior and posterior tibial
- arteries are transected. He has no other injuries. A clinical photograph is shown
- in Figure 52. Treatment should consist of
- 1- repair of the posterior tibial artery and ankle fusion.
- 2- repair of the anterior tibial artery and pantalar fusion.
- 3- repair of both tibial arteries and internal fixation of the talus.
- 4- repair of the posterior tibial artery and nerve and external fixation.
- 5- amputation.
- answer
- back
- Figure 52
- Question 01.157
- Answer = 5
- back to this question
- next question
- Reference(s)
- Gregory P, Sanders R: The management of severe fractures of the lower extremities. Clip Orthop 1995;318:95-105.
- Tornetta P III, Olson SA (eds): Amputation versus limb salvage. Instr Course Lect 1997;46:511-518.
- 01.158 In an animal model, the use of anabolic steroids on muscle contusion injury has
- been shown to
- 1- be similar to placebo controls.
- 2- be similar to corticosteroids with respect to muscle strength recovery in the long
- term.
- 3- be more effective than corticosteroids with respect to muscle strength recovery in the
- long term.
- 4- be less effective than corticosteroids with respect to muscle strength recovery in the
- long term.
- 5- result in severely disorganized muscle fiber architecture.
- answer
- back
- Question 01.158
- Answer = 3
- back to this question
- next question
- Reference(s)
- Beiner JM, Jokl P, Cholewicki J, Panjabi MM: The effect of anabolic steroids and corticosteroids on healing of muscle contusion injury. Am J Sports Med 1999;27:2-9. Tingus SJ, Carlsen RC: Effect of continuous infusion of an anabolic steroid on marine skeletal muscle. Med Sci Sports Exert 1993;25:485-494.
- 01.159 When performing a surgical debridement for a painful irreparable rotator cuff
- tear, it is important to
- 1- tenodese the biceps.
- 2- excise the distal clavicle (Mumford procedure).
- 3- preserve the coracohumeral ligament.
- 4- preserve the coracoacromial ligament.
- 5- advance the deltoid origin.
- answer
- back
- Question 01.159
- Answer = 4
- back to this question
- next question
- Reference(s)
- Ellman H, Hanker G, Bayer M: Repair of the rotator cuff: End-result study of factors influencing reconstruction. J Bone Joint Surg Am 1986;68:1136-1144. Hanyman DT II, Mack LA. Wang KY, Jackins SE, Richardson ML, Matsen FA III: Repairs of the rotator cuff: Correlation of functional results with integrity of
- the cuff. J Bone Joint Surg Am 1991;73:982-989.
- 01.160 A patient with developmental dysplasia of the hip is undergoing open
- reduction. Which of the following is considered an advantage of using a medial
- approach compared with an anterior approach?
- 1- A lower incidence of osteonecrosis
- 2- Access for performance of capsulorrhaphy
- 3- Access to the transverse acetabular ligament
- 4- Better ability to reduce an inverted labrum
- 5- Better visualization of the lateral femoral cutaneous nerve
- answer
- back
- Question 01.160
- Answer = 3
- back to this question
- next question
- Reference(s)
- Turner Y, Ward WT, Grudziak J: Medial open reduction in the treatment of developmental dislocation of the hip. J Pediatr Orthop 1997;17:176-180. Mankey MG, Arntz GT, Staheli LT: Open reduction through a medial approach for congenital dislocation of the hip: A critical review of the Ludloff approach in sixty-six hips. J Bone Joint Surg Am 1993;75:1334-1345.
- 01.161 A 35-year-old laborer sustains a irreparable fracture of the radial head after
- falling 12 feet. Examination reveals tenderness at the elbow and wrist and pain
- with manipulation of the distal radioulnar joint. In addition to resection of the
- radial head, management should include
- 1- hinged bracing of the elbow.
- 2- immobilization in a long arm cast.
- 3- prosthetic replacement of the radial head.
- 4- repair of the distal radioulnar joint.
- 5- application of a hinged external fixator.
- answer
- back
- Question 01.161
- Answer = 3
- back to this question
- next question
- Reference(s)
- Kellam JF, Fischer TJ, Tornetta P III, Bosse MJ, Harris MB (eds): Orthopaedic Knowledge Update: Trauma 2. Rosemont, IL,, American Academy of Orthopaedic Surgeons, 2000, pp 39-51. Hotchkiss RN: An KN, Sowa DT, Banta S, Weiland AJ: An anatomic and mechanical study of the interosseous membrane of the forearm: Pathomechanics of proximal migration of the radius. J Hand Surg Am 1989;14:256-261.
- 01.162 Which of the following treatments has been shown to prevent the formation of
- heterotopic ossification after total hip arthroplasty in patients who are at high
- risk?
- 1- Alendronate
- 2- Acetaminophen
- 3- Preoperative radiation
- 4- Calcitonin
- 5- Parathormone
- answer
- back
- Question 01.162
- Answer = 3
- back to this question
- next question
- Reference(s)
- Beaty JH (ed): Orthopaedic Knowledge Update 6. Rosemont, IL, American Academy of Orthopaedic Surgeons, 1999, pp 455-492. Knelles D, Barthel T, Karrer A, Kraus U, Eulert J, Kolbl O: Prevention of heterotopic ossification after total hip replacement: A prospective, randomised study using acetylsalicylic acid, indomethacin and fractional or single-dose, irradiation. J Bone Joint Surg Br 1997;79:596-602. Pellegrini VD Jr, Gregoritch SJ: Preoperative irradiation for prevention of heterotopic ossification following total hip arthroplasty. J Bone Joint Sung Am 1996;78:870-881.
- 01.163 A 38-year-old landscaper was treated with internal and external fixation for a
- severe pilon fracture. Radiographs obtained at 3 months and 1 year are shown
- in Figures 53a and 53b. He now reports increasing pain over the past 4 months
- and is unable to walk uphill or stand for more than 2 hours. Examination
- reveals range of motion from neutral to 5° of plantar flexion. Use of a short leg
- brace with a rocker bottom sole after screw removal provides some pain relief,
- but he still has too much pain to work. Management should now consist of
- 1- manipulation of the ankle under anesthesia.
- 2- arthroscopic debridement of the ankle.
- 3- ankle arthroplasty.
- 4- ankle fusion.
- 5- subtalar fusion.
- answer
- back
- A
- B
- Figures 53
- Question 01.163
- Answer = 4
- back to this question
- next question
- Reference(s)
- Kellam JF, Fischer TJ, Tornetta P III, Bosse MJ, Harris MB (eds): Orthopaedic Knowledge Update: Trauma 2. Rosemont, IL, American Academy of Orthopaedic
- Surgeons, 2000, pp 191-202.
- 01.164 Which of the following factors best predicts the increased risk for development
- of a foot ulcer in a patient with diabetes mellitus?
- 1- A history of a previous foot ulcer
- 2- A history of poor blood glucose control
- 3- Type I diabetes for more than 10 years
- 4- Ability to detect a 5.07 Semmes-Weinstein monofilament, on the plantar surface of
- the foot
- 5- Nonpalpable pulses with an ankle-brachial index (ABI) of greater than 0.5
- answer
- back
- Question 01.164
- Answer = 1
- back to this question
- next question
- Reference(s)
- McDermott JE (ed): The Diabetic Foot. Rosemont, IL, American Academy of Orthopaedic Surgeons, 1995, pp 1-12. Mizel MS, Miller RA, Scioli MW (eds): Orthopaedic Knowledge Update: Foot and Ankle 2. Rosemont, IL, American Academy of Orthopaedic Surgeons, 1998,
- pp 113-12I. Brodsky JW: The diabetic foot, in Coughlin MJ, Mann RA (eds): Surgery of the Foot and Ankle, ed 7. St Louis, MO, Harcourt Health Science, 1999, pp 895-969.
- 01.165 A study is being designed to compare the results of two new drugs on bone
- mineral density. The number of subjects needed for this study should be
- determined by
- 1- Student's t test.
- 2- power analysis.
- 3- probability distribution.
- 4- regression analysis.
- 5- Spearman rank correlation.
- answer
- back
- Question 01.165
- Answer = 2
- back to this question
- next question
- Reference(s)
- Buckwalter JA, Einhorn TA, Simon SR (eds): Orthopaedic Basic Science: Biology and Biomechanics of the Musculoskeletal System, ed 2. Rosemont, IL, American Academy of Orthopaedic Surgeons, 2000, pp 2-17.
- 01.166 A 17-year-old basketball player sustains an ankle eversion injury on a fast
- break. She notes immediate anteromedial ankle pain and swelling and is unable
- to bear weight. The next most appropriate step in management should consist
- of
- 1- ice and protected weight bearing.
- 2- functional ankle bracing treatment.
- 3- a short leg cast.
- 4- radiographs.
- 5- MRI.
- answer
- back
- Question 01.166
- Answer = 4
- back to this question
- next question
- Reference(s)
- Roberts CS, DeMaio M, Larkin JJ, Paine R: Eversion ankle sprains. Orthopedics 1995;18:299-304. Beaty JH (ed): Orthopaedic Knowledge Update 6. Rosemont, IL, American Academy of Orthopaedic Surgeons 1999, pp 597-612.
- 01.167 Following reinsertion of the distal biceps tendon, early rehabilitation should
- include
- 1- active elbow flexion and active forearm supination.
- 2- active elbow flexion and passive forearm supination.
- 3- active elbow extension and active forearm supination.
- 4- passive elbow extension and active forearm supination.
- 5- passive elbow flexion and passive forearm supination.
- answer
- back
- Question 01.167
- Answer = 5
- back to this question
- next question
- Reference(s)
- Money BF: Tendon injuries about the elbow, in Moray BF (ed): The Elbow and Its Disorders, ed 2. Philadelphia, PA, WB Sounders, 1993, pp 492-504. Sotereanos DG, Pierce TD, Varitimidis SE: A simplified method for repair of distal biceps tendon ruptures. J Shoulder Elbow Sung 2000;9:227-233.
- 01.168 In the absence of a visible fracture on radiographs, the presence of a positive
- posterior fat pad sign following trauma of the elbow in a 5-year-old child most
- likely represents
- 1- a normal radiographic finding.
- 2- a soft-tissue contusion.
- 3- nursemaid's elbow.
- 4- an occult fracture.
- 5- synovial hypertrophy.
- answer
- back
- Question 01.168
- Answer = 4
- back to this question
- next question
- Reference(s)
- Skaggs DL, Mirzayan R: The posterior fat pad sign in association with occult fracture of the elbow in children. J Bone Joint Sung Am 1999;81:1429-1433. Donnelly LF, Klostermeier TT, Klosterman LA: Traumatic elbow effusions in pediatric patients: Are occult fractures the rule? Am J Roentgenol 1998;171:243-245.
- 01.169 Thumb adduction in low ulnar nerve palsy is provided by the
- 1- extensor pollicis longus.
- 2- extensor pollicis brevis.
- 3- flexor pollicis brevis.
- 4- first dorsal interosseous.
- 5- accessory head of the flexor pollicis longus (Gantzer's muscle).
- answer
- back
- Question 01.169
- Answer = 1
- back to this question
- next question
- Reference(s)
- Smith RJ: Tendon Transfers of the Hand and Forearm. Boston, MA, Little Brown, 1987, pp 85-102. Hamlin C, Littler JW: Restoration of power pinch. J Hand Surg Am 1980;5:396-401.
- 01.170 The parents of a 6-month-old infant report that she has been unwilling to move
- her left upper extremity for the past 5 hours. An AP radiograph and an MRI
- scan are shown in Figures 54a and 54b. Based on these findings and after
- initial treatment, a consultation should be arranged with
- 1- child protection services.
- 2- a geneticist.
- 3- a nephrologist.
- 4- a rheumatologist.
- 5- an infectious disease
- specialist.
- answer
- back
- A
- B
- Figures 54
- Question 01.170
- Answer = 1
- back to this question
- next question
- Reference(s)
- DeLee JC, Wilkins KE, Rogers LF, Rockwood CA: Fracture-separation of the distal humeral epiphysis. J Bone Joint Surg Am 1980;62:46-51. Cramer KE, Green NE: Child abuse, in Green NE, Swiontkowski MF (eds): Skeletal Trauma in Children, ed 2. Philadelphia, PA, WB Sounders, 1998, pp 577-594. Nimkin K, Kleinman PK. Teeger S, Spevak MR: Distal humeral physeal injuries in child abuse: MR imaging and ultrasonography findings. Pediatr Radiol 1995;25:562-565. Kocher MS, Kasser JR: Orthopaedic aspects of child abuse. J Am Acad Orthop Surg 2000;8:10-20.
- 01.171 Where are the sacral roots located within the canal at the L1-2 disk level?
- 1- Random pattern
- 2- Anterior
- 3- Central
- 4- Lateral
- 5- Posterior
- answer
- back
- Question 01.171
- Answer = 3
- back to this question
- next question
- Reference(s)
- Wall E1, Cohen MS, Abitbol JJ, Garfin SR: Organization of intrathecal nerve roots at the level of the conus medullaris. J Bone Joint Surg Am 1990;72:1495-1499. Wall FJ, Cohen MS, Massie JB, Rydevik B, Garfin SR: Cauda equina anatomy: L Intrathecal nerve root organization. Spine 1990;15:1244-1247.
- 01.172 University of California Biomechanics Laboratory (UCBL) lower extremity
- orthoses are thought to work by
- 1- supporting the forefoot in rigid deformities.
- 2- supporting the midfoot in rigid deformities.
- 3- controlling the hindfoot in flexible deformities.
- 4- controlling the ankle when instability is present.
- 5- accommodating a forefoot deformity.
- answer
- back
- Question 01.172
- Answer = 3
- back to this question
- next question
- Reference(s)
- Mizel MS, Miller RA, Scioli MW (eds): Orthopaedic Knowledge Update: Foot and Ankle 2. Rosemont, IL, American Academy of Orthopaedic Surgeons, 1998, pp 55-64. Wapner KL: Conservative treatment of the foot, in Coughlin MJ, Mann RA (eds): Surgery of the Foot and Ankle, ed 7. St Louis, MO, Harcourt Health Science, 1999, pp 115-130.
- 01.173 What portion of the calcaneus typically maintains a normal relationship to the
- talus in displaced intra-articular calcaneus fractures?
- 1- Sustentaculum tali
- 2- Tuberosity
- 3- Anterolateral
- 4- Posterior facet
- 5- Lateral wall
- answer
- back
- Question 01.173
- Answer = 1
- back to this question
- next question
- Reference(s)
- Olexa TA, Ebraheim NA, Haman SP: The sustentaculum tall: Anatomic, radiographic, and surgical considerations. Foot Ankle Int 2000;21:400-403. Sanders R: Intro articular fractures of the calcaneus: Present state of the art. J Orthop Trauma 1992;6:252-265.
- 01.174 The linear relationship between an applied stress and the resultant deformation
- defines a material's
- 1- modulus of elasticity.
- 2- brittleness.
- 3- yield strength.
- 4- ultimate strength.
- 5- toughness.
- answer
- back
- Question 01.174
- Answer = 1
- back to this question
- next question
- Reference(s)
- Simon SR (ed): Orthopaedic Basic Science. Rosemont, IL, American Academy of Orthopaedic Surgeons, 1994, pp 447-486.
- 01.175 In total hip arthroplasty, which of the following characterizes the clinical pain
- pattern seen with a loose femoral component?
- 1- Gluteal
- 2- Night
- 3- Start-up
- 4- Back
- 5- At rest
- answer
- back
- Question 01.175
- Answer = 3
- back to this question
- next question
- Reference(s)
- Beaty JH (ed): Orthopaedic Knowledge Update 6. Rosemont, IL, American Academy of Orthopaedic Surgeons, 1999, pp 455-492. Katz RP, Callaghan JJ, Sullivan PM, Johnston RC: Long-term results of revision total hip arthroplasty with improved cementing technique. J Bone Joint Sung Br 1997;79:322-326. Krishnamurthy AB, MacDonald SJ, Paprosky WG: 5- to 13-year follow-up study on cementless femoral components in revision surgery. J Arthroplasty 1997;12:839-847.
- 01.176 What is the natural history of a nonossifying fibroma?
- 1- Gradual enlargement after skeletal maturity
- 2- Spontaneous resolution with skeletal maturity
- 3- Chronic pain
- 4- Late malignant degeneration
- 5- Angular deformity
- answer
- back
- Question 01.176
- Answer = 2
- back to this question
- next question
- Reference(s)
- Jaffe HL, Lichtenstein L: Non-osteogenic fibroma of bane. Am J Pathol 1942;18:205. Unni KK: Conditions that commonly simulate primary neoplasms of bone, in Dahlin's Bone Tumors, ed 5. Philadelphia, PA, 1996, pp 355-432.
- 01.177 What structure is outlined and lies at the tip of the arrow shown in Figure 55?
- 1- Inferior articular facet
- 2- Superior articular facet
- 3- Pedicle
- 4- Lamina
- 5- Spinous process
- answer
- back
- Figure 55
- Question 01.177
- Answer = 1
- back to this question
- next question
- Reference(s)
- Parke WW: Applied Anatomy of the Spine, in Herkowitz HN, Eismont FJ, Garfin SR, Bell GR, Balderston RA, Wiesel SW (eds): Rothman-Simeone: The Spine, ed 4. Philadelphia, PA, WB Saunders, 1999, pp 27-73. Netter FH (ed): The Ciba Collection of Medical Illustrations: Musculoskeletal System, Part L Anatomy, Physiology, and Metabolic Disorders. Summit, NJ,
- Ciba-Geigy, 1987, vol 8, pp 9-19.
- 01.178 Which of the following systemic conditions is associated with a genetic defect
- in skeletal formation that does not involve abnormal collagen?
- 1- Osteogenesis imperfecta
- 2- Spondyloepiphyseal dysplasia congenita
- 3- Achondroplasia
- 4- Multiple epiphyseal dysplasia
- 5- Kneist dysplasia
- answer
- back
- Question 01.178
- Answer = 3
- back to this question
- next question
- Reference(s)
- Dietz FR, Matthews KD: Update on the genetic bases of disorders with orthopaedic manifestations. J Bone Joint Surg Am 1996;78:1583-1598.
- 01.179 What is the most likely cause of mortality within the first 48 hours in patients
- who sustain a pelvic fracture from a lateral compression mechanism?
- 1- Aortic rupture
- 2- Pelvic arterial injury
- 3- Pelvic venous injury
- 4- Hollow viscous injury
- 5- Head injury
- answer
- back
- Question 01.179
- Answer = 5
- back to this question
- next question
- Reference(s)
- Dalal SA, Burgess AR, Siegel 3H, et al: Pelvic fracture in multiple trauma: Classification by mechanism is key to pattern of organ injury, resuscitative requirements, and outcome. J Trauma 1989;29;981-1002.
- 01.180 Following total hip arthroplasty, examination of the patient reveals an ischemic
- leg. A radiograph obtained in the recovery room is shown in Figure 56a, and a
- subtraction arteriogram is shown in Figure 56b. What artery has been injured?
- 1- Profunda femoris
- 2- External iliac
- 3- Obturator
- 4- Superior gluteal
- 5- Inferior gluteal
- answer
- back
- A
- B
- Figures 56
- Question 01.180
- Answer = 2
- back to this question
- next question
- Reference(s)
- Wasielewski RC, Cooperstein LA, Kruger MP, Rubash HE: Acetabular anatomy and the transacetabular fixation of screws in total hip arthroplasty. J Bone Joint Surg Am 1990;72:501-508. Shaw JA, Greer RB III: Complications of total hip replacement, in Epps CH Jr (ed): Complication in Orthopaedic Surgery. Philadelphia, PA, JB Lippincott,
- 1994, pp 1013-1056.
- 01.181 Which of the following axial pattern flaps is best used to repair fingertip
- amputations?
- 1- Axial flag
- 2- First dorsal metacarpal artery
- 3- Second dorsal metacarpal artery
- 4- Reversed dorsal metacarpal artery
- 5- Digital artery island
- answer
- back
- Question 01.181
- Answer = 5
- back to this question
- next question
- Reference(s)
- Lai CS, Lin SD, Yang CC: The reverse digital artery flap for fingertip reconstruction. Ann Plant Surg 1989;22:495-500. Lister GD, Pederson WC: Skin flaps, in Green DP, Hotchkiss RN, Pederson WC (eds): Green's Operative Hand Surgery, ed 4. New York, NY, Churchill Livingstone, 1999, pp 1783-1850.
- 01.182 What is the most common complication seen in patients undergoing surgery on
- the Achilles tendon for chronic refractory tendinitis?
- 1- Deep vein thrombosis
- 2- Partial/complete tendon rupture
- 3- Skin edge necrosis
- 4- Infection
- 5- Complex regional pain syndrome
- answer
- back
- Question 01.182
- Answer = 3
- back to this question
- next question
- Reference(s)
- Paavola M, Orava S, Leppilahti J, Kannus P, Jarvinen M: Chronic Achilles tendon overuse injury: Complications after surgical treatment: An analysis of 432 consecutive patients. Am J Sports Med 2000;28:77-82. Williams J: Achilles tendon lesions in sport. Sports Med 1986;3:114-135.
- 01.183 Figures 57a and 57b show the radiographs of a college basketball player who
- has had lateral foot pain for the past 3 weeks. Management should consist of
- 1- weight bearing as tolerated with continued play.
- 2- weight bearing as tolerated in a short leg cast.
- 3- restricted weight bearing for 4 weeks, followed by an early return to play.
- 4- electrical bone stimulation.
- 5- percutaneous screw fixation.
- answer
- back
- A
- B
- Figures 57
- Question 01.183
- Answer = 5
- back to this question
- next question
- Reference(s)
- Mindrebo N, Shelboume KD, Van Meter CD, Rettig AC: Outpatient percutaneous screw fixation of the acute Jones fracture. Am J Sports Med 1993;21:720-723. Weinfeld SB, Haddad SL, Myerson MS: Metatarsal stress fractures. Clip Sports Med 1997;16:319-338.
- 01.184 A 62-year-old man has pain with overhead activities and shoulder weakness.
- History reveals that he sustained an anterior dislocation of the right shoulder 18
- months ago and underwent open repair of the subscapularis, supraspinatus, and
- infraspinatus tendons 2 weeks after the injury. Examination reveals active total
- elevation of 160°, active external rotation of 50°, and passive internal rotation
- to T9. He has 5/5 deltoid, 4-/5 external rotation, and 5/5 internal rotation
- strength. What is the most likely cause of his symptoms?
- 1- Residual rotator cuff tear
- 2- Adhesive capsulitis
- 3- Glenohumeral instability
- 4- Axillary neuropathy
- 5- Suprascapular neuropathy
- answer
- back
- Question 01.184
- Answer = 1
- back to this question
- next question
- Reference(s)
- Gerber C, Fucks B, Holler J: The results of repair of massive tears of the rotator cuff. J Bone Joint Surg Am 2000;82:505-515. Harryman DT II, Mack LA, Wang KY, Jackins SE, Richardson ML, Matsen FA III: Repairs of the rotator cuff: Correlation of functional results with integrity of the cuff. J Bone Joint Surg Am 1991;73:982-989.
- 01.185 A 6-year-old girl has a right genu varum that has progressed over the last 6
- months. Plain radiographs of the knee reveal a sloping medial joint line with an
- obvious bony bar at the medial proximal tibial physis. A CT scan shows that
- this bar involves about 20% of the physis. Treatment at this time should include
- 1- epiphyseodesis of the proximal tibial physis.
- 2- corrective osteotomy of the tibia and fibula.
- 3- corrective osteotomy of the tibia and epiphyseodesis of the left proximal tibial
- physis.
- 4- proximal tibial physeal bar resection and corrective osteotomy of the tibia and fibula.
- 5- elevating osteotomy of the proximal tibial medial plateau.
- answer
- back
- Question 01.185
- Answer = 4
- back to this question
- next question
- Reference(s)
- Beaty JH (ed): Orthopaedic Knowledge Update 6. Rosemont, IL, American Academy of Orthopaedic Surgeons, 1999, pp 505-520. Tachdjian MA: Tibia vara, in Pediatric Orthopaedics. Philadelphia, PA, WB Saunders, 1990, p 2846. Greene WB: Infantile tibia vara. Instr Course Lect 1993;42:525-538.
- 01.186 An 18-year-old patient has the painful lesion shown in Figure 58a. A biopsy
- specimen is shown in Figure 58b. Management should consist of
- 1- preoperative chemotherapy and wide resection.
- 2- wide resection only.
- 3- marginal distal ulna resection.
- 4- curettage and bone grafting.
- 5- low-dose radiation.
- answer
- back
- A
- B
- Figures 58
- Question 01.186
- Answer = 4
- back to this question
- next question
- Reference(s)
- Martinet V, Sissons HA: Aneurysmal bone cyst: A review of 123 cases including primary lesions and those secondary to other bone pathology. Cancer 1988;61:2291-2304. Biesecker JL, Marcove RC, Huvos AG, Mike V: Aneurysmal bone cysts: A clinicopathologic study of 66 cases. Cancer 1970;26:615-625. Gibbs CP Jr, Hefele MC, Peabody TD, Montag AG, Aithal V, Simon MA: Aneurysmal bone cyst of the extremities: Factors related to local recurrence after curettage with a high-speed burr. J Bone Joint Surg Am 1999;81:1671-1678.
- 01.187 The value of intercondylar notch visualization of the posterior compartment
- during anterior cruciate ligament reconstruction is to identify
- 1- a loose body.
- 2- a popliteus injury.
- 3- meniscal root tears.
- 4- an injury to the ligament of Wrisberg.
- 5- an osteochondral injury.
- answer
- back
- Question 01.187
- Answer = 3
- back to this question
- next question
- Reference(s)
- Amin KB, Cosgarea AJ, Kaeding CC: The value of intercondylar notch visualization of the posteromedial and posterolateral compartments during knee arthroscopy. Arthroscopy 1999;15:813-817. Boytim MJ, Smith JP, Fischer DA, Quick DC: Arthroscopic posteromedial visualization of the knee. Clip Orthop 1995;310:82-86.
- 01.188 Figures 59a and 59b show the radiographs of an 8-year-old boy who has
- atraumatic recurrent lateral dislocation of the left patella. Examination reveals
- no fixed genu varum or valgum, and the lower extremity lengths are equal. The
- Q angle is 25°. The extended hips show internal rotation of 40° and external
- rotation of 60°, with a neutral thigh-foot angle. There is no generalized
- ligamentous laxity. Treatment should consist of
- 1- femoral rotational osteotomy.
- 2- tibial rotational osteotomy.
- 3- tibial tuberosity transfer (Fulkerson, Elmslie-Trillat,
- or Hauser).
- 4- tenodesis of the semitendinosus to the patella.
- 5- patellectomy and vastus medialis advancement.
- answer
- back
- A
- B
- Figures 59
- Question 01.188
- Answer = 4
- back to this question
- next question
- Reference(s)
- Hall JE, Micheli LJ, McManama GB Jr- Semitendinosus tenodesis for recurrent subluxation or dislocation of the patella. Clin Orthop 1979;144:31-35. Sponseller PD, Beaty JH: Fractures and dislocations about the knee, in Rockwood CA, Wilkins KE, Beaty JH (eds): Fractures in Children, ed 4. Philadelphia, PA, Lippincott-Raven, 1996, pp 1231-1329. Tolo V: Fractures and dislocations about the knee, in Green NE, Swiontkowski MF (eds): Skeletal Trauma in Children, ed 2. Philadelphia, PA, WB Saunders, 1998, pp 431-458. Lefts RM, Davidson D, Beaule P: Semitendinosus tenodesis for repair of recurrent dislocation of the patella in children. J Pediatr Orthop 1999;19:742-747.
- 01.189 The posterior (Thompson) approach to the proximal radial shaft lies between
- the
- 1- extensor carpi ulnaris and anconeus.
- 2- extensor carpi ulnaris and extensor carpi radialis longus.
- 3- extensor carpi radialis longus and extensor carpi radialis brevis.
- 4- extensor carpi radialis brevis and extensor digitorum communis.
- 5- brachioradialis and extensor carpi radialis longus.
- answer
- back
- Question 01.189
- Answer = 4
- back to this question
- next question
- Reference(s)
- Hoppenfeld S, deBoer P (eds): Surgical Exposures in Orthopaedics: The Anatomic Approach. Philadelphia, PA, Lippincott Williams & Wilkins, 1984, pp 109-139. Thompson BE: Anatomical methods of approach in operations on the long bones of the extremities. Ann Surg 1918;68:309.
- 01.190 Concurrent injuries to which of the following structures results in an increased
- osteogenic response to fracture?
- 1- Head
- 2- Liver
- 3- Chest
- 4- Major artery
- 5- Bladder
- answer
- back
- Question 01.190
- Answer = 1
- back to this question
- next question
- Reference(s)
- Kushwaha VP, Garland DG: Extremity fractures in the patient with a traumatic brain injury. J Am Acad Orthop Surg 1998;6:298-307. Spencer RF: The effect of head injury on fracture healing: A quantitative assessment. J Bone Joint Surg Br 1987;69:525-528.
- 01.191 A 29-year-old man sustained a talar neck fracture with an associated
- dislocation of the body of the talus from the subtalar and tibiotalar joints. He
- was treated with immediate open reduction and internal fixation. Twelve weeks
- later, the fracture has united, and lucency is observed in the superior
- subchondral bone of the talar body. The next most appropriate step in
- management should consist of
- 1- bone grafting of the talar neck and body.
- 2- application of an ultrasound bone stimulator.
- 3- a brace with calipers.
- 4- MRI to assess body viability.
- 5- protected weight bearing.
- answer
- back
- Question 01.191
- Answer = 5
- back to this question
- next question
- Reference(s)
- Mizel MS, Miller RA, Scioli MW (eds): Orthopaedic Knowledge Update: Foot and Ankle 2. Rosemont, IL, American Academy of Orthopaedic Surgeons, 1998,
- pp 201-213. Clanton TO: Athletic injuries to the soft tissues of the foot and ankle, in Coughlin MJ, Mann RA (eds): Surgery of the Foot and Ankle, ed 7. St Louis, MO, Harcourt Health Science, 1999, p 1176.
- 01.192 Within the normal healthy lumbar disk of a young person, proteoglycans
- constitute a
- 1- low percent of dry weight within the annulus, a high percent of dry weight within the
- nucleus, and interact with H2O to primarily resist compression.
- 2- low percent of dry weight within the annulus, a high percent of dry weight within the
- nucleus, and interact with H2O to primarily resist tension.
- 3- high percent of dry weight within the annulus, a low percent of dry weight within the
- nucleus, and interact with H2O to primarily resist compression.
- 4- high percent of dry weight within the annulus, a low percent of dry weight within the
- nucleus, and interact with H2O to primarily resist tension.
- 5- high percent of dry weight within the annulus, a low percent of dry weight within the
- nucleus, and have no interaction with H2O.
- answer
- back
- Question 01.192
- Answer = 1
- back to this question
- next question
- Reference(s)
- Buckwalter JA, Einhorn TA, Simon SR (eds): Orthopaedic Basic Science: Biology and Biomechanics of the Musculoskeletal System, ed 2. Rosemont, IL, American Academy of Orthopaedic Surgeons, 2000, pp 548-556. Buckwalter JA, Pedrini-Mille A, Pedrini V, Tudisco C: Proteoglycans of human infant intervertebral disc: Electron microscopic and biochemical studies. J Bone Joint Surg Am 1985;67:284-294.
- 01.193 A 40-year-old woman has right shoulder pain and limited range of motion.
- History reveals that she sustained a right proximal humerus fracture 10 years
- ago and was treated with a sling and physical therapy. Examination reveals
- active forward elevation of 100°, active external rotation of 0°, and passive
- internal rotation to L5. Passive shoulder motion is the same. Plain radiographs
- of the shoulder are shown in Figures 60a and 60b. Treatment should now
- consist of
- 1- arthroscopic debridement.
- 2- humeral osteotomy.
- 3- core decompression.
- 4- humeral head replacement.
- 5- capsular release.
- answer
- back
- A
- B
- Figures 60
- Question 01.193
- Answer = 4
- back to this question
- next question
- Reference(s)
- Norris TR, Green A, McGuigan FX: Late prosthetic arthroplasty for displaced proximal humerus fractures. J Shoulder Elbow Surg 1995;4:271-280. Schlegel TF, Hawkins RJ: Displaced proximal humeral fractures: Evaluation and treatment. J Am Acad Orthop Surg 1994;2:54-78.
- 01.194 The lateral crista of the trochlea develops from what secondary ossification
- center?
- 1- Medial condylar epiphysis
- 2- Lateral condylar epiphysis
- 3- Medial epicondylar apophysis
- 4- Lateral epicondylar apophysis
- 5- Olecranon apophysis
- answer
- back
- Question 01.194
- Answer = 2
- back to this question
- next question
- Reference(s)
- Wilkins KE: Fractures and dislocations of the elbow region: Part L The elbow region: General concepts in the pediatric patient, in Rockwood CA, Wilkins KE, Beaty JH (eds): Fractures in Children, ed 4. Philadelphia, PA, Lippincott-Raven, 1996, pp 653-669.
- 01.195 A 40-year-old man sustains a rupture of the Achilles tendon while playing
- recreational basketball. If he undergoes surgical repair rather than nonsurgical
- management, the patient should be told to expect a
- 1- lower skin complication rate and a longer period of rehabilitation.
- 2- lower re-rupture rate and a longer period of rehabilitation.
- 3- higher re-rupture rate and a shorter rehabilitation.
- 4- higher skin complication rate and a higher re-rupture rate.
- 5- higher skin complication rate and a lower re-rupture rate.
- answer
- back
- Question 01.195
- Answer = 5
- back to this question
- next question
- Reference(s)
- Coughlin MJ: Disorders of tendons, in Coughlin MJ, Mann RA (eds): Surgery of the Foot and Ankle, ed 7. St Louis, MO, Harcourt Health Science, 1999, pp 786-861. Troop RL, Losse GM, Lane JG, Robertson DB, Hastings PS, Howard ME: Early motion after repair of Achilles tendon ruptures. Foot Ankle Int 1995;16:705-709.
- 01.196 Which of the following is considered the most sensitive sensory test for
- detecting early carpal tunnel syndrome?
- 1- Light touch sensation
- 2- Pinprick sensation
- 3- Two-point discrimination
- 4- Moving two-point discrimination
- 5- Semmes-Weinstein monofilament
- answer
- back
- Question 01.196
- Answer = 5
- back to this question
- next question
- Reference(s)
- American Society for Surgery of the Hand: Hand Surgery Update. Rosemont, B., American Academy of Orthopaedic Surgeons, 1996, pp 221-231. Szabo RM, Gelberman RH, Dimick MD: Sensibility testing in patients with carpal tunnel syndrome. J Bone Joint Surg Am 1984;66:60-64.
- 01.197 A 16-year-old boy has had pain in the lateral ankle and hindfoot after sustaining
- a minor ankle sprain 6 months ago. The pain is worse with any twisting activity
- of the foot. Examination reveals normal alignment of the foot and ankle. An AP
- radiograph of the ankle and foot is normal. A lateral radiograph is shown in
- Figure 61. What is the most likely cause of his persistent pain?
- 1- Fracture of the lateral process of the talus
- 2- Fracture of the anterior process of the calcaneus
- 3- Fracture of the tibial plafond
- 4- Talocalcaneal coalition
- 5- Stress fracture of the calcaneus
- answer
- back
- Figure 61
- Question 01.197
- Answer = 4
- back to this question
- next question
- Reference(s)
- Richardson EG: Flatfoot in children and adults, in Coughlin MJ, Mate RA (eds): Surgery of the Foot and Ankle, ed 7. St Louis, MO, Harcourt Health Science,
- 1999, pp 702-733. Scranton PE Jr: Treatment of symptomatic talocalcaneal coalition. J Bone Joint Surg Am 1987;69:533-539.
- 01.198 A patient who underwent total knee arthroplasty 2 years ago has a range of
- motion of 0° to 60°. The implants are well fixed, and the knee is well aligned
- on AP radiographs. Lateral .radiographs show that the femoral component is
- appropriately sized and the tibial component is in 5° of anterior tilt. Treatment
- should consist of
- 1- revision of the femoral component.
- 2- revision of the tibial component.
- 3- closed knee manipulation.
- 4- open lysis of adhesions.
- 5- open quadricepsplasty.
- answer
- back
- Question 01.198
- Answer = 2
- back to this question
- next question
- Reference(s)
- Callaghan JJ, Dennis DA, Paprosky WG, Rosenberg AG (eds): Orthopaedic Knowledge Update: Hip and Knee Reconstruction. Rosemont, IL, American Academy of Orthopaedic Surgeons, 1995, pp 317-322.
- 01.199 When compared to plate fixation, antegrade intramedullary nailing of humeral
- shaft fractures results in
- 1- better elbow function.
- 2- a higher rate of union.
- 3- a higher rate of complications.
- 4- a higher rate of infection.
- 5- longer surgical time.
- answer
- back
- Question 01.199
- Answer = 3
- back to this question
- next question
- Reference(s)
- Farragos AF, Schemitsch EH, McKee MD: Complications of intramedullary nailing for fractures of the humeral shaft: A review. J Orthop Trauma 1999;13:258-267. McCormack RG, Brien D, Buckley RE, McKee MD, Powell J, Schemitsch EH: Fixation of fractures of the shaft of the humerus by dynamic compression plate or intramedullary nail: A prospective randomized trial. J Bone Joint Surg Br 2000;82:336-339.
- 01.200 Which of the following groups is most at risk for osteoporosis?
- 1- Caucasian men
- 2- Caucasian women
- 3- African-American women
- 4- Hispanic men
- 5- Hispanic women
- answer
- back
- Question 01.200
- Answer = 2
- back to this question
- next question
- Reference(s)
- Lane JM, Nydick M: Osteoporosis: Current modes of prevention and treatment. J Am Acad Orthop Surg 1999;7:19-31. Melton LJ III: Epidemiology of spinal osteoporosis. Spine 1997;22:2S-115
- 01.201 A 49-year-old man with advanced glenohumeral arthritis undergoes total
- shoulder replacement. Following surgery, he reports pain relief but now has
- weakness when using his arm for activities in front of his body. He is unable to
- hold the dorsum of his hand away from his back. The weakness is most likely
- related to what muscle?
- 1- Supraspinatus
- 2- Subscapularis
- 3- Infraspinatus
- 4- Deltoid
- 5- Pectoralis major
- answer
- back
- Question 01.201
- Answer = 2
- back to this question
- next question
- Reference(s)
- Gerber C, Farrow MD: Isolated tears of the subscapularis tendon. Orthop Trans 1995;19:457. Gerber C, Hersche O, Farron A: Isolated rupture of the subscapularis tendon. J Bone Joint Surg Am 1996;78:1015-1023.
- 01.202 An ankle fracture heals with an anatomically aligned mortise and 2 mm of
- displacement of the distal fibula fracture. What affect will these findings have
- on the tibiotalar joint?
- 1- Decreased contact loading
- 2- Increased contact loading
- 3- Increased external rotation
- 4- Increased medial-lateral translation
- 5- Normal loading, rotation, and translation
- answer
- back
- Question 01.202
- Answer = 5
- back to this question
- next question
- Reference(s)
- Brown TD, Hurlbut PT, Hale JE, et a1: Effects of imposed hindfoot constraint on ankle contact mechanics for displaced lateral malleolar fractures. J Orthop Trauma 1994;8:511-519. Michelson JD: Fractures about the ankle. J Bone Joint Surg Am 1995;77:142-152.
- 01.203 Figure 62 shows the MRI scan of a 30-year-old male volleyball player who has
- had shoulder pain for the past 6 months. Which of the following physical
- findings in the shoulder would be most consistent with this lesion?
- 1- Weakness of internal rotation
- 2- Weakness of external rotation
- 3- Weakness of abduction
- 4- Positive impingement sign
- 5- Positive apprehension sign
- answer
- back
- Figure 62
- Question 01.203
- Answer = 2
- back to this question
- next question
- Reference(s)
- Thompson RC Jr, Schneider W, Kennedy T: Entrapment neuropathy of the inferior branch of the suprascapular nerve by ganglia. Clin Orthop 1982;166:185-187. Fehrman DA, Orwin JF, Jennings RM: Suprascapular nerve entrapment by ganglion cysts: A report of six cases with arthroscopic findings and review of the literature. Arthroscopy 1995;11:727-734.
- 01.204 In cemented polyethylene acetabular components, the reported average
- polyethylene wear rate on a yearly basis is how many millimeters?
- 1- 0.01
- 2- 0.05
- 3- 0.1
- 4- 1.0
- 5- 2.0
- answer
- back
- Question 01.204
- Answer = 3
- back to this question
- next question
- Reference(s)
- Beaty JH (ed): Orthopaedic Knowledge Update 6. Rosemont, IL, American Academy of Orthopaedic Surgeons, 1999, pp 455-492. Jasty M, Goetz DD, Bragdon CP, et al: Wear of polyethylene acetabular components in total hip arthroplasty: An analysis of one hundred and twenty-eight components retrieved at autopsy or revision operations. J Bone Joint Surg Am 1997;79:349-358.
- 01.205 The Lisfranc ligament connects what two bones?
- 1- Middle cuneiform and first metatarsal
- 2- Middle cuneiform and second metatarsal
- 3- Medial cuneiform and first metatarsal
- 4- Medial cuneiform and second metatarsal
- 5- Medial cuneiform and middle cuneiform
- answer
- back
- Question 01.205
- Answer = 4
- back to this question
- next question
- Reference(s)
- Sanafian SK: Osteology, in Anatomy of the Foot and Ankle: Descriptive, Topographic, Functional, ed 2. Philadelphia, PA, JB Lippincott, 1993, pp 37-112. Clanton TO: Athletic injuries to the soft tissues of the foot and ankle, in Coughlin MJ, Mann RA (eds): Surgery of the Foot and Ankle, ed 7. SL Louis, MO, Harcourt Health Science, 1999, pp 1090-1209.
- 01.206 The arrow in the axial MRI scan shown in Figure 63 is pointing to what
- muscle?
- 1- Gracilis
- 2- Adductor brevis
- 3- Sartorius
- 4- Semitendinosus
- 5- Pectineus
- answer
- back
- Figure 63
- Question 01.206
- Answer = 1
- back to this question
- next question
- Reference(s)
- Hoppenfeld S, deBoer P (eds): Surgical Exposures in Orthopaedics: The Anatomic Approach, ed 2. Philadelphia, PA, JB Lippincott, 1994, pp 401-429.
- 01.207 Item deleted after statistical review
- (and no answer or references cited)
- back
- next question
- 01.208 Which of the following conditions will most likely cause disability at some
- point in the life of a patient with achondroplasia?
- 1- Degenerative joint disease of the knees
- 2- Cervical instability with myelopathy
- 3- Scoliosis of the thoracic or lumbar spine
- 4- Spinal stenosis
- 5- Progressive contractures of the limbs
- answer
- back
- Question 01.208
- Answer = 4
- back to this question
- next question
- Reference(s)
- Pyeritz RE, Sack GH Jr, Udvarhelyi GB: Thoracolumbosacral laminectomy in achondroplasia: Long-term results in 22 patients. Am J Med Genet 1987;28:433-444. Tolo VT: Spinal deformity in skeletal dysplasia, in Weinstein SL (ed): The Pediatric Spine: Principles and Practice. New York, NY, Raven Press, 1994, pp 369-393.
- 01.209 The use of calcium supplements should be
- 1- encouraged in girls at puberty to help prevent osteoporosis.
- 2- restricted until menopause to decrease the risk of renal stones.
- 3- left to individual preference until perimenopause, then started routinely.
- 4- started only after a bone mineral density study indicates a deficiency of 2 standard
- deviations.
- 5- started only after a bone mineral density study indicates a deficiency of 1 standard
- deviation.
- answer
- back
- Question 01.209
- Answer = 1
- back to this question
- next question
- Reference(s)
- Garfin SR, Vaccaro AR (eds): Orthopaedic Knowledge Update: Spine. Rosemont, IL, American Academy of Orthopaedic Surgeons, 1997, pp 127-139.
- 01.210 The bending strength of fractured long bones fixed with an experimental
- compression plate is compared with unfractured controls. What statistical test
- should be used to compare the mean bending strength of the two groups?
- 1- Student's t test
- 2- Analysis of variance
- 3- Regression analysis
- 4- Chi-square test
- 5- Wilcoxon two-sample test
- answer
- back
- Question 01.210
- Answer = 1
- back to this question
- next question
- Reference(s)
- Simon SR (ed): Orthopaedic Basic Science. Rosemont, IL, American Academy of Orthopaedic Surgeons, 1994, pp 397-446.
- 01.211 Which of the following factors has been shown to be an independent risk factor
- for infection after open reduction and internal fixation of the calcaneus through
- an extensile lateral incision?
- 1- Age of greater than 50 years
- 2- History of smoking
- 3- A comminuted posterior facet
- 4- Extension of the fracture into the sustentaculum
- 5- Use of phenytoin
- answer
- back
- Question 01.211
- Answer = 2
- back to this question
- next question
- Reference(s)
- Folk JW, Stan AJ, Early JS: Early wound complications of operative treatment of calcaneus fractures: Analysis of 190 fractures. J Orthop Trauma 1999;13:369-372.
- 01.212 Examination of a 65-year-old woman who sustained a stroke 18 months ago
- reveals a clenched fist deformity that is causing significant hygiene problems
- because of skin maceration and malodor. She has no observed voluntary motor
- control of the hand or forearm. Management should consist of
- 1- open phenol blocks.
- 2- botulinum toxin blocks.
- 3- proximal interphalangeal and distal interphalangeal arthrodesis.
- 4- flexor tenotomies.
- 5- a superficialis-to-profundus tendon transfer.
- answer
- back
- Question 01.212
- Answer = 5
- back to this question
- next question
- Reference(s)
- Braun RN, Vise GT, Roger B: Preliminary experience with superficialis-to profundus tendon transfer in the hemiplegic upper extremity. J Bone Joint Surg Am 1974;56:466-472. Hisex MS, Keenan MAE: Orthopaedic management of upper extremity dysfunction following stroke or brain injury, in Green DP, Hotchkiss RN, Pederson WC (eds): Green's Operative Hand Surgery, ed 4. New York, NY, Churchill Livingstone, 1999, pp 287-324.
- 01.213 Revision of a failed acetabular component with a bipolar endoprosthesis and
- acetabular bone grafting is most likely to fail because of what mechanism?
- 1- Osteolysis
- 2- Femoral stem loosening
- 3- Recurrent dislocation
- 4- Bipolar head migration
- 5- Bipolar head disengagement
- answer
- back
- Question 01.213
- Answer = 4
- back to this question
- next question
- Reference(s)
- Brien WW, Bruce WJ, Salvati EA, Wilson PD 1r, Pellicci PM: Acetabular reconstruction with a bipolar prosthesis and morseled bone grafts. J Bone Joint Surg Am 1990;72:1230-1235. McFarland EG, Lewallen DG, CabaneIa ME: Use of bipolar endoprosthesis and.bone grafting for acetabular reconstruction. Clin Orthop 1991; 268:128-129. Papagelopoulus PJ, Lewallen DG, Cabanela ME, McFarland EG, Wallnichs SL: Acetabular reconstruction using bipolar endoprosthesis and bone grafting in patients with severe bone deficiency. Clin Orthop 1995;314:170-184.
- 01.214 An 18-year-old man sustains the proximal femur fracture shown in Figures 64a
- and 64b. Definitive management should consist of
- 1- traction for 6 weeks.
- 2- a functional brace.
- 3- a 135° angle sliding hip screw.
- 4- a 95° fixed angle plate.
- 5- antegrade nailing with transverse screws.
- answer
- back
- A
- B
- Figures 64
- Question 01.214
- Answer = 4
- back to this question
- next question
- Reference(s)
- Kinast C, Bolhofner BR, Mast 1W, Ganz R: Subtrochanteric fractures of the femur: Results of treatment with the 95 degrees blade-plate. Clin Orthop 1989;238:122-130.
- 01.215 In the treatment of femoral shaft fractures, the lowest union rate has been
- reported after which of the following types of nailing?
- 1- Reamed antegrade locked
- 2- Reamed antegrade unlocked
- 3- Reamed retrograde locked
- 4- Unreamed antegrade locked
- 5- Unreamed retrograde locked
- answer
- back
- Question 01.215
- Answer = 5
- back to this question
- next question
- Reference(s)
- Moed BR, Watson JT, Cramer KE, Karges DE, Teefey JS: Unreamed retrograde intramedullary nailing of fractures of the femoral shaft. J Orthop Trauma 1998;12334-342. Moed BR, Watson JT: Retrograde intramedullary nailing, without reaming, of fractures of the femoral shaft in multiply injured patients. J Bone Joint Surg Am 1995;77:1520-1527.
- 01.216 A 36-year-old man has pain in the metatarsophalangeal (MTP) joint of the
- great toe with all weight-bearing activities, and management consisting of shoe
- modification and an insert has failed to provide relief. Examination reveals a
- painful 10° arc of motion. Radiographs show degenerative changes with dorsal
- and medial osteophytes and joint narrowing. Treatment should now consist of
- 1- excision of the osteophytes and the dorsal third of the metatarsal head.
- 2- a dorsiflexion osteotomy of the metatarsal head.
- 3- resection arthroplasty of the MTP joint.
- 4- a Silastic implant of the MTP joint.
- 5- arthrodesis of the MTP joint.
- answer
- back
- Question 01.216
- Answer = 5
- back to this question
- next question
- Reference(s)
- Mann RA, Clanton TO: Hallux rigidus: Treatment by cheilectomy. J Bone Joint Surg Am 1988;70:400-406. Mizel MS, Miller RA, Scioli MW (eds): Orthopaedic Knowledge Update: Foot and Ankle 2. Rosemont, IL, American Academy of Orthopaedic Surgeons, 1998, pp 151-161. Coughlin MJ: Arthritides, in Coughlin MJ, Mann RA (eds): Surgery of the Foot and Ankle, ed 7. St Louis, MO, Harcourt Health Science, 1999, pp 605-633.
- 01.217 A 14-year-old boy who plays football sustains a valgus force to his knee.
- Examination reveals a 1+ effusion and decreased range of motion. Lachman
- test results are negative. He has no joint line tenderness, but he does have
- tenderness over the proximal origin of the medial collateral ligament and pain
- with valgus stressing. Initial plain radiographs of the knee are normal. Further
- evaluation should include
- 1- arthrography.
- 2- MRI.
- 3- a bone scan.
- 4- emergent arthroscopy.
- 5- stress radiographs.
- answer
- back
- Question 01.217
- Answer = 5
- back to this question
- next question
- Reference(s)
- Sponseller PD, Beaty JH: Fractures and dislocations about the knee, in Rockwood CA, Wilkins KE, Beaty JH (eds): Fractures in Children, ed 4. Philadelphia, PA, Lippincott-Raven, 1996, pp 1231-1329. Cook PC, Leit ME: Issues in the pediatric athlete. Orthop Clin North Am 1995;26:453-464.
- 01.218 A 35-year-old man sustained an isolated closed displaced bicondylar tibial
- plateau fracture after being struck by a car. Reduction and internal fixation of
- both condyles can be most safely achieved using which of the following
- surgical approaches?
- 1- Isolated anterior midline
- 2- Isolated lateral parapatellar
- 3- Isolated posteromedial
- 4- Combined anterolateral and posteromedial
- 5- Combined anterior midline and posterior transpopliteal
- answer
- back
- Question 01.218
- Answer = 4
- back to this question
- next question
- Reference(s)
- Georgiadis GM: Combined anterior and posterior approaches for complex tibial plateau fixations. J Bone Joint Surg Br 1994;76:285-289.
- 01.219 Which of the following conditions is most commonly associated with
- congenital fibular hemimelia?
- 1- Congenital absence of the patella
- 2- Congenital absence of the first ray
- 3- Genu varum
- 4- Equinovarus ankle
- 5- Talocalcaneal coalition
- answer
- back
- Question 01.219
- Answer = 5
- back to this question
- next question
- Reference(s)
- Grogan DP, Holt GR, Ogden JA: Talocalcaneal coalition in patients who have fibular hemimelia or proximal femoral focal deficiency: A comparison of the radiographic and pathological findings. J Bone Joint Surg Am 1994;76:1363-1370. Epps CH Jr, Schneider PL: Treatment of hemimelias of the lower extremity: Long term results. J Bone Joint Surg Am 1989;71273-277.
- 01.220 A 58-year-old woman has severe neck pain after falling at home. Examination
- reveals intact deltoid strength, 2/5 weakness in the rest of the right upper
- extremity, and 2/5 weakness in the left upper extremity, except for a grip
- strength of 3/5. She has 4/5 strength in both lower extremities, good rectal
- tone, and an intact bulbocavernosus reflex. Sensation is intact. Which of the
- following conditions best characterizes her neurologic injury?
- 1- A complete spinal cord injury
- 2- Bilateral brachial plexopathies
- 3- Multilevel cervical radiculopathy
- 4- Anterior cord syndrome
- 5- Central cord syndrome
- answer
- back
- Question 01.220
- Answer = 5
- back to this question
- next question
- Reference(s)
- McGuire RA: Physical examination in spinal trauma, in Levine AM, Eismont FJ, Garfin SR, Zigler JE (eds): Spine Trauma. Philadelphia, PA. WB Saunders, 1998, pp 17-27. Bohiman HH, Docker TB: Spine trauma in adults: Spine and spinal cord injuries, in Herkowitz HN, Eismont FJ, Garfin SR, Bell GR, Balderston RA, Wiesel SW (eds): Rothman-Simeone: The Spine, ed 4. Philadelphia, PA, WB Saunders, 1999, pp 889-914.
- 01.221 A 25-year-old construction worker reports a mass on the dorsum of his hand
- that is painful with strenuous use. Examination reveals a 4 x 2 x 1 cm soft mass
- that overlies the proximal portions of the index and middle metacarpals. It
- moves with flexion and extension of those digits, becomes firmer with forceful
- grasp, and does not transilluminate. What is the most likely diagnosis?
- 1- Dorsal wrist ganglion
- 2- Extensor tenosynovitis
- 3- Giant cell tumor of the tendon sheath
- 4- Carpal boss
- 5- Anomalous extensor muscle
- answer
- back
- Question 01.221
- Answer = 5
- back to this question
- next question
- Reference(s)
- Tan ST, Smith PJ: Anomalous extensor muscles of the hand: A review. J Hand Sung Am 1999;24:449-455. Doyle JR: Extensor tendons: Acute injuries, in Green DP, Hotchkiss RN, Pederson WC (eds): Green's Operative Hand Surgery, ed 4. New York, NY, Churchill Livingstone, 1999, pp 1950-1987.
- 01.222 Which of the following cytokines have been implicated in the pathogenesis of
- rheumatoid arthritis?
- 1- Fibroblastic growth factor and transforming growth factor
- 2- Parathyroid hormone-related protein and interferon-gamma
- 3- Interleukin- 1 and tumor necrosis factor
- 4- Interleukin-4 and granulocyte-macrophage colony-stimulating factor
- 5- Interleukin- 10 and vascular endothelial growth factor
- answer
- back
- Question 01.222
- Answer = 3
- back to this question
- next question
- Reference(s)
- Beaty 3H (ed): Orthopaedic Knowledge Update 6. Rosemont, IL, American Academy of Orthopaedic Surgeons, 1999, pp 205-216. Buckwalter JA, Einhorn TA, Simon SR (eds): Orthopaedic Basic Science: Biology and Biomechanics of the Musculoskeletal System, ed 2. Rosemont, IL, American Academy of Orthopaedic Surgeons, 2000, pp 490-530.
- 01.223 Item deleted after statistical review
- (and no answer or references cited)
- back
- next question
- 01.224 An 11-year-old girl has had progressive medial midfoot pain bilaterally for the
- past 6 months. Her mother states that the child's feet appeared normal until the
- pain started, and she is concerned about the development of flatfeet. What is
- the most likely diagnosis?
- 1- Physiologic pes planus
- 2- Charcot foot
- 3- Posterior tibial tendon insufficiency
- 4- Congenital vertical talus
- 5- Accessory navicular
- answer
- back
- Question 01.224
- Answer = 5
- back to this question
- next question
- Reference(s)
- Prichasuk S, Sinphurmsuksknl O: Kidner procedure for symptomatic accessory navicular and its relation to pes planus. Foot Ankle Int 1995;16:500-503.
- 01.225 Which of the following is considered the most important factor in preventing
- failure of fixation of displaced femoral neck fractures?
- 1- Use of a compression screw and side plate
- 2- Use of cannulated screws
- 3- Accuracy of reduction
- 4- Reduction on a traction table
- 5- Release of a capsular hematoma
- answer
- back
- Question 01.225
- Answer = 3
- back to this question
- next question
- Reference(s)
- Chug D, Jaglal SB, Schatzker J: Predictors of early failure of fixation in the treatment of displaced subcapital hip fractures. J Orthop Trauma 1998;12:230-234. Swiontkowski MF: Intracapsular fractures of the hip. J Bone Joint Surg Am 1994;76:129-138.
- 01.226 Item deleted after statistical review
- (and no answer or references cited)
- back
- next question
- 01.227 The process of host repair following osteonecrosis is referred to as
- 1- haversian remodeling.
- 2- osteogenesis.
- 3- osteoinduction.
- 4- fracture healing.
- 5- creeping substitution.
- answer
- back
- Question 01.227
- Answer = 5
- back to this question
- next question
- Reference(s)
- Buckwalter JA, Einhorn TA, Simon SR (eds)- Orthopaedic Basic Science: Biology and Biomechanics of the Musculoskeletal System, ed 2. Rosemont, IL, American Academy of Orthopaedic Surgeons, 2000, pp 372-399.
- 01.228 What type of knee dislocation is most likely to be irreducible by closed means?
- 1- Posterior
- 2- Anterior
- 3- Lateral
- 4- Posterolateral
- 5- Posteromedial
- answer
- back
- Question 01.228
- Answer = 4
- back to this question
- next question
- Reference(s)
- Kellam JF, Fischer TJ, Tornetta P III, Bosse MJ, Harris MB (eds): Orthopaedic Knowledge Update: Trauma 2. Rosemont, IL, American Academy of Orthopaedic Surgeons, 2000, pp 151-155. Quinlan AG, Sharrard WJW: Posterolateral dislocation of the knee with capsular interposition. J Bone Joint Surg Br 1958;40:660-663.
- 01.229 A 16-year-old boy who is 5 ft 4 in tall reports pain in the knees and ankles.
- Examination reveals that his knees are in 15° of valgus. The articular surfaces
- of the knees and ankles are irregular, and the femoral heads are slightly
- flattened. A lateral radiograph shows that the patellae have a double layer.
- What is the most likely diagnosis?
- 1- Kneist syndrome
- 2- Spondyloepiphyseal dysplasia congenita
- 3- Multiple epiphyseal dysplasia
- 4- Achondroplasia
- 5- Dyschondrosteosis
- answer
- back
- Question 01.229
- Answer = 3
- back to this question
- next question
- Reference(s)
- Dietz FR, Matthews KD: Update on the genetic bases of disorders with orthopaedic manifestations. J Bone Joint Surg Am 1996;78:1583-1598. Spranger J: The epiphyseal dysplasias. Clip Orthop 1976;114:46-59. Sheffield EG: Double-layered patella in multiple epiphyseal dysplasia: A valuable clue in the diagnosis. J Pediatr Orthop 1998;18:123-128.
- 01.230 What is the most common primary malignancy that metastasizes to the bones
- of the hand?
- 1- Breast
- 2- Prostate
- 3- Renal
- 4- Colon
- 5- Lung
- answer
- back
- Question 01.230
- Answer = 5
- back to this question
- next question
- Reference(s)
- American Society for Surgery of the Hand: Hand Surgery Update. Rosemont, IL, American Academy of Orthopaedic Surgeons, 1996, pp 369-376. Athanasian EA: Bone and soft-tissue tumors, in Green DP, Hotchkiss RN, Pederson WC (eds): Green's Operative Hand Surgery, ed 4. New York, NY, Churchill Livingstone, 1999, pp 2223-2253.
- 01.231 A 14-year-old girl has been limping and has had pain with weight bearing on
- the right lower leg for the past 48 hours. She has a temperature of 100°F
- (37.7°C). She prefers a prone position with the right hip and knee extended.
- Pain is produced by placing the hip in flexion, abduction, and external rotation.
- Which of the following studies will best confirm the diagnosis?
- 1- Hip joint aspiration
- 2- AP radiograph of the pelvis
- 3- Oblique radiograph of the lumbar spine
- 4- CT of the abdomen
- 5- MRI of the pelvis
- answer
- back
- Question 01.231
- Answer = 5
- back to this question
- next question
- Reference(s)
- Bollow M, Braun 1, Biedermann T, et al: Use of contrast-enhanced MR imaging to detect sacroiliitis in children. Skeletal Radiol 1998;27:606-616. Tisserant R, Loeuille D, Pere P, Gancher A, Ponrel J, Blum A: Septic sacroiliitis during the postpartal period: Diagnostic contribution of magnetic resonance imaging. Rev Rheum Engl Ed 1999;66:512-515.
- 01.232 A surgeon who is planning a total elbow arthroplasty would like to use a
- prosthesis that he helped to develop. A royalty payment is received each time
- the prosthesis is used. What is the surgeon's ethical responsibility?
- 1- The prosthesis should not be used.
- 2- The prosthesis can be used, but the royalty payment information should not be
- discussed with the patient.
- 3- The royalty payment information should be included on the informed consent
- document but not discussed with the patient.
- 4- The patient should be informed that the surgeon receives a royalty payment for using
- the prosthesis.
- 5- The surgeon should offer to split the royalty payment with the patient.
- answer
- back
- Question 01.232
- Answer = 4
- back to this question
- next question
- Reference(s)
- Wenger NS, Liu H, Lieberman JR: Teaching medical ethics to orthopaedic surgery residents. J Bone Joint Surg Am 1998;80:1125-1131.
- 01.233 In trauma patients older than age 60 years, mortality most closely correlates
- with
- 1- the injury severity score (ISS).
- 2- the extremity abbreviated injury score (AIS).
- 3- the need for orthopaedic surgery.
- 4- the timing of orthopaedic surgery.
- 5- a history of type I diabetes mellitus.
- answer
- back
- Question 01.233
- Answer = 1
- back to this question
- next question
- Reference(s)
- Tornetta P BI, Mostafavi H, Riina J, et al: Morbidity and mortality in elderly trauma patients. J Trauma 1999;46:702-706.
- 01.234 A 46-year-old woman has had plantar heel pain for the past 5 months. She
- reports that the pain is most severe when she arises out of bed in the morning
- and when she stands after being seated for a period of time. Initial management
- should consist of
- 1- surgical lengthening of the Achilles tendon.
- 2- surgical release of the plantar fascia.
- 3- a custom orthosis.
- 4- a stretching program and a cushioned heel insert.
- 5- a corticosteroid injection.
- answer
- back
- Question 01.234
- Answer = 4
- back to this question
- next question
- Reference(s)
- Pfeffer G, Bacchetti P, Deland J, et al: Comparison of custom and prefabricated orthoses in the initial treatment of proximal plantar fasciitis. Foot Ankle Int 1999;20:214-221. Richardson EG: Heel pain, in Coughlin MJ, Mann RA (eds): Surgery of the Foot and Ankle, ed 7. St Louis, MO, Harcourt Health Science, 1999, pp 1090-1209.
- 01.235 The triceps reflex is largely a function of what neurologic level?
- 1- C5
- 2- C6
- 3- C7
- 4- C8
- 5- T1
- answer
- back
- Question 01.235
- Answer = 3
- back to this question
- next question
- Reference(s)
- Snider RK (ed): Essentials of Musculoskeletal Care. Rosemont, IL, American Academy of Orthopaedic Surgeons, 1997, pp 491-546.
- 01.236 A 19-year-old female swimmer has had right shoulder pain for the past 5 years.
- Although she had responded previously to physical therapy, she has been in
- rehabilitation for the past 6 months without improvement. Examination reveals
- active total elevation of 170°, active external rotation of 70°, and passive
- internal rotation to T3. There is symmetric 2+ glenohumeral translation in the
- anterior, posterior, and inferior directions and a positive Neer impingement
- sign. Treatment should consist of
- 1- open Bankart repair.
- 2- an inferior capsular shift.
- 3- arthroscopic coracoacromial ligament resection.
- 4- arthroscopic acromioplasty.
- 5- arthroscopic Bankart repair.
- answer
- back
- Question 01.236
- Answer = 2
- back to this question
- next question
- Reference(s)
- Neer CS II, Foster CR: Inferior capsular shift for involuntary inferior and multi-directional instability of the shoulder. A preliminary report J Bone Joint Surg Am 1980;62:897-908. Schenk TJ, Brems JJ: Multi-directional instability of the shoulder Pathophysiology, diagnosis, and management. J Am Acad Orthop Surg 1998;6:65-72.
- 01.237 What is the most common complication at a minimum of 1 year after treatment
- with a reamed antegrade nail for a femoral shaft fracture?
- 1- Nonunion
- 2- Malunion
- 3- Trendelenburg gait
- 4- Hip discomfort
- 5- Osteonecrosis of the femoral head
- answer
- back
- Question 01.237
- Answer = 4
- back to this question
- next question
- Reference(s)
- Bain GI, Zacest AC, Paterson DC, Middleton J, Pohl AP: Abduction strength following intramedullary nailing of the femur. J Orthop Trauma 1997;11:93-97.
- 01.238 A 13-year-old boy has nonrigid Scheuermann's kyphosis. Weight-bearing
- radiographs show a kyphosis of 70° from T7 to L1, with scoliosis that
- measures 10° at Risser 2 maturity. Management should consist of
- 1- postural exercises and analgesics.
- 2- a Charleston bending brace.
- 3- an extension-type spinal orthosis.
- 4- posterior spinal fusion with instrumentation.
- 5- anterior spinal release and posterior spinal instrumentation.
- answer
- back
- Question 01.238
- Answer = 3
- back to this question
- next question
- Reference(s)
- Lowe TG: Scheuermann disease. J Bone Joint Surg Am 1990;72:940-945. Tribes CB: Scheuermann's kyphosis in adolescents and adults: Diagnosis and management. J Am Acad Orthop Surg 1998;6:36-43.
- 01.239 A 21-year-old male wrestler sustained a right posterolateral elbow dislocation
- with an associated type I coronoid fracture 2 years ago. Management at the
- time of injury consisted of application of a splint for 2 weeks. He now reports
- recurrent elbow subluxation and pain. What is the most likely cause of the
- instability?
- 1- Displaced coronoid process fracture
- 2- Insufficiency of the lateral ulnar collateral ligament
- 3- Insufficiency of the anterior band of the medial collateral ligament
- 4- Insufficiency of the posterior band of the medial collateral ligament
- 5- Anterior capsular insufficiency
- answer
- back
- Question 01.239
- Answer = 2
- back to this question
- next question
- Reference(s)
- Josefsson PO, Johnell O, Gentz CF: Long-term sequelae of simple dislocation of the elbow. J Bone Joint Surg Am 1984;66:927-930. Nestor BJ, O'Driscoll SW, Morrey BF: Ligamentous reconstruction for posterolateral rotatory instability of the elbow. J Bone Joint Surg Am 1992;74:1235-1241. O'Driscoll SW, Money BF, Korinek S, An KN: Elbow subluxation and dislocation: A spectrum of instability. Clin Orthop 1992;280:186-197.
- 01.240 The degree of ulnar variance is best defined by
- 1- arthrography.
- 2- MRI.
- 3- cineradiographs.
- 4- stress radiographs.
- 5- plain radiographs.
- answer
- back
- Question 01.240
- Answer = 5
- back to this question
- next question
- Reference(s)
- Nagle DJ: Evaluation of chronic wrist pain. J Am Acad Orthop Surg 2000;8:45-55. Epner RA, Bowers WH, Guilford WB: Ulnar variance: The effect of wrist positioning and roentgen filming technique. J Hand Surg Am 1982;7:298-305.
- 01.241 An 18-year-old woman has had left hip pain for the past 2 months.
- Examination reveals audible snapping with extension of a flexed, abducted,
- and externally rotated hip. What study is most likely to establish the diagnosis?
- 1- Bone scan
- 2- Iliopsoas bursography
- 3- Plain radiography
- 4- Hip arthrography
- 5- Hip arthroscopy
- answer
- back
- Question 01.241
- Answer = 2
- back to this question
- next question
- Reference(s)
- Schaberg JF, Harper MC, Allen WC: The snapping hip syndrome. Am J Sports Med 1984;12:361-365. Jacobson T, Allen WC: Surgical correction of the snapping iliopsoas tendon. Am J Sports Med 1990;18:470-474.
- 01.242 What metabolic bone disease is associated with abnormal osteoclastic
- function?
- 1- X-linked hypophosphatemic rickets
- 2- Fanconi's syndrome
- 3- Osteopetrosis
- 4- Osteomalacia
- 5- Paget's disease of bone
- answer
- back
- Question 01.242
- Answer = 3
- back to this question
- next question
- Reference(s)
- Beaty JH (ed): Orthopaedic Knowledge Update 6. Rosemont, IL. American Academy of Orthopaedic Surgeons, 1999, pp 149-165. Shapiro F: Osteopetrosis: Current clinical considerations. Clin Orthop 1993;294:344.
- 01.243 The Glasgow Coma Scale categorizes the neurologic status of a multiply
- injured patient by assessing verbal response, motor response, and
- 1- orientation.
- 2- response to commands.
- 3- pupillary response.
- 4- withdrawal to pain.
- 5- eye opening response.
- answer
- back
- Question 01.243
- Answer = 5
- back to this question
- next question
- Reference(s)
- Turen CH, Dube MA, LeCroy MC: Approach to the polytraumatized patient with musculoskeletal injuries. J Am Acad Orthop Surg 1999;7:154-165. Teasdale G, Jennett B: Assessment of coma and impaired consciousness: A practical scale. Lancet 1974;2:81-84.
- 01.244 Which of the following is considered the preferred total knee design for a
- patient with a history of a patellectomy?
- 1- Posterior cruciate ligament-retaining
- 2- Posterior cruciate ligament-substituting
- 3- Rotating hinge
- 4- Unicondylar
- 5- Meniscal bearing
- answer
- back
- Question 01.244
- Answer = 2
- back to this question
- next question
- Reference(s)
- Beaty JH (ed): Orthopaedic Knowledge Update 6. Rosemont, IL, American Academy of Orthopaedic Surgeons, 1999, pp 559-582. Paletta GA Jr, Laskin RS: Total knee arthroplasty after a previous patellectomy. J Bone Joint Surg Am 1995;77:1708-1712.
- 01.245 A child with chronic recurrent multifocal osteomyelitis has painful swelling
- and tenderness in the right medial clavicle with no fluctuance. She has a
- temperature of 99°F (37.2°C). The palms and soles show pustular lesions.
- Radiographs reveal periosteal new bone formation in the medial clavicle.
- Management should consist of
- 1- a steroid injection into the medial clavicle.
- 2- oral nonsteroidal anti-inflammatory drugs.
- 3- IV administration of oxacillin for 4 weeks.
- 4- IV administration of gamma globulin.
- 5- incision and drainage of the medial clavicle.
- answer
- back
- Question 01.245
- Answer = 2
- back to this question
- next question
- Reference(s)
- Godette GA, Murray DP, Gruel CR, Leonard 1C: Chronic recurrent multifocal osteomyelitis. Orthopedics 1992;15:520-521, 525-526. Bjorksten B, Gustavson K-H, Eriksson B, Lindholm A, Nordstrom S: Chronic recurrent multifocal osteomyelitis and pustulosis palmoplantaris. J Pediatr 1978;93:227-231. Stanton RP, Lopez-Sosa FH, Doidge R: Chronic recurrent multifocal osteomyelitis. Orthop Rev 1993;22:229-233.
- 01.246 A 32-year-old woman reports right shoulder pain and has difficulty with
- overhead activities. History reveals that she underwent an open anterior labral
- repair and capsular shift to treat anterior glenohumeral instability 3 years ago.
- Examination reveals tenderness over the anterior shoulder, active and passive
- total elevation of 120°, and external rotation of 30°. Shoulder strength is
- normal. Plain radiographs are normal. Physical therapy has failed to provide
- relief. Treatment should now consist of
- 1- arthroscopic acromioplasty.
- 2- biceps tenodesis.
- 3- open subscapularis lengthening and capsular release.
- 4- humeral head replacement.
- 5- derotational humeral osteotomy.
- answer
- back
- Question 01.246
- Answer = 3
- back to this question
- next question
- Reference(s)
- Bigliani LU: Glenohumeral instability repairs: Complications and failures, in Bigliani LU (ed): The Unstable Shoulder. Rosemont, IL, American Academy of Orthopaedic Surgeons, 1996, pp 99-106. MacDonald PB, Hawkins RJ, Fowler PJ, Miniaci A: Release of the subscapularis for internal rotation contracture and pain after anterior repair for recurrent anterior dislocation of the shoulder. J Bone Joint Surg Am 1992;74:734-737.
- 01.247 What statistical test should be used to determine whether a significant
- difference exists between the means of more than two independent samples
- with normal distributions?
- 1- Student's t test
- 2- Analysis of variance
- 3- Regression analysis
- 4- Chi-square test
- 5- Kruskal-Wallis test
- answer
- back
- Question 01.247
- Answer = 2
- back to this question
- next question
- Reference(s)
- Simon SR (ed): Orthopaedic Basic Science. Rosemont, IL, American Academy of Orthopaedic Surgeons, 1994, pp 623-665. Freedman KB, Bernstein J: Sample size and statistical power in clinical orthopaedic research. J Bone Joint Surg Am 1999;81:1454-1460.
- 01.248 A female patient is most likely the victim of domestic abuse when the
- 1- injury is inconsistent with the offered explanation.
- 2- spouse does not express any interest in the patient's injuries.
- 3- patient expresses an overly animated affect.
- 4- patient has a lower socioeconomic status.
- 5- patient is eager to leave the hospital or clinic.
- answer
- back
- Question 01.248
- Answer = 1
- back to this question
- next question
- Reference(s)
- Zillmer DA: Domestic violence: The role of the orthopaedic surgeon in identification and treatment. J Am Acad Orthop Surg 2000;8:91-96.
- 01.249 A varus malreduction of a comminuted talar neck fracture will result in
- 1- varus hindfoot and decreased subtalar motion.
- 2- increased contact loading of the posterior facet.
- 3- subtalar instability.
- 4- anterior ankle impingement.
- 5- talonavicular subluxation.
- answer
- back
- Question 01.249
- Answer = 1
- back to this question
- next question
- Reference(s)
- Daniels TR, Smith JW, Ross TI: Varus malalignment of the talar neck: Its effect in the position of the foot and on subtalar motion. J Bone Joint Surg Am 1996;78:1559-1567. Sangeorzan BJ, Wagner UA, Harrington RIM, Tencer AF: Contact characteristics of the subtalar joint: The effect of talar neck misalignment. J Orthop Res 1992;10:544.-551.
- 01.250 While performing a revision total knee replacement with a trial component in
- place, it is noted that the knee has full extension but is loose in flexion. To
- resolve this flexion-extension discrepancy, the surgeon should
- 1- use a thicker polyethylene insert.
- 2- use a larger femoral component with posterior condyle metallic wedges.
- 3- use a more constrained polyethylene insert.
- 4- release the posterior capsule.
- 5- cut more posterior slope on the tibia.
- answer
- back
- Question 01.250
- Answer = 2
- back to this question
- next question
- Reference(s)
- Callaghan JJ, Dennis DA, Paprosky WG, Rosenberg AG (eds): Orthopaedic Knowledge Update: Hip and Knee Reconstruction. Rosemont, IL, American Academy of Orthopaedic Surgeons, 1995, pp 323-327.
- 01.251 During harvest of an anterior iliac crest bone graft, what nerve is at greatest
- risk for injury?
- 1- Lateral femoral cutaneous
- 2- Inguinal
- 3- Genitofemoral
- 4- Ilioinguinal
- 5- Femoral
- answer
- back
- Question 01.251
- Answer = 1
- back to this question
- next question
- Reference(s)
- Bridwell KH, DeWald RL (eds): The Textbook of Spinal Surgery, ed 2. Philadelphia, PA, Lippincott-Raven, 1997, p 1736.
- 01.252 What structure is at greatest risk for injury when the anterolateral portal is used
- for ankle arthroscopy?
- 1- Superficial peroneal nerve
- 2- Saphenous nerve
- 3- Sural nerve
- 4- Deep peroneal nerve
- 5- Peroneal artery
- answer
- back
- Question 01.252
- Answer = 1
- back to this question
- next question
- Reference(s)
- Ferkel RD: Arthroscopy of the foot and ankle, in Coughlin MJ, Mate RA (eds): Surgery of the Foot and Ankle, ed 7. St Louis, MO, Harcourt Health Science, 1999, pp 1257-1268.
- 01.253 A 53-year-old patient sustains a traumatic anterior dislocation of the
- glenohumeral joint. The glenohumeral joint is reduced, and postreduction
- radiographs show a concentric reduction and no evidence of fracture. One
- week later, the patient cannot actively abduct his arm; however, passive
- abduction is normal. What is the most likely cause for the lack of active
- shoulder abduction?
- 1- Axillary nerve injury
- 2- Brachial plexus injury
- 3- Deltoid muscle avulsion
- 4- Rotator cuff tear
- 5- Glenoid labral tear
- answer
- back
- Question 01.253
- Answer = 4
- back to this question
- next question
- Reference(s)
- Stayner LR, Cummings J, Andersen J, Jobe CM: Shoulder dislocations in patients older than 40 years of age. Orthop Clin North Am 2000;31:231-239. Hawkins RJ, Bell RH, Hawkins RH, Koppert GJ: Anterior dislocation of the shoulder in the older patient. Clin Orthop 1986;206:192-195.
- 01.254 What patient-related risk factor is associated with an increased risk of
- dislocation in total hip arthroplasty?
- 1- Obesity
- 2- Smoking
- 3- Alcohol intake
- 4- Male gender
- 5- Use of systemic steroids
- answer
- back
- Question 01.254
- Answer = 3
- back to this question
- next question
- Reference(s)
- Espehaug B, Havelin LL Engesaester LB, Langeland N, Vollset SE: Patient related risk factors for early revision of total hip replacements: A population register-based case-control study of 674 revised hips. Acts Orthop Scand 1997;68:207-215. Beaty JH (ed): Orthopaedic Knowledge Update 6. Rosemont, IL, American Academy of Orthopaedic Surgeons, 1999, pp 455-492.
- 01.255 The poor results following surgical treatment of posterior wall acetabular
- fractures are most commonly associated with
- 1- sciatic nerve injury.
- 2- articular comminution.
- 3- heterotopic ossification.
- 4- deep venous thrombosis.
- 5- osteonecrosis of the femoral head.
- answer
- back
- Question 01.255
- Answer = 2
- back to this question
- next question
- Reference(s)
- Browner BD, Jupiter JB, Levine AM Trafton PB (eds): Skeletal Trauma, ed 2. Philadelphia, PA, WB Saunders 1998, pp 1204-1208. Matta JM: Fractures of the acetabulum: Accuracy of reduction and clinical results in patients managed operatively within three weeks after the injury. J Bone Joint Surg Am 1996;78:1632-1645. Saterbak AM, Marsh JL, Nepola JV, Brandser EA, Turbett T: Clinical failure after posterior wall acetabular fractures: The influence of initial fracture patterns. J Orthop Trauma 2000;14:230-237.
- 01.256 Which of the following conditions is considered a common clinical
- manifestation of multiple hereditary exostoses?
- 1- Radial bowing
- 2- Scoliosis
- 3- Acetabular dysplasia
- 4- Genu varum
- 5- Dwarfism
- answer
- back
- Question 01.256
- Answer = 1
- back to this question
- next question
- Reference(s)
- Stanton RP, Hansen MO: Function of the upper extremities in hereditary multiple exostoses. J Bone Joint Surg Am 1969;78:68-573. Arms DM, Strecker WB, Manske PR, Schoenecker PL: Management of forearm deformity in multiple hereditary osteochondromatosis. J Pediatr Orthop 1997;17:450-454. Schmale GA, Conrad EU III, Raskind WH: The natural history of hereditary multiple exostoses. J Bone Joint Surg Am 1994;76:986-992.
- 01.257 When comparing women who sustained a pelvic ring fracture with women
- who have multiple injuries without a pelvic ring fracture, those with a pelvic
- fracture have been found to have a higher subsequent rate of
- 1- miscarriage.
- 2- infertility.
- 3- depression.
- 4- failure to achieve physiologic sexual arousal.
- 5- urinary difficulties.
- answer
- back
- Question 01.257
- Answer = 5
- back to this question
- next question
- Reference(s)
- Copeland CE, Bosse MJ, McCarthy ML, et al: Effect of trauma and pelvic fracture on female genitourinary, sexual, and reproductive function. J Orthop Trauma 1997;11:73-81.
- 01.258 When performing palmar fasciectomy for Dupuytren's contracture, what other
- procedure should not be performed at the same time?
- 1- Trigger finger release
- 2- Intraoperative digital nerve laceration repair
- 3- Knuckle pad excision
- 4- Proximal interphalangeal joint arthrodesis
- 5- Carpal tunnel release
- answer
- back
- Question 01.258
- Answer = 5
- back to this question
- next question
- Reference(s)
- American Society for Surgery of the Hand: Hand Surgery Update. Rosemont, IL, American Academy of Orthopaedic Surgeons, 1996, pp 271-279. Nissenbaum M, Kleinert HE: Treatment considerations in carpal tunnel syndrome with coexistent Dnpuytren's disease. J Hand Surg Am 1980;5:544-547.
- 01.259 A 44-year-old man has persistent anteromedial joint line pain after sustaining
- multiple ankle sprains. At the time of surgery, thickening of the deltoid
- ligament on its most anterior aspect is noted. What fascicle of the deltoid
- ligament is involved with this anterior impingement?
- 1- Anterior tibial
- 2- Anterior tibiotalar
- 3- Tibionavicular
- 4- Tibiocalcaneal
- 5- Talonavicular
- answer
- back
- Question 01.259
- Answer = 2
- back to this question
- next question
- Reference(s)
- Egol KA, Parisian JS: Impingement syndrome of the ankle caused by a medial meniscoid lesion. Arthroscopy 1997;13:522-525. Mosier-La Clair SM, Monroe MT, Manoli A: Medial impingement syndrome of the anterior tibiotalar fascicle of the deltoid ligament on the talus. Foot Ankle Int 2000;21:385-391.
- 01.260 What metabolic bone disease is associated with the presence of virus-like
- inclusion bodies found in the osteoclast?
- 1- X-linked hypophosphatemic rickets
- 2- Fanconi's syndrome
- 3- Osteopetrosis
- 4- Osteomalacia
- 5- Paget's disease of bone
- answer
- back
- Question 01.260
- Answer = 5
- back to this question
- next question
- Reference(s)
- Beaty 1H (ed): Orthopaedic Knowledge Update 6. Rosemont, IL, American Academy of Orthopaedic Surgeons, 1999, pp 149-165. Hadjipavlou A, Lander P: Paget disease of the spine. J Bone Joint Surg Am 1991;73:1376-1381. Delmas PD, Mennier P1: The management of Paget's disease of bone. N Engl J Med 1997;336:58-566.
- 01.261 Pseudoachondroplasia, characterized by disproportionate short-limbed
- dwarfism and ligamentous laxity, is caused by a deletion or alteration in the
- gene encoding what protein?
- 1- Fibroblast growth factor receptor
- 2- Cartilage oligomeric matrix protein
- 3- Type 11 collagen
- 4- Type IX collagen
- 5- Parathyroid hormone receptor
- answer
- back
- Question 01.261
- Answer = 2
- back to this question
- next question
- Reference(s)
- Buckwalter JA, Einhorn TA, Simon SR (eds): Orthopaedic Basic Science: Biology and Biomechanics of the Musculoskeletal System, ed 2. Rosemont, IL, American Academy of Orthopaedic Surgeons, 2000, pp 112-131.
- 01.262 Semmes-Weinstein monofilaments are used to test the foot for abnormal
- sensory threshold in patients with diabetes mellitus. Loss of protective
- sensation is the inability to feel
- 1- the 4.17 filament.
- 2- the 5.07 filament.
- 3- the 6.10 filament.
- 4- 5 g of pressure.
- 5- 15 g of pressure.
- answer
- back
- Question 01.262
- Answer = 2
- back to this question
- next question
- Reference(s)
- Jeng C, Michelson J, Mizel M: Sensory thresholds of normal human feet. Foot Ankle Int 2000;21:501-504. Rith-Najarian SJ, Stolusky T, Gohdes DM: Identifying diabetic patients at him risk for lower-extremity amputation in a primary health care setting: A prospective evaluation of simple screening criteria. Diabetes Care 1992;15:1386-1389.
- 01.263 A 13-year-old girl with scoliosis has mild intermittent back pain. A bone scan,
- CT scan, and an MRI scan would most likely reveal which of the following
- conditions?
- 1- Spondylolysis
- 2- Spondylolisthesis
- 3- Scheuermann's disorder
- 4- Slipped vertebral apophysis
- 5- No other condition
- answer
- back
- Question 01.263
- Answer = 5
- back to this question
- next question
- Reference(s)
- Ramirez N, Johnston CE, Browne RH: The prevalence of back pain in children who have idiopathic scoliosis. J Bone Joint Surg Am 1997;79:364-368.
- 01.264 What is the most common cause of failure of the pars interarticularis in
- spondylolysis?
- 1- Repetitive hyperextension
- 2- Repetitive axial loading
- 3- Repetitive torsion
- 4- Single-load extension
- 5- Single-load flexion
- answer
- back
- Question 01.264
- Answer = 1
- back to this question
- next question
- Reference(s)
- Bradford D.S: Spondylolysis and spondylolisthesis in children and adolescents: Current concepts in management, in Bradford DS, Hensinger RM (eds): The Pediatric Spine. New York, NY, Thieme, 1985, pp 403-423. Bridwell KH, DeWald RL (eds): The Textbook of Spinal Surgery, ed 2. Philadelphia, PA, Lippincott-Raven, 1997, pp 1337-1347.
- 01.265 What is the advantage of impaction allografting during femoral revision hip
- arthroplasty?
- 1- Lower cost
- 2- Lower incidence of femoral component subsidence
- 3- Lower risk of femoral fracture and perforation
- 4- Ability to reconstitute bone stock
- 5- Delivery of depot antibiotics
- answer
- back
- Question 01.265
- Answer = 4
- back to this question
- next question
- Reference(s)
- Leopold SS, Rosenberg AG: Current status of impaction allografting for revision of a femoral component. Inst Course Lect 2000;49:111-118.
- 01.266 Which of the following is considered a characteristic of a prosthesis used for a
- Syme's amputation?
- 1- No auxiliary suspension mechanisms
- 2- Posterior window
- 3- Non-weight-bearing distal portion
- 4- Patellar tendon bearing
- 5- Availability of multiple foot prostheses
- answer
- back
- Question 01.266
- Answer = 1
- back to this question
- next question
- Reference(s)
- Coughlin MJ, Mann RA: Surgery of the Foot and Ankle, ed 7. St Louis, MO, Mosby, 1999, pp 1003-1004.
- 01.267 A 62-year-old woman who plays tennis underwent an acromioplasty and
- rotator cuff repair using four suture anchors 1 month ago. Three days ago, she
- was allowed to begin using her arm to lift light weights, but she now reports a
- dramatic increase in pain and is unable to elevate the arm. The next most
- appropriate step in management should consist of
- 1- further immobilization and discontinuation of physical therapy.
- 2- plain radiography.
- 3- MRI.
- 4- arthrography.
- 5- electromyography of the axillary and suprascapular nerves.
- answer
- back
- Question 01.267
- Answer = 2
- back to this question
- next question
- Reference(s)
- Hanyman DT II: Mack LA, Wang KY, Jackins SE, Richardson ML, Matsen FA III: Repairs of the rotator cuff: Correlation of functional results with integrity of the cuff. J Bone Joint Surg Am 1991;73:982-989. Barber FA, Herbert MA, Click JN: The ultimate strength of suture anchors. Arthroscopy 1995;11:21-28.
- 01.268 Which of the following factors will increase the rigidity of an external fixator?
- 1- Decreased pin diameter
- 2- Decreased pin number
- 3- Increased pin spread within a segment
- 4- Increased bone-to-rod distance
- 5- Increased distance between fragment pin sets
- answer
- back
- Question 01.268
- Answer = 3
- back to this question
- next question
- Reference(s)
- Buckwalter JA. Einhorn TA, Simon SR (eds): Orthopaedic Basic Science: Biology and Biomechanics of the Musculoskeletal System, ed 2. Rosemont, IL, American Academy of Orthopaedic Surgeons, 2000, pp 372-399.
- 01.269 The likelihood of a contralateral slip occurring in a boy with a unilateral
- slipped capital femoral epiphysis is greatest when combined with which of the
- following risk factors?
- 1- An unstable slip
- 2- A chronic slip
- 3- A grade III slip
- 4- Patient age of 11 years
- 5- Patient age of 13 years
- answer
- back
- Question 01.269
- Answer = 4
- back to this question
- next question
- Reference(s)
- Stasikelis PJ, Sullivan CM, Phillips WA, Polard JA: Slipped capital femoral epiphysis: Prediction of contralateral involvement. J Bone Joint Surg Am 1996;78:1149-1155. Loder RT, Aronson DD, Greenfield ML: The epidemiology of bilateral slipped capital femoral epiphysis: A study of children in Michigan. J Bone Joint Surg Am 1993;75:1141-1147.
- 01.270 During intramedullary tibial nailing, compartment pressures in the leg are most
- elevated by the use of
- 1- reaming.
- 2- continuous traction.
- 3- a medial starting point.
- 4- a lateral starting point.
- 5- a solid nail.
- answer
- back
- Question 01.270
- Answer = 2
- back to this question
- next question
- Reference(s)
- Shakespeare DT, Henderson NJ: Compartmental pressure changes during calcaneal traction in tibial fractures. J Bone Joint Surg Br 1982;64:498-499. McQueen MM, Christie J, Court-Brown CM: Compartment pressures after intramedullary nailing of the tibia. J Bone Joint Surg Br 1990;72:395-397.
- 01.271 Surgical intervention is first indicated for Dupuytren's disease when which of
- the following findings is present?
- 1- A metacarpophalangeal joint contracture that is greater than 60° and a proximal
- interphalangeal joint contracture of any degree
- 2- A metacarpophalangeal joint contracture and a proximal interphalangeal joint
- contracture that are each greater than 40°
- 3- A metacarpophalangeal joint contracture of 30° and a proximal interphalangeal joint
- contracture of any degree
- 4- A metacarpophalangeal joint contracture of any degree and a proximal
- interphalangeal joint contracture that is greater than or equal to 30°
- 5- Any contracture of either the metacarpophalangeal or proximal interphalangeal joints
- answer
- back
- Question 01.271
- Answer = 3
- back to this question
- next question
- Reference(s)
- McFarlane RM, Botz JS: The results of treatment, in McFarlane RM, McGrouther DA, Flint MA (eds): Dupuytren's Disease: Biology and Treatment (The hand and upper limb series, vol 5). Edinburgh, Scotland, 1990, pp 387-412.
- 01.272 Following a left-sided approach for surgery on the anterior cervical spine, the
- patient reports a drooping left upper eyelid and dryness on the left side of the
- face. Which of the following structures has most likely been injured?
- 1- Recurrent laryngeal nerve
- 2- Superior laryngeal nerve
- 3- Hypoglossal nerve
- 4- Phrenic nerve
- 5- Sympathetic chain
- answer
- back
- Question 01.272
- Answer = 5
- back to this question
- next question
- Reference(s)
- Bridwell KH, DeWald RL (eds): The Textbook of Spinal Surgery, ed 2. Philadelphia, PA, Lippincott-Raven, 1997, pp 1427-1438. Flynn TB: Neurologic complications of anterior cervical interbody fusion. Spine 1982;7:536-539.
- 01.273 An 18-year-old man with recurrent bilateral ankle instability reports that his
- symptoms have been slowly progressing. He denies problems with
- coordination, but he notes easy fatigability when he types. History reveals that
- his grandmother had "disfigured feet." Examination reveals bilateral cavovarus
- feet and peroneal weakness. He has intrinsic muscular weakness in his hands.
- To confirm the diagnosis, which of the following studies should be obtained?
- 1- Radiographs of the spine
- 2- Electrocardiography
- 3- Electromyography
- 4- MRI of the feet
- 5- CBC
- answer
- back
- Question 01.273
- Answer = 3
- back to this question
- next question
- Reference(s)
- Beaty JH (ed): Orthopaedic Knowledge Update 6. Rosemont, IL, American Academy of Orthopaedic Surgeons, 1999, pp 235-245. Mizel MS, Miller RA, Scioli MW (eds): Orthopaedic Knowledge Update: Foot and Ankle 2. Rosemont, IL, American Academy of Orthopaedic Surgeons, 1998, pp 79-100.
- 01.274 Which of the following is considered the most common complication of an
- extensive medial release for resistant medial epicondylitis?
- 1- Ulnar palsy
- 2- Medial elbow instability
- 3- Wrist flexion weakness
- 4- Forearm pronation weakness
- 5- Elbow flexor weakness
- answer
- back
- Question 01.274
- Answer = 2
- back to this question
- next question
- Reference(s)
- Callaway GH, Field LD, Deng ML et al: Biomechanical evaluation of the medial collateral ligament of the elbow. J Bone Joint Surg Am 1997;79:1223-1231. Vangsness CT Jr, Jobe FW: Surgical treatment of medial epicondylitis: Results in 35 elbows. J Bone Joint Surg Br 1991;73:409-411.
- 01.275 What neurosensory receptor is responsible for detecting the sensation of a
- vibration?
- 1- Merkel cells
- 2- Pacinian corpuscles
- 3- Ruffini end organs
- 4- Meissner corpuscles
- 5- Free-ending nerve fibers
- answer
- back
- Question 01.275
- Answer = 3
- back to this question
- End of 2001 Exam
- Reference(s)
- Simon SR (ed): Orthopaedic Basic Science. Rosemont, IL, American Academy of Orthopaedic Surgeons, 1994, pp 325-396.
for more mcqs in the bank